371 Exam 4

Lakukan tugas rumah & ujian kamu dengan baik sekarang menggunakan Quizwiz!

1. Vasopressin (Pitressin) 0.2 units/min infusion is prescribed for a patient with acute arterial gastrointestinal (GI) bleeding. The vasopressin label states vasopressin 20 units/50 mL normal saline. How many mL/min will the nurse infuse? ____________________

0.5 There are 0.4 units/1 mL. An infusion of 0.5 mL/min will result in the patient receiving 0.2 mL/min as prescribed. DIF: Cognitive Level: Application REF: 982-983 OBJ: Special Questions: Alternate Item Format TOP: Nursing Process: Implementation

1. Vasopressin 0.1 unit/min infusion is prescribed for a patient with acute arterial gastrointestinal (GI) bleeding. The vasopressin label states vasopressin 100 units/250 mL normal saline. How many mL/hr will the nurse infuse?

15 There are 0.4 unit/1 mL. An infusion of 15 mL/hr will result in the patient receiving 0.1 units/min as prescribed. REF: 923 1. The nurse is caring for a patient who develops watery diarrhea and a fever after prolonged omeprazole (Prilosec) therapy. In which order will the nurse take actions? (Put a comma and a space between each answer choice [A, B, C, D].) a. Contact the health care provider. b. Assess blood pressure and heart rate. c. Give the PRN acetaminophen (Tylenol). d. Place the patient on contact precautions. ANS: D, B, A, C Proton pump inhibitors including omeprazole (Prilosec) may increase the risk of Clostridium difficile-associated colitis. Because the patient's history and symptoms are consistent with C. difficile infection, the initial action should be initiation of infection control measures to protect other patients. Assessment of blood pressure and pulse is needed to determine whether the patient has symptoms of hypovolemia or shock. The health care provider should be notified so that actions such as obtaining stool specimens and antibiotic therapy can be started. Tylenol may be administered but is the lowest priority of the actions. DIF: Cognitive Level: Analyze (analysis) REF: 902 OBJ: Special Questions: Prioritization TOP: Nursing Process: Implementation

2. A client wants to lose 1.5 pounds a week. After reviewing a diet history, the nurse determines the client typically eats 2450 calories a day. What should the client's calorie goal be to achieve this weight loss? (Record your answer using a whole number.) __ calories/day

1700 calories/day To encourage a weight loss of 1 pound (2.2 kg) a week, 500 calories per day would be subtracted. To encourage a weight loss of 2 pounds (4.4 kg) a week, 1000 calories each day are subtracted. In this scenario, to lose 1.5 pounds a week the client needs to cut 750 calories per day from the diet: 2450 - 750 = 1700 calories. DIF: Applying/Application REF: 1249 KEY: Nutritional disorders| nutritional assessment

3. A client is receiving continuous tube feeding at 70 mL/hr. When the bag is empty, how much formula does the nurse add? (Record your answer using a whole number.) _____ mL

280 mL The nurse never adds more than 4 hours' worth of formula to a hanging bag of enteral feedings. 70 mL/hr × 4 hr = 280 mL. DIF: Applying/Application REF: 1242 KEY: Nutritional disorders| tube feedings

1. A client weighs 228 pounds (103.6 kg) and is 5'3" (160 cm) tall. What is this client's body mass index (BMI)? (Record your answer using a decimal rounded up to the nearest tenth.) _____

40.4 Using the formula : , or 40.4 rounded up to the nearest tenth. DIF: Applying/Application REF: 1236 KEY: Nutritional assessment

1. A nurse prepares to administer 12 mg/kg of 5-fluorouracil chemotherapy intravenously to a client who has colon cancer. The client weights 132 lb. How many milligrams should the nurse administer? (Record your answer using a whole number.) _____ mg

720 mg 132 lb = 60 kg.. 60 kg × 12 mg/kg = 720 mg. DIF: Applying/Application REF: 1152 KEY: Medication safety

50. Which menu choice by the patient with diverticulosis is best for preventing diverticulitis? a. Navy bean soup and vegetable salad b. Whole grain pasta with tomato sauce c. Baked potato with low-fat sour cream d. Roast beef sandwich on whole wheat bread

A A diet high in fiber and low in fats and red meat is recommended to prevent diverticulitis. Although all of the choices have some fiber, the bean soup and salad will be the highest in fiber and the lowest in fat. DIF: Cognitive Level: Analyze (analysis) REF: 964 OBJ: Special Questions: Prioritization TOP: Nursing Process: Evaluation

11. Which topic would be of importance for the nurse to include when teaching a patient about testing for possible metabolic syndrome? a. Blood glucose test b. Cardiac enzyme tests c. Postural blood pressures d. Resting electrocardiogram

A A fasting blood glucose test >100 mg/dL is one of the diagnostic criteria for metabolic syndrome. The other tests are not used to diagnose metabolic syndrome, although they may be used to check for cardiovascular complications of the disorder. DIF: Cognitive Level: Application REF: 959-960 TOP: Nursing Process: Implementation

9. When teaching a patient about testing to diagnose metabolic syndrome, which topic would the nurse include? a. Blood glucose test c. Postural blood pressures b. Cardiac enzyme tests d. Resting electrocardiogram

A A fasting blood glucose test greater than 100 mg/dL is one of the diagnostic criteria for metabolic syndrome. The other tests are not used to diagnose metabolic syndrome, but they may be used to check for cardiovascular complications of the disorder. DIF: Cognitive Level: Apply (application) REF: 890 TOP: Nursing Process: Implementation

46. A patient with a gunshot wound to the abdomen undergoes surgery, and a colostomy is formed as illustrated. Which information will be included in patient teaching? a. This type of colostomy is usually temporary. b. Soft, formed stool can be expected as drainage. c. Stool will be expelled from both ostomy stomas. d. Irrigations can regulate drainage from the stomas.

A A loop, or double-barrel stoma, is usually temporary. Stool will be expelled from the proximal stoma only. The stool from the transverse colon will be liquid and regulation through irrigations will not be possible. DIF: Cognitive Level: Application REF: 1040-1041 | 1042 | 1044-1045 TOP: Nursing Process: Implementation

25. The nurse admitting a patient with acute diverticulitis explains that the initial plan of care is to a. administer IV fluids. b. prepare for colonoscopy. c. give stool softeners and enemas. d. order a diet high in fiber and fluids.

A A patient with acute diverticulitis will be NPO and given parenteral fluids. A diet high in fiber and fluids will be implemented before discharge. Bulk-forming laxatives, rather than stool softeners, are usually given, and these will be implemented later in the hospitalization. The patient with acute diverticulitis will not have enemas or a colonoscopy because of the risk for perforation and peritonitis. DIF: Cognitive Level: Apply (application) REF: 964 TOP: Nursing Process: Planning

30. A patient calls the clinic and tells the nurse about a new onset of severe and frequent, diarrhea. The nurse anticipates that the patient will need to a. collect a stool specimen. b. prepare for colonoscopy. c. schedule a barium enema. d. have blood cultures drawn.

A Acute diarrhea is usually caused by an infectious process and stool specimens are obtained for culture and examined for parasites or white blood cells. There is no indication that the patient needs a colonoscopy, blood cultures, or a barium enema. DIF: Cognitive Level: Application REF: 1007-1008 TOP: Nursing Process: Planning

29. A patient calls the clinic to report a new onset of severe diarrhea. The nurse anticipates that the patient will need to a. collect a stool specimen. c. schedule a barium enema. b. prepare for colonoscopy. d. have blood cultures drawn.

A Acute diarrhea is usually caused by an infectious process, and stool specimens are obtained for culture and examined for parasites or white blood cells. There is no indication that the patient needs a colonoscopy, blood cultures, or a barium enema. DIF: Cognitive Level: Apply (application) REF: 931 TOP: Nursing Process: Planning

6. When admitting a patient with acute glomerulonephritis, it is most important that the nurse ask the patient about a. recent sore throat and fever. b. history of high blood pressure. c. frequency of bladder infections. d. family history of kidney stones.

A Acute glomerulonephritis frequently occurs after a streptococcal infection such as strep throat. It is not caused by hypertension, urinary tract infection (UTI), or kidney stones. DIF: Cognitive Level: Application REF: 1131-1132 TOP: Nursing Process: Assessment

24. A patient with a new ileostomy asks how much drainage to expect. The nurse explains that after the bowel adjusts to the ileostomy, the usual drainage will be about _____ cups daily. a. 2 c. 4 b. 3 d. 5

A After the proximal small bowel adapts to reabsorb more fluid, the average amount of ileostomy drainage is about 500 mL daily. One cup is about 240 mL. DIF: Cognitive Level: Understand (comprehension) REF: 958 TOP: Nursing Process: Implementation

7. A malnourished patient is receiving a parenteral nutrition (PN) infusion containing amino acids and dextrose from a bag that was hung with a new tubing and filter 24 hours ago. The nurse observes that about 50 mL remain in the PN container. Which action is best for the nurse to take? a. Add a new container of PN using the current tubing and filter. b. Hang a new container of PN and change the IV tubing and filter. c. Infuse the remaining 50 mL and then hang a new container of PN. d. Ask the health care provider to clarify the written PN prescription.

A All PN solutions are changed at 24 hours. PN solutions containing dextrose and amino acids require a change in tubing and filter every 72 hours rather than daily. Infusion of the additional 50 mL will increase patient risk for infection. Changing the IV tubing and filter more frequently than required will unnecessarily increase costs. The nurse (not the health care provider) is responsible for knowing the indicated times for tubing and filter changes. DIF: Cognitive Level: Apply (application) REF: 870 TOP: Nursing Process: Implementation

13. A nurse cares for a client admitted from a nursing home after several recent falls. What prescription should the nurse complete first? a. Obtain urine sample for culture and sensitivity. b. Administer intravenous antibiotics. c. Encourage protein intake and additional fluids. d. Consult physical therapy for gait training.

A Although all interventions are or might be important, obtaining a urine sample for urinalysis takes priority. Often urinary tract infection (UTI) symptoms in older adults are atypical, and a UTI may present with new onset of confusion or falling. The urine sample should be obtained before starting antibiotics. Dietary requirements and gait training should be implemented after obtaining the urine sample. DIF: Applying/Application REF: 1387 KEY: Cystitis| assessment/diagnostic examination| older adult

31. Which nursing action is of highest priority for a patient with renal calculi who is being admitted to the hospital with gross hematuria and severe colicky left flank pain? a. Administer prescribed analgesics. b. Monitor temperature every 4 hours. c. Encourage increased oral fluid intake. d. Give antiemetics as needed for nausea.

A Although all of the nursing actions may be used for patients with renal lithiasis, the patient's presentation indicates that management of pain is the highest priority action. If the patient has urinary obstruction, increasing oral fluids may increase the symptoms. There is no evidence of infection or nausea. DIF: Cognitive Level: Analyze (analysis) REF: 1049 OBJ: Special Questions: Prioritization TOP: Nursing Process: Implementation

5. A patient complains of gas pains and abdominal distention 2 days after a small bowel resection. Which nursing action should the nurse take? a. Encourage the patient to ambulate. b. Instill a mineral oil retention enema. c. Administer the prescribed IV morphine sulfate. d. Offer the prescribed promethazine (Phenergan).

A Ambulation will improve peristalsis and help the patient eliminate flatus and reduce gas pain. A mineral oil retention enema is helpful for constipation with hard stool. A return-flow enema might be used to relieve persistent gas pains. Morphine will further reduce peristalsis. Promethazine is used as an antiemetic rather than to decrease gas pains or distention. DIF: Cognitive Level: Analyze (analysis) REF: 940 TOP: Nursing Process: Implementation

9. A patient hospitalized with an acute exacerbation of ulcerative colitis is having 14 to 16 bloody stools a day and crampy abdominal pain associated with the diarrhea. The nurse will plan to a. place the patient on NPO status. b. administer IV metoclopramide (Reglan). c. teach the patient about total colectomy surgery. d. administer cobalamin (vitamin B12) injections.

A An initial therapy for an acute exacerbation of inflammatory bowel disease (IBD) is to rest the bowel by making the patient NPO. Cobalamin (vitamin B12) is absorbed in the ileum, which is not affected by ulcerative colitis. Although total colectomy is needed for some patients, there is no indication that this patient is a candidate. Metoclopramide increases peristalsis and will worsen symptoms. DIF: Cognitive Level: Application REF: 1022-1025 TOP: Nursing Process: Planning

18. Following rectal surgery, a patient voids about 50 mL of urine every 30 to 60 minutes. Which nursing action is most appropriate? a. Use an ultrasound scanner to check the postvoiding residual. b. Monitor the patient's intake and output over the next few hours. c. Have the patient take small amounts of fluid frequently throughout the day. d. Reassure the patient that this is normal after rectal surgery because of anesthesia.

A An ultrasound scanner can be used to check for residual urine after the patient voids. Because the patient's history and clinical manifestations are consistent with overflow incontinence, it is not appropriate to have the patient drink small amounts. Although overflow incontinence is not unusual after surgery, the nurse should intervene to correct the physiologic problem, not just reassure the patient. The patient may develop reflux into the renal pelvis as well as discomfort from a full bladder if the nurse waits to address the problem for several hours. DIF: Cognitive Level: Application REF: 1146-1147 | 1154 TOP: Nursing Process: Implementation

28. After the nurse has completed teaching a patient with newly diagnosed celiac disease, which breakfast choice by the patient indicates good understanding of the information? a. Corn tortilla with eggs b. Bagel with cream cheese c. Oatmeal with non-fat milk d. Whole wheat toast with butter

A Avoidance of gluten-containing foods is the only treatment for celiac disease. Corn does not contain gluten, while oatmeal and wheat do. DIF: Cognitive Level: Application REF: 1050 | 1051-1052 TOP: Nursing Process: Evaluation

13. A 32-year-old patient is diagnosed with polycystic kidney disease. Which information is most appropriate for the nurse to include in teaching at this time? a. Importance of genetic counseling b. Complications of renal transplantation c. Methods for treating chronic and severe pain d. Differences between hemodialysis and peritoneal dialysis

A Because a 32-year-old patient may be considering having children, the nurse should include information about genetic counseling when teaching the patient. The well-managed patient will not need to choose between hemodialysis and peritoneal dialysis or know about the effects of transplantation for many years. There is no indication that the patient has chronic pain. DIF: Cognitive Level: Application REF: 1143 TOP: Nursing Process: Implementation

8. Which nursing action will be included in the plan of care for a patient with bowel irregularity and a new diagnosis of irritable bowel syndrome (IBS)? a. Encourage the patient to express feelings and ask questions about IBS. b. Suggest that the patient increase the intake of milk and other dairy products. c. Educate the patient about the use of alosetron (Lotronex) to reduce symptoms. d. Teach the patient to avoid using nonsteroidal anti-inflammatory drugs (NSAIDs).

A Because psychologic and emotional factors can affect the symptoms for IBS, encouraging the patient to discuss emotions and ask questions is an important intervention. Alosetron has serious side effects and is used only for female patients who have not responded to other therapies. Although yogurt may be beneficial, milk is avoided because lactose intolerance can contribute to symptoms in some patients. NSAIDs can be used by patients with IBS. DIF: Cognitive Level: Application REF: 1017-1018 TOP: Nursing Process: Implementation

8. Which nursing action will be included in the plan of care for a 25-yr-old male patient with a new diagnosis of irritable bowel syndrome (IBS)? a. Encourage the patient to express concerns and ask questions about IBS. b. Suggest that the patient increase the intake of milk and other dairy products. c. Teach the patient to avoid using nonsteroidal antiinflammatory drugs (NSAIDs). d. Teach the patient about the use of alosetron (Lotronex) to reduce IBS symptoms.

A Because psychologic and emotional factors can affect the symptoms for IBS, encouraging the patient to discuss emotions and ask questions is an important intervention. Alosetron has serious side effects and is used only for female patients who have not responded to other therapies. Although yogurt may be beneficial, milk is avoided because lactose intolerance can contribute to symptoms in some patients. NSAIDs can be used by patients with IBS. DIF: Cognitive Level: Apply (application) REF: 940 TOP: Nursing Process: Planning

43. A patient is admitted to the emergency department with possible renal trauma after an automobile accident. Which prescribed intervention will the nurse implement first? a. Check blood pressure and heart rate. b. Administer morphine sulfate 4 mg IV. c. Transport to radiology for an intravenous pyelogram. d. Insert a urethral catheter and obtain a urine specimen.

A Because the kidney is very vascular, the initial action with renal trauma will be assessment for bleeding and shock. The other actions are also important after the patient's cardiovascular status has been determined and stabilized. DIF: Cognitive Level: Analyze (analysis) REF: 1050 OBJ: Special Questions: Prioritization TOP: Nursing Process: Planning

36. The health care provider prescribes the following therapies for a patient who has been admitted with dehydration and hypotension after 3 days of nausea and vomiting. Which order will the nurse implement first? a. Infuse normal saline at 250 mL/hr. b. Administer IV ondansetron (Zofran). c. Provide oral care with moistened swabs. d. Insert a 16-gauge nasogastric (NG) tube.

A Because the patient has severe dehydration, rehydration with IV fluids is the priority. The other orders should be accomplished as quickly as possible after the IV fluids are initiated. DIF: Cognitive Level: Application REF: 964-965 OBJ: Special Questions: Prioritization TOP: Nursing Process: Implementation

32. The nurse is caring for a patient who has had an ileal conduit for several years. Which nursing action could be delegated to unlicensed assistive personnel (UAP)? a. Change the ostomy appliance. b. Choose the appropriate ostomy bag. c. Monitor the appearance of the stoma. d. Assess for possible urinary tract infection (UTI).

A Changing the ostomy appliance for a stable patient could be done by UAP. Assessments of the site, choosing the appropriate ostomy bag, and assessing for UTI symptoms require more education and scope of practice and should be done by the registered nurse (RN). DIF: Cognitive Level: Apply (application) REF: 1059 OBJ: Special Questions: Delegation TOP: Nursing Process: Planning

8. A nurse and a registered dietitian are assessing clients for partial parenteral nutrition (PPN). For which client would the nurse suggest another route of providing nutrition? a. Client with congestive heart failure b.. Older client with dementia c. Client who has multiorgan failure d. Client who is post gastric resection

A Clients receiving PPN typically get large amounts of fluid volume, making the client with heart failure a poor candidate. The other candidates are appropriate for this type of nutritional support. DIF: Analyzing/Analysis REF: 1244 KEY: Nutritional disorders| heart failure| parenteral nutrition| nursing assessment

1. When assessing a patient who is a vegan, which finding may indicate the need for cobalamin supplementation? a. Paresthesias b. Ecchymoses c. Dry, scaly skin d. Gingival swelling

A Cobalamin (vitamin B12) cannot be obtained from foods of plant origin, so the patient will be most at risk for signs of cobalamin deficiency, such as anemia and peripheral neuropathy. The other symptoms listed are associated with other nutritional deficiencies but would not be associated with a vegan diet. DIF: Cognitive Level: Application REF: 922-923 TOP: Nursing Process: Assessment

1. Which finding for a young adult who follows a vegan diet may indicate the need for cobalamin supplementation? a. Paresthesias c. Dry, scaly skin b. Ecchymoses d. Gingival swelling

A Cobalamin (vitamin B12) cannot be obtained from foods of plant origin, so the patient will be most at risk for signs of cobalamin deficiency, such as paresthesias, peripheral neuropathy, and anemia. The other symptoms listed are associated with other nutritional deficiencies but would not be associated with a vegan diet. DIF: Cognitive Level: Apply (application) REF: 856 TOP: Nursing Process: Assessment

16. Cobalamin injections have been prescribed for a patient with chronic atrophic gastritis. The nurse determines that teaching regarding the injections has been effective when the patient states, a. "The cobalamin injections will prevent me from becoming anemic." b. "These injections will increase the hydrochloric acid in my stomach." c. "These injections will decrease my risk for developing stomach cancer." d. "The cobalamin injections need to be taken until my inflamed stomach heals."

A Cobalamin supplementation prevents the development of pernicious anemia. The incidence of stomach cancer is higher in patients with chronic gastritis, but cobalamin does not reduce the risk for stomach cancer. Chronic gastritis may cause achlorhydria, but cobalamin does not correct this. The loss of intrinsic factor secretion with chronic gastritis is permanent, and the patient will need lifelong supplementation with cobalamin. DIF: Cognitive Level: Application REF: 986-987 TOP: Nursing Process: Evaluation

15. A nurse assesses a client who is prescribed 5-fluorouracil (5-FU) chemotherapy intravenously for the treatment of colon cancer. Which assessment finding should alert the nurse to contact the health care provider? a. White blood cell (WBC) count of 1500/mm3 b. Fatigue c. Nausea and diarrhea d. Mucositis and oral ulcers

A Common side effects of 5-FU include fatigue, leukopenia, diarrhea, mucositis and mouth ulcers, and peripheral neuropathy. However, the client's WBC count is very low (normal range is 5000 to 10,000/mm3), so the provider should be notified. He or she may want to delay chemotherapy by a day or two. Certainly the client is at high risk for infection. The other assessment findings are consistent with common side effects of 5-FU that would not need to be reported immediately. DIF: Applying/Application REF: 1151 KEY: Colorectal cancer| medications| adverse effects

31. The nurse is assessing a patient with abdominal pain. The nurse, who notes that there is ecchymosis around the area of umbilicus, will document this finding as a. Cullen sign. c. McBurney sign. b. Rovsing sign. d. Grey-Turner's sign.

A Cullen sign is ecchymosis around the umbilicus. Rovsing sign occurs when palpation of the left lower quadrant causes pain in the right lower quadrant. Grey Turner's sign is bruising over the flanks. Deep tenderness at McBurney's point (halfway between the umbilicus and the right iliac crest), known as McBurney's sign, is a sign of acute appendicitis. DIF: Cognitive Level: Understand (comprehension) REF: 941 TOP: Nursing Process: Assessment

10. After teaching a client with a history of renal calculi, the nurse assesses the client's understanding. Which statement made by the client indicates a correct understanding of the teaching? a. "I should drink at least 3 liters of fluid every day." b. "I will eliminate all dairy or sources of calcium from my diet." c. "Aspirin and aspirin-containing products can lead to stones." d. "The doctor can give me antibiotics at the first sign of a stone."

A Dehydration contributes to the precipitation of minerals to form a stone. Although increased intake of calcium causes hypercalcemia and leads to excessive calcium filtered into the urine, if the client is well hydrated the calcium will be excreted without issues. Dehydration increases the risk for supersaturation of calcium in the urine, which contributes to stone formation. The nurse should encourage the client to drink more fluids, not decrease calcium intake. Ingestion of aspirin or aspirin-containing products does not cause a stone. Antibiotics neither prevent nor treat a stone. DIF: Applying/Application REF: 1384 KEY: Urolithiasis| hydration

13. A patient has just arrived on the postoperative unit after having a laparoscopic esophagectomy for treatment of esophageal cancer. Which nursing action should be included in the postoperative plan of care? a. Elevate the head of the bed to at least 30 degrees. b. Reposition the nasogastric (NG) tube if drainage stops or decreases. c. Notify the doctor immediately about bloody NG drainage. d. Start oral fluids when the patient has active bowel sounds.

A Elevation of the head of the bed decreases the risk for reflux and aspiration of gastric secretions. The NG tube should not be repositioned without consulting with the health care provider. Bloody NG drainage is expected for the first 8 to 12 hours. A swallowing study is needed before oral fluids are started. DIF: Cognitive Level: Application REF: 979 TOP: Nursing Process: Planning

19. The nurse will anticipate preparing an older patient who is vomiting "coffee-ground" emesis for a. endoscopy. c. barium studies. b. angiography. d. gastric analysis.

A Endoscopy is the primary tool for visualization and diagnosis of upper gastrointestinal (GI) bleeding. Angiography is used only when endoscopy cannot be done because it is more invasive and has more possible complications. Barium studies are helpful in determining the presence of gastric lesions, but not whether the lesions are actively bleeding. Gastric analysis testing may help with determining the cause of gastric irritation, but it is not used for acute GI bleeding. DIF: Cognitive Level: Apply (application) REF: 922 TOP: Nursing Process: Planning

19. A patient is hospitalized with vomiting of "coffee-ground" emesis. The nurse will anticipate preparing the patient for a. endoscopy. b. angiography. c. gastric analysis testing. d. barium contrast studies.

A Endoscopy is the primary tool for visualization and diagnosis of upper gastrointestinal (GI) bleeding. Angiography is used only when endoscopy cannot be done because it is more invasive and has more possible complications. Gastric analysis testing may help with determining the cause of gastric irritation, but it is not used for acute GI bleeding. Barium studies are helpful in determining the presence of gastric lesions, but not whether the lesions are actively bleeding. DIF: Cognitive Level: Application REF: 982 TOP: Nursing Process: Planning

4. The nurse is coaching a community group for individuals who are overweight. Which participant behavior is an example of the best exercise plan for weight loss? a. Walking for 40 minutes 6 or 7 days/week b. Lifting weights with friends 3 times/week c. Playing soccer for an hour on the weekend d. Running for 10 to 15 minutes 3 times/week

A Exercise should be done daily for 30 minutes to an hour. Exercising in highly aerobic activities for short bursts or only once a week is not helpful and may be dangerous in an individual who has not been exercising. Running may be appropriate, but a patient should start with an exercise that is less stressful and can be done for a longer period. Weight lifting is not as helpful as aerobic exercise in weight loss. DIF: Cognitive Level: Analyze (analysis) REF: 883 TOP: Nursing Process: Evaluation

4. Which patient behavior indicates that an overweight patient has understood the nurse's teaching about the best exercise plan for weight loss? a. Walking for 40 minutes 6 or 7 days/week b. Lifting weights with friends 3 times/week c. Playing soccer for an hour on the weekend d. Running for 10 to 15 minutes 3 times/week

A Exercise should be done daily for 30 minutes to an hour. Exercising in highly aerobic activities for short bursts or only once a week is not helpful and may be dangerous in an individual who has not been exercising. Running may be appropriate, but a patient should start with an exercise that is less stressful and can be done for a longer period. Weight lifting is not as helpful as aerobic exercise in weight loss. DIF: Cognitive Level: Application REF: 952-953 TOP: Nursing Process: Evaluation

6. A nurse cares for adult clients who experience urge incontinence. For which client should the nurse plan a habit training program? a. A 78-year-old female who is confused b. A 65-year-old male with diabetes mellitus c. A 52-year-old female with kidney failure d. A 47-year-old male with arthritis

A For a bladder training program to succeed in a client with urge incontinence, the client must be alert, aware, and able to resist the urge to urinate. Habit training will work best for a confused client. This includes going to the bathroom (or being assisted to the bathroom) at set times. The other clients may benefit from another type of bladder training. DIF: Applying/Application REF: 1380 KEY: Urinary incontinence| health screening

12. When planning teaching for a patient with benign nephrosclerosis the nurse should include instructions regarding a. monitoring and recording blood pressure. b. obtaining and documenting daily weights. c. measuring daily intake and output amounts. d. preventing bleeding caused by anticoagulants.

A Hypertension is the major symptom of nephrosclerosis. Measurements of intake and output and daily weights are not necessary unless the patient develops renal insufficiency. Anticoagulants are not used to treat nephrosclerosis. DIF: Cognitive Level: Application REF: 1141-1142 TOP: Nursing Process: Planning

25. Twelve hours after undergoing a gastroduodenostomy (Billroth I), a patient complains of increasing abdominal pain. The patient has absent bowel sounds and 200 mL of bright red nasogastric (NG) drainage in the last hour. The most appropriate action by the nurse at this time is to a. notify the surgeon. b. irrigate the NG tube. c. administer the prescribed morphine. d. continue to monitor the NG drainage.

A Increased pain and 200 mL of bright red NG drainage 12 hours after surgery indicate possible postoperative hemorrhage, and immediate actions such as blood transfusion and/or return to surgery are needed. Because the NG is draining, there is no indication that irrigation is needed. The patient may need morphine, but this is not the highest priority action. Continuing to monitor the NG drainage is not an adequate response. DIF: Cognitive Level: Application REF: 996 | 997-998 TOP: Nursing Process: Implementation

25. A patient who underwent a gastroduodenostomy (Billroth I) 12 hours ago complains of increasing abdominal pain. The patient has no bowel sounds and 200 mL of bright red nasogastric (NG) drainage in the past hour. The highest priority action by the nurse is to a. contact the surgeon. b. irrigate the NG tube. c. monitor the NG drainage. d. administer the prescribed morphine.

A Increased pain and 200 mL of bright red NG drainage 12 hours after surgery indicate possible postoperative hemorrhage, and immediate actions such as blood transfusion or return to surgery are needed (or both). Because the NG is draining, there is no indication that irrigation is needed. Continuing to monitor the NG drainage is not an adequate response. The patient may need morphine, but this is not the highest priority action. DIF: Cognitive Level: Analyze (analysis) REF: 917 OBJ: Special Questions: Prioritization TOP: Nursing Process: Implementation

9. Which information will the nurse plan to include in discharge teaching for a patient after gastric bypass surgery? a. Avoid drinking fluids with meals. b. Choose high-fat foods for at least 30% of intake. c. Choose foods that are high in fiber to promote bowel function. d. Development of flabby skin can be prevented by daily exercise.

A Intake of fluids with meals tends to cause dumping syndrome and diarrhea. Food choices should be low in fat and fiber. Exercise does not prevent the development of flabby skin. DIF: Cognitive Level: Application REF: 959 TOP: Nursing Process: Planning

7. The nurse will be teaching self-management to patients after gastric bypass surgery. Which information will the nurse plan to include? a. Drink fluids between meals but not with meals. b. Choose high-fat foods for at least 30% of intake. c. Developing flabby skin can be prevented by exercise. d. Choose foods high in fiber to promote bowel function.

A Intake of fluids with meals tends to cause dumping syndrome and diarrhea. Food choices should be low in fat and fiber. Exercise does not prevent the development of flabby skin. DIF: Cognitive Level: Apply (application) REF: 889 TOP: Nursing Process: Planning

2. A nurse assesses a client who is prescribed alosetron (Lotronex). Which assessment question should the nurse ask this client? a. "Have you been experiencing any constipation?" b. "Are you eating a diet high in fiber and fluids?" c. "Do you have a history of high blood pressure?" d. "What vitamins and supplements are you taking?"

A Ischemic colitis is a life-threatening complication of alosetron. The nurse should assess the client for constipation. The other questions do not identify complications related to alosetron. DIF: Applying/Application REF: 1146 KEY: Medications| adverse effects

14. Which of these nursing actions included in the plan of care for a patient who is receiving intermittent tube feedings through a percutaneous endoscopic gastrostomy (PEG) tube may be delegated to an LPN/LVN? a. Providing skin care to the area around the tube site b. Assessing the patient's nutritional status at least weekly c. Determining the need for the addition of water to the feedings d. Teaching the patient and family how to administer tube feedings

A LPN/LVN education and scope of practice include actions such as dressing changes and wound care. Patient teaching and complex assessments (such as patient nutrition and hydration status) require RN-level education and scope of practice. DIF: Cognitive Level: Application REF: 935 OBJ: Special Questions: Delegation TOP: Nursing Process: Planning

5. After teaching a client who has stress incontinence, the nurse assesses the client's understanding. Which statement made by the client indicates a need for additional teaching? a. "I will limit my total intake of fluids." b. "I must avoid drinking alcoholic beverages." c. "I must avoid drinking caffeinated beverages." d. "I shall try to lose about 10% of my body weight."

A Limiting fluids concentrates urine and can irritate tissues, leading to increased incontinence. Many people try to manage incontinence by limiting fluids. Alcoholic and caffeinated beverages are bladder stimulants. Obesity increases intra-abdominal pressure, causing incontinence. DIF: Applying/Application REF: 1380 KEY: Cystitis| hydration

5. A patient who has just been started on continuous tube feedings of a full-strength commercial formula at 100 mL/hr using a closed system method has six diarrhea stools the first day. Which action should the nurse plan to take? a. Slow the infusion rate of the tube feeding. b. Check gastric residual volumes more frequently. c. Change the enteral feeding system and formula every 8 hours. d. Discontinue administration of water through the feeding tube.

A Loose stools indicate poor absorption of nutrients and indicate a need to slow the feeding rate or decrease the concentration of the feeding. Water should be given when patients receive enteral feedings to prevent dehydration. When a closed enteral feeding system is used, the tubing and formula are changed every 24 hours. High residual volumes do not contribute to diarrhea. DIF: Cognitive Level: Application REF: 933-934 TOP: Nursing Process: Planning

5. A patient who has just been started on tube feedings of full-strength formula at 100 mL/hr has 6 diarrhea stools the first day. Which action should the nurse plan to take? a. Slow the infusion rate of the tube feeding. b. Check gastric residual volumes more frequently. c. Change the enteral feeding system and formula every 8 hours. d. Discontinue administration of water through the feeding tube.

A Loose stools indicate poor absorption of nutrients and indicate a need to slow the feeding rate or decrease the concentration of the feeding. Water should be given when patients receive enteral feedings to prevent dehydration. When a closed enteral feeding system is used, the tubing and formula are changed every 24 hours. High residual volumes do not contribute to diarrhea. DIF: Cognitive Level: Apply (application) REF: 865 TOP: Nursing Process: Planning

15. When caring for a patient who has a new diagnosis of Crohn's disease after having frequent diarrhea and a weight loss of 10 pounds (4.5 kg) over 2 months, the nurse will plan to teach the patient about a. medication use. b. fluid restriction. c. enteral nutrition. d. activity restrictions.

A Medications are used to induce and maintain remission in patients with inflammatory bowel disease (IBD). Decreased activity level is indicated only if the patient has severe fatigue and weakness. Fluids are needed to prevent dehydration. There is no advantage to enteral feedings. DIF: Cognitive Level: Application REF: 1025-1027 TOP: Nursing Process: Planning

15. A patient has a new diagnosis of Crohn's disease after having frequent diarrhea and a weight loss of 10 lb (4.5 kg) over 2 months. The nurse will plan to teach about a. medication use. c. enteral nutrition. b. fluid restriction. d. activity restrictions.

A Medications are used to induce and maintain remission in patients with inflammatory bowel disease (IBD). Decreased activity level is indicated only if the patient has severe fatigue and weakness. Fluids are needed to prevent dehydration. There is no advantage to enteral feedings. DIF: Cognitive Level: Apply (application) REF: 947 TOP: Nursing Process: Planning

19. A patient in the hospital has a history of functional urinary incontinence. Which nursing action will be included in the plan of care? a. Place a bedside commode near the patient's bed. b. Demonstrate the use of the Credé maneuver to the patient. c. Use an ultrasound scanner to check postvoiding residuals. d. Teach the use of Kegel exercises to strengthen the pelvic floor.

A Modifications in the environment make it easier to avoid functional incontinence. Checking for residual urine and performing the Credé maneuver are interventions for overflow incontinence. Kegel exercises are useful for stress incontinence. DIF: Cognitive Level: Application REF: 1148 TOP: Nursing Process: Planning

40. A patient in the urology clinic is diagnosed with monilial urethritis. Which action will the nurse include in the plan of care? a. Teach the patient about the use of antifungal medications. b. Tell the patient to avoid tub baths until the symptoms resolve. c. Instruct the patient to refer recent sexual partners for treatment. d. Teach the patient to avoid nonsteroidal antiinflammatory drugs (NSAIDs).

A Monilial urethritis is caused by a fungus and antifungal medications such as nystatin or fluconazole are usually used as treatment. Because monilial urethritis is not sexually transmitted, there is no need to refer sexual partners. Warm baths and NSAIDS may be used to treat symptoms. DIF: Cognitive Level: Apply (application) REF: 1039 TOP: Nursing Process: Planning

3. A client is scheduled to have a fundoplication. What statement by the client indicates a need to review preoperative teaching? a. "After the operation I can eat anything I want." b. "I will have to eat smaller, more frequent meals." c. "I will take stool softeners for several weeks." d. "This surgery may not totally control my symptoms."

A Nutritional and lifestyle changes need to continue after surgery as the procedure does not offer a lifetime cure. The other statements show good understanding. DIF: Evaluating/Synthesis REF: 1117 KEY: Gastrointestinal disorders| patient education

18. A patient who has had several episodes of bloody diarrhea is admitted to the emergency department. Which action should the nurse anticipate taking? a. Obtain a stool specimen for culture. b. Administer antidiarrheal medications. c. Teach about adverse effects of nonsteroidal anti-inflammatory drugs (NSAIDs). d. Provide education about antibiotic therapy.

A Patients with bloody diarrhea should have a stool culture for E. coli O157:H7. NSAIDs may cause occult blood in the stools, but not diarrhea. Antidiarrheal medications usually are avoided for possible infectious diarrhea to avoid prolonging the infection. Antibiotic therapy in the treatment of infectious diarrhea is controversial because it may precipitate kidney complications. DIF: Cognitive Level: Application REF: 1001-1002 TOP: Nursing Process: Planning

18. Which action should the nurse in the emergency department anticipate for a young adult patient who has had several episodes of bloody diarrhea? a. Obtain a stool specimen for culture. b. Administer antidiarrheal medication. c. Provide teaching about antibiotic therapy. d. Teach the adverse effects of acetaminophen (Tylenol).

A Patients with bloody diarrhea should have a stool culture for Escherichia coli O157:H7. Antidiarrheal medications are usually avoided for possible infectious diarrhea to avoid prolonging the infection. Antibiotic therapy in the treatment of infectious diarrhea is controversial because it may precipitate kidney complications. Acetaminophen does not cause bloody diarrhea. DIF: Cognitive Level: Apply (application) REF: 925 TOP: Nursing Process: Planning

54. Which prescribed intervention for a patient with chronic short bowel syndrome will the nurse question? a. Senna 1 tablet every day b. Ferrous sulfate 325 mg daily c. Psyllium (Metamucil) 3 times daily d. Diphenoxylate with atropine (Lomotil) prn loose stools

A Patients with short bowel syndrome have diarrhea because of decreased nutrient and fluid absorption and would not need stimulant laxatives. Iron supplements are used to prevent iron-deficiency anemia, bulk-forming laxatives help make stools less watery, and opioid antidiarrheal drugs are helpful in slowing intestinal transit time. DIF: Cognitive Level: Apply (application) REF: 936 TOP: Nursing Process: Implementation

6. A nurse is caring for a client receiving enteral feedings through a Dobhoff tube. What action by the nurse is best to prevent hyperosmolarity? a. Administer free-water boluses. b. Change the client's formula. c. Dilute the client's formula. d. Slow the rate of infusion.

A Proteins and sugar molecules in the enteral feeding product contribute to dehydration due to increased osmolarity. The nurse can administer free-water boluses after consulting with the provider on the appropriate amount and timing of the boluses, or per protocol. The client may not be able to switch formulas. Diluting the formula is not appropriate. Slowing the rate of the infusion will not address the problem. DIF: Analyzing/Analysis REF: 1242 KEY: Nutritional disorders| tube feedings

21. The health care provider orders intravenous (IV) ranitidine (Zantac) for a patient with gastrointestinal (GI) bleeding caused by peptic ulcer disease. When teaching the patient about the effect of the medication, which information will the nurse include? a. "Ranitidine decreases secretion of gastric acid." b. "Ranitidine neutralizes the acid in the stomach." c. "Ranitidine constricts the blood vessels in the stomach and decreases bleeding." d. "Ranitidine covers the ulcer with a protective material that promotes healing."

A Ranitidine is a histamine-2 (H2) receptor blocker, which decreases the secretion of gastric acid. The response beginning, "Ranitidine constricts the blood vessels" describes the effect of vasopressin. The response beginning "Ranitidine neutralizes the acid" describes the effect of antacids. And the response beginning "Ranitidine covers the ulcer" describes the action of sucralfate (Carafate). DIF: Cognitive Level: Comprehension REF: 965 | 983 TOP: Nursing Process: Implementation

26. Which nursing action will be most helpful in decreasing the risk for hospital-acquired infection (HAI) of the urinary tract in patients admitted to the hospital? a. Avoid unnecessary catheterizations. b. Encourage adequate oral fluid intake. c. Test urine with a dipstick daily for nitrites. d. Provide thorough perineal hygiene to patients.

A Since catheterization bypasses many of the protective mechanisms that prevent urinary tract infection (UTI), avoidance of catheterization is the most effective means of reducing HAI. The other actions will also be helpful, but are not as useful as decreasing urinary catheter use. DIF: Cognitive Level: Application REF: 1125-1127 TOP: Nursing Process: Planning

44. A patient with Crohn's disease who is taking infliximab (Remicade) calls the nurse in the outpatient clinic about new symptoms. Which symptom is most important to communicate to the health care provider? a. Fever c. Joint pain b. Nausea d. Headache

A Since infliximab suppresses the immune response, rapid treatment of infection is essential. The other patient complaints are common side effects of the medication, but they do not indicate any potentially life-threatening complications. DIF: Cognitive Level: Analyze (analysis) REF: 970 OBJ: Special Questions: Prioritization TOP: Nursing Process: Evaluation

34. A patient is unable to void after having an open loop resection and fulguration of the bladder. Which nursing action should be implemented? a. Assist the patient to soak in a 15-minute sitz bath. b. Restrict oral fluids to equal previous urine volume. c. Insert a straight urethral catheter and drain the bladder. d. Teach the patient how to do isometric perineal exercises.

A Sitz baths will relax the perineal muscles and promote voiding. The patient should be to drink fluids. Kegel exercises are helpful in the prevention of incontinence, but would not be helpful for a patient experiencing retention. Catheter insertion increases the risk for urinary tract infection and should be avoided when possible DIF: Cognitive Level: Apply (application) REF: 1039 TOP: Nursing Process: Implementation

32. When counseling a patient with a family history of stomach cancer about ways to decrease risk for developing stomach cancer, the nurse will teach the patient to avoid a. smoked foods such as bacon and ham. b. foods that cause abdominal distention. c. chronic use of H2 blocking medications. d. emotionally or physically stressful situations.

A Smoked foods such as bacon, ham, and smoked sausage increase the risk for stomach cancer. Use of H2 blockers, stressful situations, and abdominal distention are not associated with an increased incidence of stomach cancer. DIF: Cognitive Level: Comprehension REF: 998-999 TOP: Nursing Process: Implementation

16. A nurse obtains the health history of a client with a suspected diagnosis of bladder cancer. Which question should the nurse ask when determining this client's risk factors? a. "Do you smoke cigarettes?" b. "Do you use any alcohol?" c. "Do you use recreational drugs?" d. "Do you take any prescription drugs?"

A Smoking is known to be a factor that greatly increases the risk of bladder cancer. Alcohol use, recreational drug use, and prescription drug use (except medications that contain phenacetin) are not known to increase the risk of developing bladder cancer. DIF: Applying/Application REF: 1388 KEY: Urothelial cancer| health screening

11. After the nurse has finished teaching a patient with ulcerative colitis about sulfasalazine (Azulfidine), which patient statement indicates that the teaching has been effective? a. "I will need to use a sunscreen when I am outdoors." b. "I will need to avoid contact with people who are sick." c. "The medication will need to be tapered if I need surgery." d. "The medication will prevent infections that cause the diarrhea."

A Sulfasalazine may cause photosensitivity in some patients. It is not used to treat infections. Sulfasalazine does not reduce immune function. Unlike corticosteroids, tapering of sulfasalazine is not needed. DIF: Cognitive Level: Application REF: 1026-1027 TOP: Nursing Process: Evaluation

30. Which information will be best for the nurse to include when teaching a patient with peptic ulcer disease (PUD) about dietary management of the disease? a. "Avoid foods that cause pain after you eat them." b. "High-protein foods are least likely to cause pain." c. "You will need to remain on a bland diet indefinitely." d. "You should avoid eating many raw fruits and vegetables."

A The best information is that each individual should choose foods that are not associated with postprandial discomfort. Raw fruits and vegetables may irritate the gastric mucosa, but chewing well seems to decrease this and some patients may tolerate these well. High-protein foods help to neutralize acid, but they also stimulate hydrochloric (HCl) acid secretion and may increase discomfort for some patients. Bland diets may be recommended during an acute exacerbation of PUD, but there is little scientific evidence to support their use. DIF: Cognitive Level: Application REF: 992 TOP: Nursing Process: Implementation

3. After teaching a client who has a femoral hernia, the nurse assesses the client's understanding. Which statement indicates the client needs additional teaching related to the proper use of a truss? a. "I will put on the truss before I go to bed each night." b. "I'll put some powder under the truss to avoid skin irritation." c. "The truss will help my hernia because I can't have surgery." d. "If I have abdominal pain, I'll let my health care provider know right away."

A The client should be instructed to apply the truss before arising, not before going to bed at night. The other statements show an accurate understanding of using a truss. DIF: Applying/Application REF: 1147 KEY: Herniation

18. A nurse teaches a young female client who is prescribed amoxicillin (Amoxil) for a urinary tract infection. Which statement should the nurse include in this client's teaching? a. "Use a second form of birth control while on this medication." b. "You will experience increased menstrual bleeding while on this drug." c. "You may experience an irregular heartbeat while on this drug." d. "Watch for blood in your urine while taking this medication."

A The client should use a second form of birth control because penicillin seems to reduce the effectiveness of estrogen-containing contraceptives. She should not experience increased menstrual bleeding, an irregular heartbeat, or blood in her urine while taking the medication. DIF: Understanding/Comprehension REF: 1371 KEY: Cystitis| medication safety

15. A client has dumping syndrome after a partial gastrectomy. Which action by the nurse would be most helpful? a. Arrange a dietary consult. b. Increase fluid intake. c.. Limit the client's foods. d. Make the client NPO.

A The client with dumping syndrome after a gastrectomy has multiple dietary needs. A referral to the registered dietitian will be extremely helpful. Food and fluid intake is complicated and needs planning. The client should not be NPO. DIF: Applying/Application REF: 1140 KEY: Gastrointestinal disorders| referrals| nutrition

39. A patient is brought to the emergency department with a knife impaled in the abdomen following a domestic fight. During the initial assessment of the patient, the nurse should a. assess the BP and pulse. b. remove the knife to assess the wound. c. determine the presence of Rovsing sign. d. insert a urinary catheter and assess for hematuria.

A The initial assessment is focused on determining whether the patient has hypovolemic shock. The knife should not be removed until the patient is in surgery, where bleeding can be controlled. Rovsing sign is assessed in the patient with suspected appendicitis. A patient with a knife in place will be taken to surgery and assessed for bladder trauma there. DIF: Cognitive Level: Application REF: 1019-1020 OBJ: Special Questions: Prioritization TOP: Nursing Process: Assessment

36. Which action should the nurse take first when a patient calls the clinic complaining of diarrhea of 24 hours' duration? a. Ask the patient to describe the character of the stools and any associated symptoms. b. Inform the patient that laboratory testing of blood and stool specimens will be necessary. c. Suggest that the patient drink clear liquid fluids with electrolytes, such as Gatorade or Pedialyte. d. Advise the patient to use over-the-counter loperamide (Imodium) to slow gastrointestinal (GI) motility.

A The initial response by the nurse should be further assessment of the patient. The other responses may be appropriate, depending on what is learned in the assessment. DIF: Cognitive Level: Application REF: 1008-1009 OBJ: Special Questions: Prioritization TOP: Nursing Process: Assessment

24. After teaching a patient to irrigate a new colostomy, the nurse will determine that the teaching has been effective if the patient a. hangs the irrigating container about 18 inches above the stoma. b. stops the irrigation and removes the irrigating cone if cramping occurs. c. inserts the irrigation tubing no further than 4 to 6 inches into the stoma. d. fills the irrigating container with 1000 to 2000 mL of lukewarm tap water.

A The irrigating container should be hung 18 to 24 inches above the stoma. If cramping occurs, the irrigation should be temporarily stopped and the cone left in place. Five hundred to 1000 mL of water should be used for irrigation. An irrigation cone, rather than tubing, should be inserted into the stoma; 4 to 6 inches would be too far for safe insertion. DIF: Cognitive Level: Application REF: 1045 TOP: Nursing Process: Evaluation

38. A patient with a history of polycystic kidney disease is admitted to the surgical unit after having shoulder surgery. Which of the routine postoperative orders is most important for the nurse to discuss with the health care provider? a. Give ketorolac 10 mg PO PRN for pain. b. Infuse 5% dextrose in normal saline at 75 mL/hr. c. Order regular diet after patient is awake and alert. d. Draw blood urea nitrogen (BUN) and creatinine in 2 hours.

A The nonsteroidal antiinflammatory drugs (NSAIDs) should be avoided in patients with decreased renal function because nephrotoxicity is a potential adverse effect. The other orders do not need any clarification or change. DIF: Cognitive Level: Apply (application) REF: 1044 TOP: Nursing Process: Implementation

14. A client is in the bariatric clinic 1 month after having gastric bypass surgery. The client is crying and says "I didn't know it would be this hard to live like this." What response by the nurse is best? a. Assess the client's coping and support systems. b. Inform the client that things will get easier. c. Re-educate the client on needed dietary changes. d. Tell the client lifestyle changes are always hard.

A The nurse should assess this client's coping styles and support systems in order to provide holistic care. The other options do not address the client's distress. DIF: Applying/Application REF: 1252 KEY: Nutritional disorders| obesity| psychosocial response| coping

11. A nurse cares for a client who states, "My husband is repulsed by my colostomy and refuses to be intimate with me." How should the nurse respond? a. "Let's talk to the ostomy nurse to help you and your husband work through this." b. "You could try to wear longer lingerie that will better hide the ostomy appliance." c. "You should empty the pouch first so it will be less noticeable for your husband.." d. "If you are not careful, you can hurt the stoma if you engage in sexual activity."

A The nurse should collaborate with the ostomy nurse to help the client and her husband work through intimacy issues. The nurse should not minimize the client's concern about her husband with ways to hide the ostomy. The client will not hurt the stoma by engaging in sexual activity. DIF: Applying/Application REF: 1156 KEY: Ostomy care| support| coping

20. A nurse cares for a client who has a new colostomy. Which action should the nurse take? a. Empty the pouch frequently to remove excess gas collection. b. Change the ostomy pouch and wafer every morning. c. Allow the pouch to completely fill with stool prior to emptying it. d. Use surgical tape to secure the pouch and prevent leakage.

A The nurse should empty the new ostomy pouch frequently because of excess gas collection, and empty the pouch when it is one-third to one-half full of stool. The ostomy pouch does not need to be changed every morning. Ostomy wafers with paste should be used to secure and seal the ostomy appliance; surgical tape should not be used. DIF: Applying/Application REF: 1154 KEY: Ostomy care

15. A 40-yr-old obese woman reports that she wants to lose weight. Which question should the nurse ask first? a. "What factors led to your obesity?" b. "Which types of food do you like best?" c. "How long have you been overweight?" d. "What kind of activities do you enjoy?"

A The nurse should obtain information about the patient's perceptions of the reasons for the obesity to develop a plan individualized to the patient. The other information also will be obtained from the patient, but the patient is more likely to make changes when the patient's beliefs are considered in planning. DIF: Cognitive Level: Analyze (analysis) REF: 881 OBJ: Special Questions: Prioritization TOP: Nursing Process: Assessment

29. A patient who is diagnosed with nephrotic syndrome has 3+ ankle and leg edema and ascites. Which nursing diagnosis is a priority for the patient? a. Excess fluid volume related to low serum protein levels b. Activity intolerance related to increased weight and fatigue c. Disturbed body image related to peripheral edema and ascites d. Altered nutrition: less than required related to protein restriction

A The patient has massive edema, so the priority problem at this time is the excess fluid volume. The other nursing diagnoses also are appropriate, but the focus of nursing care should be resolution of the edema and ascites. DIF: Cognitive Level: Application REF: 1133-1135 OBJ: Special Questions: Prioritization TOP: Nursing Process: Diagnosis

44. After change-of-shift report, which patient should the nurse assess first? a. Patient with a urethral stricture who has not voided for 12 hours b. Patient who has cloudy urine after orthotopic bladder reconstruction c. Patient with polycystic kidney disease whose blood pressure is 186/98 mm Hg d. Patient who voided bright red urine immediately after returning from lithotripsy

A The patient information suggests acute urinary retention, which is a medical emergency. The nurse will need to assess the patient and consider whether to insert a retention catheter. The other patients will also be assessed, but their findings are consistent with their diagnoses and do not require immediate assessment or possible intervention. DIF: Cognitive Level: Analyze (analysis) REF: 1050 OBJ: Special Questions: Prioritization | Special Questions: Multiple Patients TOP: Nursing Process: Planning

27. A patient recovering from a gastrojejunostomy (Billroth II) for treatment of a duodenal ulcer develops dizziness, weakness, and palpitations about 20 minutes after eating. To avoid recurrence of these symptoms, the nurse teaches the patient to a. lie down for about 30 minutes after eating. b. choose foods that are high in carbohydrates. c. increase the amount of fluid intake with meals. d. drink sugared fluids or eat candy after each meal.

A The patient is experiencing symptoms of dumping syndrome, which may be reduced by lying down after eating. Increasing fluid intake and choosing high carbohydrate foods will increase the risk for dumping syndrome. Having a sweet drink or hard candy will correct the hypoglycemia that is associated with dumping syndrome but will not prevent dumping syndrome. DIF: Cognitive Level: Application REF: 996-998 TOP: Nursing Process: Implementation

30. A 76-yr-old with benign prostatic hyperplasia (BPH) is agitated and confused, with a markedly distended bladder. Which intervention prescribed by the health care provider should the nurse implement first? a. Insert a urinary retention catheter. b. Draw blood for a serum creatinine level. c. Schedule an intravenous pyelogram (IVP). d. Administer lorazepam (Ativan) 0.5 mg PO.

A The patient's history and clinical manifestations are consistent with acute urinary retention, and the priority action is to relieve the retention by catheterization. The BUN and creatinine measurements can be obtained after the catheter is inserted. The patient's agitation may resolve after the bladder distention is corrected, and sedative drugs should be used cautiously in older patients. The IVP may be used as a diagnostic test but does not need to be done urgently. DIF: Cognitive Level: Analyze (analysis) REF: 1060 OBJ: Special Questions: Prioritization TOP: Nursing Process: Implementation

30. An 88-year-old with benign prostatic hyperplasia (BPH) has a markedly distended bladder and is agitated and confused. Which of the following interventions prescribed by the health care provider should the nurse implement first? a. Insert a urinary retention catheter. b. Schedule an intravenous pyelogram. c. Administer lorazepam (Ativan) 0.5 mg PO. d. Draw blood for blood urea nitrogen (BUN) and creatinine testing.

A The patient's history and clinical manifestations are consistent with acute urinary retention, and the priority action is to relieve the retention by catheterization. The BUN and creatinine measurements can be obtained after the catheter is inserted. The patient's agitation may resolve once the bladder distention is corrected, and sedative drugs should be used cautiously in older patients. The IVP is an appropriate test, but does not need to be done urgently. DIF: Cognitive Level: Application REF: 1135-1136 OBJ: Special Questions: Prioritization TOP: Nursing Process: Implementation

8. A client is in the emergency department with an esophageal trauma. The nurse palpates subcutaneous emphysema in the mediastinal area and up into the lower part of the client's neck. What action by the nurse takes priority? a. Assess the client's oxygenation.. b. Facilitate a STAT chest x-ray. c. Prepare for immediate surgery.. d. Start two large-bore IVs.

A The priorities of care are airway, breathing, and circulation. The priority option is to assess oxygenation. This occurs before diagnostic or therapeutic procedures. The client needs two large-bore IVs as a trauma client, but oxygenation comes first. DIF: Applying/Application REF: 1123 KEY: Gastrointestinal disorders| trauma nursing| nursing assessment

23. A patient with a bleeding duodenal ulcer has a nasogastric (NG) tube in place, and the health care provider orders 30 mL of aluminum hydroxide/magnesium hydroxide (Maalox) to be instilled through the tube every hour. To evaluate the effectiveness of this treatment, the nurse a. periodically aspirates and tests gastric pH. b. monitors arterial blood gas values on a daily basis. c. checks each stool for the presence of occult blood. d. measures the amount of residual stomach contents hourly.

A The purpose for antacids is to increase gastric pH. Checking gastric pH is the most direct way of evaluating the effectiveness of the medication. Arterial blood gases may change slightly, but this does not directly reflect the effect of antacids on gastric pH. Because the patient has upper gastrointestinal (GI) bleeding, occult blood in the stools will appear even after the acute bleeding has stopped. The amount of residual stomach contents is not a reflection of resolution of bleeding or of gastric pH. DIF: Cognitive Level: Application REF: 973 TOP: Nursing Process: Evaluation

13. A healthy adult woman who weighs 145 lb (66 kg) asks the clinic nurse about the minimum daily requirement for protein. How many grams of protein will the nurse recommend? a. 53 c. 75 b. 66 d. 98

A The recommended daily protein intake is 0.8 to 1 g/kg of body weight. Therefore, the minimum for this patient is 66 kg × 0.8 g = 52.8 or 53 g/day. DIF: Cognitive Level: Apply (application) REF: 855 TOP: Nursing Process: Implementation

22. During the initial postoperative assessment of a patient's stoma formed from a transverse colostomy, the nurse finds it to be deep pink with moderate edema and a small amount of bleeding. The nurse should a. document the stoma assessment. b. monitor the stoma every 30 minutes. c. notify the surgeon about the stoma appearance. d. place an ice pack on the stoma to reduce swelling.

A The stoma appearance indicates good circulation to the stoma. There is no indication that surgical intervention is needed or that frequent stoma monitoring is required. Swelling of the stoma is normal for 2 to 3 weeks after surgery, and an ice pack is not needed. DIF: Cognitive Level: Application REF: 1041-1042 TOP: Nursing Process: Implementation

16. A nurse cares for a client who had a colostomy placed in the ascending colon 2 weeks ago. The client states, "The stool in my pouch is still liquid." How should the nurse respond? a. "The stool will always be liquid with this type of colostomy." b. "Eating additional fiber will bulk up your stool and decrease diarrhea." c. "Your stool will become firmer over the next couple of weeks." d. "This is abnormal. I will contact your health care provider."

A The stool from an ascending colostomy can be expected to remain liquid because little large bowel is available to reabsorb the liquid from the stool. This finding is not abnormal. Liquid stool from an ascending colostomy will not become firmer with the addition of fiber to the client's diet or with the passage of time. DIF: Applying/Application REF: 1151 KEY: Ostomy care

12. A patient who is receiving continuous enteral nutrition through a small-bore silicone feeding tube has a computed tomography (CT) scan ordered and will have to be placed in a flat position for the scan. Which action by the nurse is best? a. Shut the feeding off 30 to 60 minutes before the scan. b. Ask the health care provider to reschedule the CT scan. c. Connect the feeding tube to continuous suction during the scan. d. Send the patient to CT scan with oral suction in case of aspiration.

A The tube feeding should be shut off 30 to 60 minutes before any procedure requiring the patient to lie flat. Because the CT scan is ordered for diagnosis of patient problems, rescheduling is not usually an option. Prevention, rather than treatment, of aspiration is needed. Small-bore feeding tubes are soft and collapse easily with aspiration or suction, making nasogastric suction of gastric contents unreliable. DIF: Cognitive Level: Application REF: 932-933 TOP: Nursing Process: Planning

23. A nurse assesses a client who presents with renal calculi. Which question should the nurse ask? a. "Do any of your family members have this problem?" b. "Do you drink any cranberry juice?" c. "Do you urinate after sexual intercourse?" d. "Do you experience burning with urination?"

A There is a strong association between family history and stone formation and recurrence. Nephrolithiasis is associated with many genetic variations; therefore, the nurse should ask whether other family members have also had renal stones. The other questions do not refer to renal calculi but instead are questions that should be asked of a client with a urinary tract infection. DIF: Applying/Application REF: 1384 KEY: Urolithiasis| health screening

47. A patient is awaiting surgery for acute peritonitis. Which action will the nurse include in the plan of care? a. Position patient with the knees flexed. b. Avoid use of opioids or sedative drugs. c. Offer frequent small sips of clear liquids. d. Assist patient to breathe deeply and cough.

A There is less peritoneal irritation with the knees flexed, which will help decrease pain. Opioids and sedatives are typically given to control pain and anxiety. Preoperative patients with peritonitis are given IV fluids for hydration. Deep breathing and coughing will increase the patient's discomfort. DIF: Cognitive Level: Apply (application) REF: 944 TOP: Nursing Process: Planning

9. A client is receiving total parenteral nutrition (TPN). On assessment, the nurse notes the client's pulse is 128 beats/min, blood pressure is 98/56 mm Hg, and skin turgor is dry. What action should the nurse perform next? a. Assess the 24-hour fluid balance. b. Assess the client's oral cavity. c. Prepare to hang a normal saline bolus. d. Turn up the infusion rate of the TPN.

A This client has clinical indicators of dehydration, so the nurse calculates the client's 24-hour intake, output, and fluid balance. This information is then reported to the provider. The client's oral cavity assessment may or may not be consistent with dehydration. The nurse may need to give the client a fluid bolus, but not as an independent action.. The client's dehydration is most likely due to fluid shifts from the TPN, so turning up the infusion rate would make the problem worse, and is not done as an independent action. DIF: Analyzing/Analysis REF: 1245 KEY: Nutritional disorders| parenteral nutrition| intake and output

1. A client is in the family practice clinic. Today the client weighs 186.4 pounds (84.7 kg). Six months ago the client weighed 211.8 pounds (96.2 kg). What action by the nurse is best? a. Ask the client if the weight loss was intentional. b. Determine if there are food allergies or intolerances. c. Perform a comprehensive nutritional assessment. d. Perform a rapid bedside blood glucose test.

A This client has had a 12% weight loss. The nurse first determines if the weight loss was intentional. If not, then the nurse proceeds to a comprehensive nutritional assessment. Food intolerances are part of this assessment. Depending on risk factors and other findings, a blood glucose test may be warranted. DIF: Applying/Application REF: 1236 KEY: Nutrition| nutritional disorders| nutritional assessment| nursing assessment

15. A client has been prescribed lorcaserin (Belviq). What teaching is most appropriate? a. "Increase the fiber and water in your diet." b. "Reduce fat to less than 30% each day." c. "Report dry mouth and decreased sweating." d.. "Lorcaserin may cause loose stools for a few days."

A This drug can cause constipation, so the client should increase fiber and water in the diet to prevent this from occurring. Reducing fat in the diet is important with orlistat. Lorcaserin can cause dry mouth but not decreased sweating. Loose stools are common with orlistat. DIF: Understanding/Comprehension REF: 1249 KEY: Nutritional disorders| obesity| patient education| anorectic drugs

2. After the nurse has completed teaching a patient about the recommended amounts of foods from animal and plant sources, which of these menu selections indicates that the initial instructions about diet have been understood? a. 3 oz of pork roast, a cup of corn, and a sliced tomato b. A chicken breast and a cup of tossed salad with nonfat dressing c. A 6 oz can of tuna mixed with nonfat mayonnaise and chopped celery d. 3 oz of roast beef, 2 oz of low-fat cheese, and a half-cup of carrot sticks

A This selection is most consistent with the recommendation of the American Institute for Cancer Research that one third of the diet should be from animal sources and two thirds from plant source foods. The other choices all have higher ratios of animal origin foods to plant source foods than would be recommended. DIF: Cognitive Level: Application REF: 952-953 TOP: Nursing Process: Evaluation

5. To decrease the risk for cancers of the tongue and buccal mucosa, which information will the nurse include when teaching a patient who is seen for an annual physical exam in the outpatient clinic? a. Avoid use of cigarettes and smokeless tobacco. b. Use sunscreen when outside even on cloudy days. c. Complete antibiotics used to treat throat infections. d. Use antivirals to treat herpes simplex virus (HSV) infections.

A Tobacco use greatly increases the risk for oral cancer. Acute throat infections do not increase risk for oral cancer, although chronic irritation of the oral mucosa does increase risk. Sun exposure does not increase the risk for cancers of the buccal mucosa. Human papillomavirus (HPV) infection is associated with increased risk, but HSV infection is not a risk factor for oral cancer. DIF: Cognitive Level: Comprehension REF: 968-969 TOP: Nursing Process: Assessment

5. Which information will the nurse include when teaching adults to decrease the risk for cancers of the tongue and buccal mucosa? a. Avoid use of cigarettes and smokeless tobacco. b. Use sunscreen when outside even on cloudy days. c. Complete antibiotic courses used to treat throat infections. d. Use antivirals to treat herpes simplex virus (HSV) infections.

A Tobacco use greatly increases the risk for oral cancer. Acute throat infections do not increase the risk for oral cancer, although chronic irritation of the oral mucosa does increase risk. Sun exposure does not increase the risk for cancers of the buccal mucosa. Human papillomavirus (HPV) infection is associated with an increased risk, but HSV infection is not a risk factor for oral cancer. DIF: Cognitive Level: Apply (application) REF: 898 TOP: Nursing Process: Planning

7. A nurse teaches a client about self-care after experiencing a urinary calculus treated by lithotripsy. Which statements should the nurse include in this client's discharge teaching? (Select all that apply.) a. "Finish the prescribed antibiotic even if you are feeling better." b. "Drink at least 3 liters of fluid each day." c. "The bruising on your back may take several weeks to resolve." d. "Report any blood present in your urine." e. "It is normal to experience pain and difficulty urinating."

A, B, C The client should be taught to finish the prescribed antibiotic to ensure that he or she does not get a urinary tract infection. The client should drink at least 3 liters of fluid daily to dilute potential stone-forming crystals, prevent dehydration, and promote urine flow. After lithotripsy, the client should expect bruising that may take several weeks to resolve. The client should also experience blood in the urine for several days. The client should report any pain, fever, chills, or difficulty with urination to the provider as these may signal the beginning of an infection or the formation of another stone. DIF: Applying/Application REF: 1388 KEY: Urolithiasis| patient education

4. The nurse has taught a client about lifestyle modifications for gastroesophageal reflux disease (GERD). What statements by the client indicate good understanding of the teaching? (Select all that apply.) a. "I just joined a gym, so I hope that helps me lose weight." b. "I sure hate to give up my coffee, but I guess I have to." c. "I will eat three small meals and three small snacks a day." d. "Sitting upright and not lying down after meals will help." e. "Smoking a pipe is not a problem and I don't have to stop."

A, B, C, D Lifestyle modifications can help control GERD and include losing weight if needed; avoiding chocolate, caffeine, and carbonated beverages; eating frequent small meals or snacks; and remaining upright after meals. Tobacco is a risk factor for GERD and should be avoided in all forms. DIF: Understanding/Comprehension REF: 1113 KEY: Gastrointestinal disorders| lifestyle modifications| patient education

3. A client has been discharged to an inpatient rehabilitation center after an esophagogastrectomy. What menu selections by the client at the rehabilitation center indicate a good understanding of dietary instructions? (Select all that apply.) a. Boost™ supplement b. Greek yogurt c. Scrambled eggs d. Whole milk shake e. Whole wheat toast

A, B, C, D Malnutrition is a serious problem after this procedure. The client needs high-protein, high-calorie foods that are easy to chew and swallow. The Boost supplement, Greek yogurt, scrambled eggs, and whole milk shake are all good choices. The whole wheat bread, while heart healthy, is not a good choice as it is dry and not easy to chew and swallow. DIF: Evaluating/Synthesis REF: 1122 KEY: Gastrointestinal disorders| nutrition| patient education

1. The nurse is aware that which factors are related to the development of gastroesophageal reflux disease (GERD)? (Select all that apply.) a. Delayed gastric emptying b. Eating large meals c. Hiatal hernia d. Obesity e. Viral infections

A, B, C, D Many factors predispose a person to GERD, including delayed gastric emptying, eating large meals, hiatal hernia, and obesity. Viral infections are not implicated in the development of GERD, although infection with Helicobacter pylori is. DIF: Remembering/Knowledge REF: 1111 KEY: Gastrointestinal disorders

1. While caring for a comatose patient who is receiving continuous enteral nutrition through a soft nasogastric tube, the nurse notes the presence of new crackles in the patient's lungs. In which order will the nurse take the following actions? Put a comma and space between each answer choice (a, b, c, d, etc.) ____________________ a. Turn off the tube feeding. b. Obtain the patient's oxygen saturation. c. Check the tube feeding residual volume. d. Notify the patient's health care provider.

A, B, C, D The assessment data indicate that aspiration may have occurred, and the nurse's first action should be to turn off the tube feeding to avoid further aspiration. The next action should be to check the oxygen saturation because this may indicate the need for immediate respiratory suctioning or oxygen administration. The residual volume should be obtained because it provides data about possible causes of aspiration. Finally, the health care provider should be notified and informed of all the assessment data the nurse has just obtained. DIF: Cognitive Level: Application REF: 931-934 OBJ: Special Questions: Alternate Item Format, Prioritization TOP: Nursing Process: Implementation

3. The student nurse learns about risk factors for gastric cancer. Which factors does this include? (Select all that apply.) a. Achlorhydria b. Chronic atrophic gastritis c. Helicobacter pylori infection d. Iron deficiency anemia e. Pernicious anemia

A, B, C, E Achlorhydria, chronic atrophic gastritis, H. pylori infection, and pernicious anemia are all risk factors for developing gastric cancer. Iron deficiency anemia is not a risk factor. DIF: Remembering/Knowledge REF: 1138 KEY: Gastrointestinal disorders| gastrointestinal assessment

5. When working with older adults to promote good nutrition, what actions by the nurse are most appropriate? (Select all that apply.) a. Allow uninterrupted time for eating. b. Assess dentures for appropriate fit. c. Ensure the client has glasses on when eating. d. Provide salty foods that the client can taste. e. Serve high-calorie, high-protein snacks.

A, B, C, E Older adults need unhurried and uninterrupted time for eating. Dentures should fit appropriately and glasses, if used, should be on. High-calorie, high-protein snacks are a good choice. Salty snacks are not recommended because all adults should limit sodium in their diets. DIF: Applying/Application REF: 1238 KEY: Nutritional disorders| older adult| nutrition

1. The student nurse studying stomach disorders learns that the risk factors for acute gastritis include which of the following? (Select all that apply.) a. Alcohol b. Caffeine c. Corticosteroids d. Fruit juice e. Nonsteroidal anti-inflammatory drugs (NSAIDs)

A, B, C, E Risk factors for acute gastritis include alcohol, caffeine, corticosteroids, and chronic NSAID use. Fruit juice is not a risk factor, although in some people it does cause distress. DIF: Remembering/Knowledge REF: 1127 KEY: Gastrointestinal disorders

1. A nurse assesses a client who has had two episodes of bacterial cystitis in the last 6 months. Which questions should the nurse ask? (Select all that apply.) a. "How much water do you drink every day?" b. "Do you take estrogen replacement therapy?" c. "Does anyone in your family have a history of cystitis?" d. "Are you on steroids or other immune-suppressing drugs?" e. "Do you drink grapefruit juice or orange juice daily?"

A, B, D Fluid intake, estrogen levels, and immune suppression all can increase the chance of recurrent cystitis. Family history is usually insignificant, and cranberry juice, not grapefruit or orange juice, has been found to increase the acidic pH and reduce the risk for bacterial cystitis. DIF: Applying/Application REF: 1367 KEY: Cystitis| assessment/diagnostic examination

4. A nurse plans care for a client who is recovering from an inguinal hernia repair. Which interventions should the nurse include in this client's plan of care? (Select all that apply.) a. Encouraging ambulation three times a day b. Encouraging normal urination c. Encouraging deep breathing and coughing d. Providing ice bags and scrotal support e.. Forcibly reducing the hernia

A, B, D Postoperative care for clients with an inguinal hernia includes all general postoperative care except coughing. The nurse should promote lung expansion by encouraging deep breathing and ambulation. The nurse should encourage normal urination, including allowing the client to stand, and should provide scrotal support and ice bags to prevent swelling. A hernia should never be forcibly reduced, and this procedure is not part of postoperative care. DIF: Applying/Application REF: 1148 KEY: Herniation| postoperative care

5. A nurse cares for clients with urinary incontinence. Which types of incontinence are correctly paired with their clinical manifestation? (Select all that apply.) a. Stress incontinence - Urine loss with physical exertion b. Urge incontinence - Large amount of urine with each occurrence c. Functional incontinence - Urine loss results from abnormal detrusor contractions d. Overflow incontinence - Constant dribbling of urine e. Reflex incontinence - Leakage of urine without lower urinary tract disorder

A, B, D Stress incontinence is a loss of urine with physical exertion, coughing, sneezing, or exercising. Urge incontinence presents with an abrupt and strong urge to void and usually has a large amount of urine released with each occurrence. Overflow incontinence occurs with bladder distention and results in a constant dribbling of urine. Functional incontinence is the leakage of urine caused by factors other than a disorder of the lower urinary tract. Reflex incontinence results from abnormal detrusor contractions from a neurologic abnormality. DIF: Remembering/Knowledge REF: 1374 KEY: Urinary incontinence

2. The nurse is caring for a client who had an esophagectomy 3 days ago and was extubated yesterday. What actions may the nurse delegate to the unlicensed assistive personnel (UAP)? (Select all that apply.) a. Assisting with position changes and getting out of bed b. Keeping the head of the bed elevated to at least 30 degrees c. Reminding the client to use the spirometer every 4 hours d. Taking and recording vital signs per hospital protocol e. Titrating oxygen based on the client's oxygen saturations

A, B, D The UAP can assist with mobility, keep the head of the bed elevated, and take and record vital signs. The client needs to use the spirometer every 1 to 2 hours. The nurse titrates oxygen. DIF: Applying/Application REF: 1122 KEY: Gastrointestinal disorders| postoperative nursing| delegation| unlicensed assistive personnel (UAP)

7. A nurse plans care for a client who has chronic diarrhea. Which actions should the nurse include in this client's plan of care? (Select all that apply.) a. Using premoistened disposable wipes for perineal care b. Turning the client from right to left every 2 hours c. Using an antibacterial soap to clean after each stool d. Applying a barrier cream to the skin after cleaning e. Keeping broken skin areas open to air to promote healing

A, B, D The nurse should use premoistened disposable wipes instead of toilet paper for perineal care, or mild soap and warm water after each stool. Antibacterial soap would be too abrasive and damage good bacteria on the skin. The nurse should apply a thin layer of a medicated protective barrier after cleaning the skin. The client should be re-positioned frequently so that he or she is kept off the affected area, and open skin areas should be covered with DuoDerm or Tegaderm occlusive dressing to promote rapid healing. DIF: Remembering/Knowledge REF: 1166 KEY: Bowel care

1. Which of the nurse's assigned patients should be referred to the dietitian for a complete nutritional assessment (select all that apply)? a. A 48-yr-old patient with rheumatoid arthritis who takes prednisone daily b. A 23-yr-old patient who has a history of fluctuating weight gains and losses c. A 35-yr-old patient who complains of intermittent nausea for the past 2 days d. A 64-yr-old patient who is admitted for débridement of an infected surgical wound e. A 52-yr-old patient admitted with chest pain and possible myocardial infarction (MI)

A, B, D Weight fluctuations, use of corticosteroids, and draining or infected wounds all suggest that the patient may be at risk for malnutrition. Patients with chest pain or MI are not usually poorly nourished. Although vomiting that lasts 5 days places a patient at risk, nausea that has persisted for 2 days does not always indicate poor nutritional status or risk for health problems caused by poor nutrition. DIF: Cognitive Level: Apply (application) REF: 857 OBJ: Special Questions: Multiple Patients TOP: Nursing Process: Implementation

2. A client has a gastrointestinal hemorrhage and is prescribed two units of packed red blood cells. What actions should the nurse perform prior to hanging the blood? (Select all that apply.) a. Ask a second nurse to double-check the blood. b. Prime the IV tubing with normal saline. c. Prime the IV tubing with dextrose in water. d. Take and record a set of vital signs. e. Teach the client about reaction manifestations.

A, B, D, E Prior to starting a blood transfusion, the nurse asks another nurse to double-check the blood (and client identity), primes the IV tubing with normal saline, takes and records a baseline set of vital signs, and teaches the client about manifestations to report. The IV tubing is not primed with dextrose in water. DIF: Applying/Application REF: 1136 KEY: Patient safety| blood transfusions

6. A client who had a partial gastrectomy has several expected nutritional problems. What actions by the nurse are best to promote better nutrition? (Select all that apply.) a. Administer vitamin B12 injections. b. Ask the provider about folic acid replacement. c. Educate the client on enteral feedings. d.. Obtain consent for total parenteral nutrition. e. Provide iron supplements for the client.

A, B, E After gastrectomy, clients are at high risk for anemia due to vitamin B12 deficiency, folic acid deficiency, or iron deficiency. The nurse should provide supplements for all these nutrients. The client does not need enteral feeding or total parenteral nutrition. DIF: Understanding/Comprehension REF: 1141 KEY: Gastrointestinal disorders| anemia| supplements

6. A nurse teaches a client with a history of calcium phosphate urinary stones. Which statements should the nurse include in this client's dietary teaching? (Select all that apply.) a. "Limit your intake of food high in animal protein." b. "Read food labels to help minimize your sodium intake." c. "Avoid spinach, black tea, and rhubarb." d. "Drink white wine or beer instead of red wine." e. "Reduce your intake of milk and other dairy products."

A, B, E Clients with calcium phosphate urinary stones should be taught to limit the intake of foods high in animal protein, sodium, and calcium. Clients with calcium oxalate stones should avoid spinach, black tea, and rhubarb. Clients with uric acid stones should avoid red wine. DIF: Applying/Application REF: 1388 KEY: Urolithiasis| nutritional requirements

8. A nurse teaches a female client who has stress incontinence. Which statements should the nurse include about pelvic muscle exercises? (Select all that apply.) a. "When you start and stop your urine stream, you are using your pelvic muscles." b. "Tighten your pelvic muscles for a slow count of 10 and then relax for a slow count of 10." c. "Pelvic muscle exercises should only be performed sitting upright with your feet on the floor." d. "After you have been doing these exercises for a couple days, your control of urine will improve." e. "Like any other muscle in your body, you can make your pelvic muscles stronger by contracting them."

A, B, E The client should be taught that the muscles used to start and stop urination are pelvic muscles, and that pelvic muscles can be strengthened by contracting and relaxing them. The client should tighten pelvic muscles for a slow count of 10 and then relax the muscles for a slow count of 10, and perform this exercise 15 times while in lying-down, sitting-up, and standing positions. The client should begin to notice improvement in control of urine after several weeks of exercising the pelvic muscles. DIF: Understanding/Comprehension REF: 1377 KEY: Urinary incontinence| patient education

3. A nurse assesses a client with irritable bowel syndrome (IBS). Which questions should the nurse include in this client's assessment? (Select all that apply.) a. "Which food types cause an exacerbation of symptoms?" b. "Where is your pain and what does it feel like?" c. "Have you lost a significant amount of weight lately?" d. "Are your stools soft, watery, and black in color?" e. "Do you experience nausea associated with defecation?"

A, B, E The nurse should ask the client about factors that may cause exacerbations of symptoms, including food, stress, and anxiety. The nurse should also assess the location, intensity, and quality of the client's pain, and nausea associated with defecation or meals. Clients who have IBS do not usually lose weight and stools are not black in color. DIF: Applying/Application REF: 1145 KEY: Irritable bowel| assessment/diagnostic examination

7. A nurse is preparing to administer pantoprazole (Protonix) intravenously. What actions by the nurse are most appropriate? (Select all that apply.) a. Administer the drug through a separate IV line. b. Infuse pantoprazole using an IV pump. c. Keep the drug in its original brown bag. d. Take vital signs frequently during infusion. e. Use an in-line IV filter when infusing.

A, B, E When infusing pantoprazole, use a separate IV line, a pump, and an in-line filter.. A brown wrapper and frequent vital signs are not needed. DIF: Applying/Application REF: 1130 KEY: Gastrointestinal disorders| proton pump inhibitors

6. A nurse is teaching clients with gastroesophageal reflux disease (GERD) about foods to avoid. Which foods should the nurse include in the teaching? (Select all that apply.) a. Chocolate b. Decaffeinated coffee c. Citrus fruits d. Peppermint e. Tomato sauce

A, C, D, E Chocolate, citrus fruits such as oranges and grapefruit, peppermint and spearmint, and tomato-based products all contribute to the reflux associated with GERD. Caffeinated teas, coffee, and sodas should be avoided. DIF: Understanding/Comprehension REF: 1111 KEY: Gastrointestinal disorders| patient education

1. The nurse understands that malnutrition can occur in hospitalized clients for several reasons. Which are possible reasons for this to occur? (Select all that apply.) a. Cultural food preferences b. Family bringing snacks c. Increased need for nutrition d. Need for NPO status e. Staff shortages

A, C, D, E Many factors increase the hospitalized client's risk for nutritional deficits. Cultural food preferences may make hospital food unpalatable. Ill clients have increased nutritional needs but may be NPO for testing or treatment, or have a loss of appetite from their illness. Staff shortages impact clients who need to be fed or assisted with meals. The family may bring snacks that are either healthy or unhealthy, so without further information, the nurse cannot assume the snacks are leading to malnutrition. DIF: Remembering/Knowledge REF: 1237 KEY: Nutritional disorders

5. A nurse cares for a client who has been diagnosed with a small bowel obstruction. Which assessment findings should the nurse correlate with this diagnosis? (Select all that apply.) a. Serum potassium of 2.8 mEq/L b. Loss of 15 pounds without dieting c. Abdominal pain in upper quadrants d. Low-pitched bowel sounds e. Serum sodium of 121 mEq/L

A, C, E Small bowel obstructions often lead to severe fluid and electrolyte imbalances. The client is hypokalemic (normal range is 3.5 to 5.0 mEq/L) and hyponatremic (normal range is 136 to 145 mEq/L). Abdominal pain across the upper quadrants is associated with small bowel obstruction.. Dramatic weight loss without dieting followed by bowel obstruction leads to the probable development of colon cancer. High-pitched sounds may be noted with small bowel obstructions. DIF: Applying/Application REF: 1159 KEY: Intestinal obstruction| assessment/diagnostic examination

1. A nurse inserts a nasogastric (NG) tube for an adult client who has a bowel obstruction. Which actions does the nurse perform correctly? (Select all that apply.) a. Performs hand hygiene and positions the client in high-Fowler's position, with pillows behind the head and shoulders b. Instructs the client to extend the neck against the pillow once the NG tube has reached the oropharynx c. Checks for correct placement by checking the pH of the fluid aspirated from the tube d. Secures the NG tube by taping it to the client's nose and pinning the end to the pillowcase e. Connects the NG tube to intermittent medium suction with an anti-reflux valve on the air vent

A, C, E The client's head should be flexed forward once the NG tube has reached the oropharynx. The NG tube should be secured to the client's gown, not to the pillowcase, because it could become dislodged easily. All the other actions are appropriate. DIF: Applying/Application REF: 1159 KEY: Intestinal obstruction

1. During a busy day, the nurse admits all of the following patients to the medical-surgical unit. Which patients are most important to refer to the dietitian for a complete nutritional assessment (select all that apply)? a. A 24-year-old who has a history of weight gains and losses b. A 53-year-old who complains of intermittent nausea for the past 2 days c. A 66-year-old who is admitted for débridement of an infected surgical wound d. A 45-year-old admitted with chest pain and possible myocardial infarction (MI) e. A 32-year-old with rheumatoid arthritis who takes prednisone (Deltasone) daily

A, C, E Weight fluctuations, use of corticosteroids, and draining or infected wounds all suggest that the patient may be at risk for malnutrition. Patients with chest pain or MI are not usually poorly nourished. Although vomiting that lasts 5 days places a patient at risk, nausea that has persisted for 2 days does not always indicate poor nutritional status or risk for health problems caused by poor nutrition. DIF: Cognitive Level: Application REF: 924 OBJ: Special Questions: Alternate Item Format, Multiple Patients TOP: Nursing Process: Implementation

2. A nurse teaches a client about self-catheterization in the home setting. Which statements should the nurse include in this client's teaching? (Select all that apply.) a. "Wash your hands before and after self-catheterization." b. "Use a large-lumen catheter for each catheterization." c. "Use lubricant on the tip of the catheter before insertion." d. "Self-catheterize at least twice a day or every 12 hours." e. "Use sterile gloves and sterile technique for the procedure." f. "Maintain a specific schedule for catheterization."

A, C, F The key points in self-catheterization include washing hands, using lubricants, and maintaining a regular schedule to avoid distention and retention of urine that leads to bacterial growth. A smaller rather than a larger lumen catheter is preferred. The client needs to catheterize more often than every 12 hours. Self-catheterization in the home is a clean procedure. DIF: Applying/Application REF: 1382 KEY: Urinary incontinence| patient education

4. A client has dumping syndrome. What menu selections indicate the client understands the correct diet to manage this condition? (Select all that apply.) a. Canned unsweetened apricots b. Coffee cake c. Milk shake d. Potato soup e. Steamed broccoli

A, D Canned apricots and potato soup are appropriate selections as they are part of a high-protein, high-fat, low- to moderate-carbohydrate diet. Coffee cake and other sweets must be avoided. Milk products and sweet drinks such as shakes must be avoided. Gas-forming foods such as broccoli must also be avoided. DIF: Remembering/Knowledge REF: 1141 KEY: Gastrointestinal disorders| nutrition| patient education

8. A nurse cares for a client who has a nasogastric (NG) tube. Which actions should the nurse take? (Select all that apply.) a. Assess for proper placement of the tube every 4 hours. b. Flush the tube with water every hour to ensure patency. c. Secure the NG tube to the client's upper lip. d. Disconnect suction when auscultating bowel peristalsis. e. Monitor the client's skin around the tube site for irritation.

A, D, E The nurse should assess for proper placement, tube patency, and output every 4 hours. The nurse should also monitor the skin around the tube for irritation and secure the tube to the client's nose. When auscultating bowel sounds for peristalsis, the nurse should disconnect suction. DIF: Applying/Application REF: 1159 KEY: Drain

3. A nurse is reviewing laboratory values for several clients. Which value causes the nurse to conduct nutritional assessments as a priority? a. Albumin: 3.5 g/dL b. Cholesterol: 142 mg/dL c. Hemoglobin: 9.8 mg/dL d. Prealbumin: 28 mg/dL

B A cholesterol level below 160 mg/dL is a possible indicator of malnutrition, so this client would be at highest priority for a nutritional assessment. The albumin and prealbumin levels are normal. The low hemoglobin could be from several problems, including dietary deficiencies, hemodilution, and bleeding. DIF: Remembering/Knowledge REF: 1239 KEY: Nutritional disorders| nutritional assessment| laboratory values

4. The nurse is obtaining a history for a 23-year-old woman who is being evaluated for acute lower abdominal pain and vomiting. Which question will be most useful in determining the cause of the patient's symptoms? a. "Is it possible that you are pregnant?" b. "Can you tell me more about the pain?" c. "What type of foods do you usually eat?" d. "What is your usual elimination pattern?"

B A complete description of the pain provides clues about the cause of the problem. The usual diet and elimination patterns are less helpful in determining the reason for the patient's symptoms. Although the nurse should ask whether the patient is pregnant to determine whether the patient might have an ectopic pregnancy and before any radiology studies are done, this information is not the most useful in determining the cause of the pain. DIF: Cognitive Level: Application REF: 1015-1016 TOP: Nursing Process: Assessment

34. Which information about a patient who has just been admitted to the hospital with nausea and vomiting will require the most rapid intervention by the nurse? a. The patient has taken only sips of water. b. The patient is lethargic and difficult to arouse. c. The patient's chart indicates a recent resection of the small intestine. d. The patient has been vomiting several times a day for the last 4 days.

B A lethargic patient is at risk for aspiration, and the nurse will need to position the patient to decrease aspiration risk. The other information also is important to collect, but it does not require as quick action as the risk for aspiration. DIF: Cognitive Level: Application REF: 964 OBJ: Special Questions: Prioritization TOP: Nursing Process: Assessment

45. A 33-yr-old male patient with a gunshot wound to the abdomen undergoes surgery, and a colostomy is formed as shown in the accompanying figure. Which information will be included in patient teaching? a. Stool will be expelled from both stomas. b. This type of colostomy is usually temporary. c. Soft, formed stool can be expected as drainage. d. Irrigations can regulate drainage from the stomas.

B A loop, or double-barrel stoma, is usually temporary. Stool will be expelled from the proximal stoma only. The stool from the transverse colon will be liquid and regulation through irrigations will not be possible. DIF: Cognitive Level: Apply (application) REF: 959 TOP: Nursing Process: Implementation

26. When implementing the initial plan of care for a patient admitted with acute diverticulitis, the nurse will plan to a. give stool softeners. b. administer IV fluids. c. order a diet high in fiber and fluids. d. prepare the patient for colonoscopy.

B A patient with acute diverticulitis will be NPO and given parenteral fluids. A diet high in fiber and fluids will be implemented before discharge. Bulk-forming laxatives, rather than stool softeners, are usually given, and these will be implemented later in the hospitalization. The patient with acute diverticulitis will not have a colonoscopy because of the risk for perforation and peritonitis. DIF: Cognitive Level: Application REF: 1047 TOP: Nursing Process: Planning

25. The nurse explains to a patient with a new ileostomy that after the bowel adjusts to the ileostomy, the usual drainage will be about a. 1 cup. b. 2 cups. c. 3 cups. d. 1 quart.

B After the proximal small bowel adapts to reabsorb more fluid, the average amount of ileostomy drainage is about 500 mL daily. DIF: Cognitive Level: Comprehension REF: 1042 | 1044-1045 TOP: Nursing Process: Implementation

7. Parenteral nutrition (PN) containing amino acids and dextrose was ordered and hung 24 hours ago for a malnourished patient. The nurse observes that about 50 mL remain in the PN container. Which action is best for the nurse to take? a. Ask the health care provider to clarify the written PN order. b. Add a new container of PN using the current tubing and filter. c. Hang a new container of PN and change the IV tubing and filter. d. Infuse the remaining 50 mL and then hang a new container of PN.

B All PN solutions are changed at 24 hours. PN solutions containing dextrose and amino acids require a change in tubing and filter every 72 hours rather than daily. Infusion of the additional 50 mL will increase patient risk for infection. Changing the IV tubing and filter more frequently than required will unnecessarily increase costs. The nurse (not the health care provider) is responsible for knowing the indicated times for tubing and filter changes. DIF: Cognitive Level: Application REF: 937 TOP: Nursing Process: Implementation

18. A nurse is caring for a morbidly obese client. What comfort measure is most important for the nurse to delegate to the unlicensed assistive personnel (UAP)? a. Designating "quiet time" so the client can rest b.. Ensuring siderails are not causing excess pressure c. Providing oral care before and after meals and snacks d. Relaying any reports of pain to the registered nurse

B All actions are good for client comfort, but when dealing with an obese client, the staff should take extra precautions, such as ensuring the siderails are not putting pressure on the client's tissues. The other options are appropriate for any client, and are not specific to obese clients. DIF: Applying/Application REF: 1246 KEY: Nutritional disorders| obesity| comfort measures| unlicensed assistive personnel (UAP)

6. An older female client has been prescribed esomeprazole (Nexium) for treatment of chronic gastric ulcers. What teaching is particularly important for this client? a. Check with the pharmacist before taking other medications. b. Increase intake of calcium and vitamin D. c. Report any worsening of symptoms to the provider. d.. Take the medication as prescribed by the provider.

B All of this advice is appropriate for any client taking this medication. However, long-term use is associated with osteoporosis and osteoporosis-related fractures. This client is already at higher risk for this problem and should be instructed to increase calcium and vitamin D intake. The other options are appropriate for any client taking any medication and are not specific to the use of esomeprazole. DIF: Applying/Application REF: 1130 KEY: Gastrointestinal disorders| osteoporosis| proton pump inhibitors| patient education

4. A client with an esophageal tumor has difficulty swallowing and has been working with a speech-language pathologist. What assessment finding by the nurse indicates that the priority goal for this problem is being met? a. Choosing foods that are easy to swallow b. Lungs clear after meals and snacks c. Properly performing swallowing exercises d. Weight unchanged after 2 weeks

B All these assessment findings are positive for this client. However, this client is at high risk for aspiration. Clear lungs after eating indicates no aspiration has occurred. Choosing easy-to-swallow foods, performing swallowing checks, and having an unchanged weight do not assess aspiration, and therefore do not indicate that the priority goal has been met. DIF: Evaluating/Synthesis REF: 118 KEY: Gastrointestinal disorders| respiratory assessment| patient safety

31. A patient with renal calculi is hospitalized with gross hematuria and severe colicky left flank pain. Which nursing action will be of highest priority at this time? a. Encourage oral fluid intake. b. Administer prescribed analgesics. c. Monitor temperature every 4 hours. d. Give antiemetics as needed for nausea.

B Although all of the nursing actions may be used for patients with renal lithiasis, the patient's presentation indicates that management of pain is the highest priority action. If the patient has urinary obstruction, increasing oral fluids may increase the symptoms. There is no evidence of infection or nausea. DIF: Cognitive Level: Application REF: 1137-1138 | 1139-1141 | 1140 OBJ: Special Questions: Prioritization TOP: Nursing Process: Implementation

52. The nurse is admitting a 67-yr-old patient with new-onset steatorrhea. Which question is most important for the nurse to ask? a. "How much milk do you usually drink?" b. "Have you noticed a recent weight loss?" c. "What time of day do your bowels move?" d. "Do you eat meat or other animal products?"

B Although all of the questions provide useful information, it is most important to determine if the patient has an imbalance in nutrition because of the steatorrhea. DIF: Cognitive Level: Analyze (analysis) REF: 968 OBJ: Special Questions: Prioritization TOP: Nursing Process: Assessment

5. Two days after an exploratory laparotomy with a resection of a short segment of small bowel, a patient complains of gas pains and abdominal distention. Which nursing action is best to take at this time? a. Give a return-flow enema. b. Assist the patient to ambulate. c. Administer the ordered IV morphine sulfate. d. Insert the ordered promethazine (Phenergan) suppository.

B Ambulation will improve peristalsis and help the patient eliminate flatus and reduce gas pain. Morphine will further reduce peristalsis. A return-flow enema may decrease the patient's symptoms, but ambulation is less invasive and should be tried first. Promethazine (Phenergan) is used as an antiemetic rather than to decrease gas pains or distention. DIF: Cognitive Level: Application REF: 1016-1017 TOP: Nursing Process: Implementation

33. The nurse is assessing a patient who had a total gastrectomy 8 hours ago. What information is most important to report to the health care provider? a. Hemoglobin (Hgb) 10.8 g/dL b. Temperature 102.1°F (38.9°C) c. Absent bowel sounds in all quadrants d. Scant nasogastric (NG) tube drainage

B An elevation in temperature may indicate leakage at the anastomosis, which may require return to surgery or keeping the patient NPO. The other findings are expected in the immediate postoperative period for patients who have this surgery and do not require any urgent action. DIF: Cognitive Level: Analyze (analysis) REF: 921 TOP: Nursing Process: Assessment

2. A nurse reviews the laboratory findings of a client with a urinary tract infection. The laboratory report notes a "shift to the left" in a client's white blood cell count. Which action should the nurse take? a. Request that the laboratory perform a differential analysis on the white blood cells. b. Notify the provider and start an intravenous line for parenteral antibiotics. c. Collaborate with the unlicensed assistive personnel (UAP) to strain the client's urine for renal calculi. d. Assess the client for a potential allergic reaction and anaphylactic shock.

B An increase in band cells creates a "shift to the left." A left shift most commonly occurs with urosepsis and is seen rarely with uncomplicated urinary tract infections. The nurse will be administering antibiotics, most likely via IV, so he or she should notify the provider and prepare to give the antibiotics. The shift to the left is part of a differential white blood cell count. The nurse would not need to strain urine for stones. Allergic reactions are associated with elevated eosinophil cells, not band cells. DIF: Applying/Application REF: 1370 KEY: Cystitis| assessment/diagnostic examination

9. A patient being admitted with an acute exacerbation of ulcerative colitis reports crampy abdominal pain and passing 15 or more bloody stools a day. The nurse will plan to a. administer IV metoclopramide (Reglan). b. discontinue the patient's oral food intake. c. administer cobalamin (vitamin B12) injections. d. teach the patient about total colectomy surgery.

B An initial therapy for an acute exacerbation of inflammatory bowel disease (IBD) is to rest the bowel by making the patient NPO. Metoclopramide increases peristalsis and will worsen symptoms. Cobalamin (vitamin B12) is absorbed in the ileum, which is not affected by ulcerative colitis. Although total colectomy is needed for some patients, there is no indication that this patient is a candidate. DIF: Cognitive Level: Apply (application) REF: 946 TOP: Nursing Process: Planning

4. A nurse assesses a client who is recovering from a hemorrhoidectomy that was done the day before. The nurse notes that the client has lower abdominal distention accompanied by dullness to percussion over the distended area. Which action should the nurse take? a. Assess the client's heart rate and blood pressure. b. Determine when the client last voided. c. Ask if the client is experiencing flatus. d. Auscultate all quadrants of the client's abdomen.

B Assessment findings indicate that the client may have an over-full bladder. In the immediate postoperative period, the client may experience difficulty voiding due to urinary retention. The nurse should assess when the client last voided. The client's vital signs may be checked after the nurse determines the client's last void. Asking about flatus and auscultating bowel sounds are not related to a hemorrhoidectomy. DIF: Applying/Application REF: 1165 KEY: Postoperative nursing| urinary retention

18. The nurse preparing for the annual physical exam of a 50-yr-old man will plan to teach the patient about a. endoscopy. b. colonoscopy. c. computerized tomography screening. d. carcinoembryonic antigen (CEA) testing.

B At age 50 years, individuals with an average risk for colorectal cancer (CRC) should begin screening for CRC. Colonoscopy is the gold standard for CRC screening. The other diagnostic tests are not recommended as part of a routine annual physical exam at age 50 years. DIF: Cognitive Level: Apply (application) REF: 954 TOP: Nursing Process: Planning

18. When preparing for an annual physical exam for a patient who is 50 years old, the nurse will plan to teach the patient about a. endoscopy. b. colonoscopy. c. computerized tomography screening. d. carcinoembryonic antigen (CEA) testing.

B At age 50, individuals with an average risk for colorectal cancer (CRC) should begin screening for CRC. Colonoscopy is the gold standard for CRC screening. The other diagnostic tests are not recommended as part of a routine annual physical exam at age 50. DIF: Cognitive Level: Application REF: 1035-1037 TOP: Nursing Process: Planning

1. A patient who is hospitalized with watery, incontinent diarrhea is diagnosed with Clostridium difficile. Which action will the nurse include in the plan of care? a. Order a diet with no dairy products for the patient. b. Place the patient in a private room with contact isolation. c. Teach the patient about why antibiotics are not being used. d. Educate the patient about proper food handling and storage.

B Because C. difficile is highly contagious, the patient should be placed in a private room and contact precautions should be used. There is no need to restrict dairy products for this type of diarrhea. Metronidazole (Flagyl) is frequently used to treat C. difficile. Improper food handling and storage do not cause C. difficile. DIF: Cognitive Level: Application REF: 1009-1010 TOP: Nursing Process: Planning

10. Which nursing action will the nurse include in the plan of care for a 35-yr-old male patient admitted with an exacerbation of inflammatory bowel disease (IBD)? a. Restrict oral fluid intake. c. Ambulate six times daily. b. Monitor stools for blood. d. Increase dietary fiber intake.

B Because anemia or hemorrhage may occur with IBD, stools should be assessed for the presence of blood. The other actions would not be appropriate for the patient with IBD. Dietary fiber may increase gastrointestinal motility and exacerbate the diarrhea, severe fatigue is common with IBD exacerbations, and dehydration may occur. DIF: Cognitive Level: Apply (application) REF: 949 TOP: Nursing Process: Planning

26. Nursing staff on a hospital unit are reviewing rates of health care-associated infections (HAI) of the urinary tract. Which nursing action will be most helpful in decreasing the risk for urinary HAI in patients admitted to the hospital? a. Testing urine with a dipstick daily for nitrites b. Avoiding unnecessary urinary catheterizations c. Encouraging adequate oral fluid and nutritional intake d. Providing perineal hygiene to patients daily and as needed

B Because catheterization bypasses many of the protective mechanisms that prevent urinary tract infection (UTI), avoidance of catheterization is the most effective means of reducing HAI. The other actions will also be helpful but are not as useful as decreasing urinary catheter use. DIF: Cognitive Level: Analyze (analysis) REF: 1061 OBJ: Special Questions: Prioritization TOP: Nursing Process: Planning

14. Which finding for a patient who has been taking orlistat (Xenical) is most important to report to the health care provider? a. The patient frequently has liquid stools. b. The patient is pale and has many bruises. c. The patient complains of bloating after meals. d. The patient is experiencing a weight loss plateau.

B Because orlistat blocks the absorption of fat-soluble vitamins, the patient may not be receiving an adequate amount of vitamin K, resulting in a decrease in clotting factors. Abdominal bloating and liquid stools are common side effects of orlistat and indicate that the nurse should remind the patient that fat in the diet may increase these side effects. Weight loss plateaus are normal during weight reduction. DIF: Cognitive Level: Analyze (analysis) REF: 884 OBJ: Special Questions: Prioritization TOP: Nursing Process: Evaluation

16. The nurse obtains these assessment data for a patient who has been taking orlistat (Xenical) for several months as part of a weight loss program. Which finding is most important to report to the health care provider? a. The patient frequently has liquid stools. b. The patient is pale and has many bruises. c. The patient is experiencing a plateau in weight loss. d. The patient complains of abdominal bloating after meals.

B Because orlistat blocks the absorption of fat-soluble vitamins, the patient may not be receiving an adequate amount of vitamin K, resulting in a decrease in clotting factors. Abdominal bloating and liquid stools are common side effects of orlistat and indicate that the nurse should remind the patient that fat in the diet may increase these side effects. Weight loss plateaus are normal during weight reduction. DIF: Cognitive Level: Application REF: 953-954 OBJ: Special Questions: Prioritization TOP: Nursing Process: Evaluation

34. A patient in the emergency department has just been diagnosed with peritonitis caused by a ruptured diverticulum. Which prescribed intervention will the nurse implement first? a. Insert a urinary catheter to drainage. b. Infuse metronidazole (Flagyl) 500 mg IV. c. Send the patient for a computerized tomography scan. d. Place a nasogastric (NG) tube to intermittent low suction.

B Because peritonitis can be fatal if treatment is delayed, the initial action should be to start antibiotic therapy (after any ordered cultures are obtained). The other actions can be done after antibiotic therapy is initiated. DIF: Cognitive Level: Analyze (analysis) REF: 947 OBJ: Special Questions: Prioritization TOP: Nursing Process: Implementation

36. A 49-yr-old man has been admitted with hypotension and dehydration after 3 days of nausea and vomiting. Which prescribed action will the nurse implement first? a. Insert a nasogastric (NG) tube. b. Infuse normal saline at 250 mL/hr. c. Administer IV ondansetron (Zofran). d. Provide oral care with moistened swabs.

B Because the patient has severe dehydration, rehydration with IV fluids is the priority. The other orders should be accomplished after the IV fluids are initiated. DIF: Cognitive Level: Analyze (analysis) REF: 915 OBJ: Special Questions: Prioritization TOP: Nursing Process: Implementation

36. A 48-yr-old male patient who weighs 242 lb (110 kg) undergoes a nephrectomy for massive kidney trauma from a motor vehicle crash. Which postoperative assessment finding is most important to communicate to the surgeon? a. Blood pressure is 102/58. b. Urine output is 20 mL/hr for 2 hours. c. Incisional pain level is reported as 9/10. d. Crackles are heard at bilateral lung bases.

B Because the urine output should be at least 0.5 mL/kg/hr, a 40-mL output for 2 hours indicates that the patient may have decreased renal perfusion because of bleeding, inadequate fluid intake, or obstruction at the suture site. The blood pressure requires ongoing monitoring but does not indicate inadequate perfusion at this time. The patient should cough and deep breathe, but the crackles do not indicate a need for an immediate change in therapy. The incisional pain should be addressed, but this is not as potentially life threatening as decreased renal perfusion. In addition, the nurse can medicate the patient for pain. DIF: Cognitive Level: Analyze (analysis) REF: 1063 OBJ: Special Questions: Prioritization TOP: Nursing Process: Assessment

1. A 46-yr-old female patient returns to the clinic with recurrent dysuria after being treated with trimethoprim and sulfamethoxazole for 3 days. Which action will the nurse plan to take? a. Remind the patient about the need to drink 1000 mL of fluids daily. b. Obtain a midstream urine specimen for culture and sensitivity testing. c. Suggest that the patient use acetaminophen (Tylenol) to relieve symptoms. d. Teach the patient to take the prescribed trimethoprim and sulfamethoxazole for 3 more days.

B Because uncomplicated urinary tract infections (UTIs) are usually successfully treated with 3 days of antibiotic therapy, this patient will need a urine culture and sensitivity to determine appropriate antibiotic therapy. Acetaminophen would not be as effective as other over-the-counter medications such as phenazopyridine in treating dysuria. The fluid intake should be increased to at least 1800 mL/day. Because the UTI has persisted after treatment with trimethoprim and sulfamethoxazole , the patient is likely to need a different antibiotic. DIF: Cognitive Level: Apply (application) REF: 1041 TOP: Nursing Process: Planning

3. When working with an obese patient who is enrolled in a behavior modification program, which nursing action is appropriate? a. Having the patient write down the caloric intake of each meal b. Asking the patient about situations that tend to increase appetite c. Encouraging the patient to eat small amounts throughout the day rather than having scheduled meals d. Suggesting that the patient have a reward, such as a piece of sugarless candy, after achieving a weight-loss goal

B Behavior modification programs focus on how and when the person eats and de-emphasize aspects such as calorie counting. Nonfood rewards are recommended for achievement of weight-loss goals. Patients are often taught to restrict eating to designated meals when using behavior modification. DIF: Cognitive Level: Application REF: 953 TOP: Nursing Process: Implementation

3. Which nursing action is appropriate when coaching obese adults enrolled in a behavior modification program? a. Having the adults write down the caloric intake of each meal b. Asking the adults about situations that tend to increase appetite c. Suggesting that the adults plan rewards, such as sugarless candy, for achieving their goals d. Encouraging the adults to eat small amounts frequently rather than having scheduled meals

B Behavior modification programs focus on how and when the person eats and de-emphasize aspects such as calorie counting. Nonfood rewards are recommended for achievement of weight-loss goals. Patients are often taught to restrict eating to designated meals when using behavior modification. DIF: Cognitive Level: Apply (application) REF: 883 TOP: Nursing Process: Implementation

40. Which information obtained by the nurse interviewing a 30-yr-old male patient is most important to communicate to the health care provider? a. The patient has a history of constipation. b. The patient has noticed blood in the stools. c. The patient had an appendectomy at age 27. d. The patient smokes a pack/day of cigarettes.

B Blood in the stools is a possible clinical manifestation of colorectal cancer and requires further assessment by the health care provider. The other patient information will also be communicated to the health care provider, but does not indicate an urgent need for further testing or intervention. DIF: Cognitive Level: Analyze (analysis) REF: 955 OBJ: Special Questions: Prioritization TOP: Nursing Process: Assessment

20. Before undergoing a colon resection for cancer of the colon, a patient has an elevated carcinoembryonic antigen (CEA) test. The nurse explains that the test is used to a. confirm the diagnosis of colon cancer. b. monitor the tumor status after surgery. c. identify the extent of cancer spread or metastasis. d. determine the need for postoperative chemotherapy.

B CEA is used to monitor for cancer recurrence after surgery. CEA levels do not help determine whether there is metastasis of the cancer. Confirmation of the diagnosis is made on the basis of biopsy. Chemotherapy use is based on factors other than CEA. DIF: Cognitive Level: Comprehension REF: 1036-1037 TOP: Nursing Process: Implementation

20. A patient preparing to undergo a colon resection for cancer of the colon asks about the elevated carcinoembryonic antigen (CEA) test result. The nurse explains that the test is used to a. identify any metastasis of the cancer. b. monitor the tumor status after surgery. c. confirm the diagnosis of a specific type of cancer. d. determine the need for postoperative chemotherapy.

B CEA is used to monitor for cancer recurrence after surgery. CEA levels do not help to determine whether there is metastasis of the cancer. Confirmation of the diagnosis is made on the basis of biopsy. Chemotherapy use is based on factors other than CEA. DIF: Cognitive Level: Understand (comprehension) REF: 955 TOP: Nursing Process: Implementation

32. Which nursing action should the nurse who is caring for a patient who has had an ileal conduit for several years delegate to nursing assistive personnel (NAP)? a. Assess for symptoms of urinary tract infection (UTI). b. Change the ostomy appliance. c. Choose the appropriate ostomy bag. d. Monitor the appearance of the stoma.

B Changing the ostomy appliance for a stable patient could be done by NAP. Assessments of the site, choosing the appropriate ostomy bag, and assessing for (UTI) symptoms require more education and scope of practice and should be done by the RN. DIF: Cognitive Level: Application REF: 1157 | 1159-1160 | 1158 OBJ: Special Questions: Delegation TOP: Nursing Process: Planning

15. After obtaining the health history for a 25-year-old who smokes two packs of cigarettes daily, the nurse will plan to do teaching about the increased risk for a. kidney stones. b. bladder cancer. c. bladder infection. d. interstitial cystitis.

B Cigarette smoking is a risk factor for bladder cancer. The patient's risk for developing interstitial cystitis, urinary tract infection (UTI), or kidney stones will not be reduced by quitting smoking. DIF: Cognitive Level: Application REF: 1145-1146 TOP: Nursing Process: Planning

15. The nurse will plan to teach a 27-yr-old woman who smokes two packs of cigarettes daily about the increased risk for a. kidney stones. c. bladder infection. b. bladder cancer. d. interstitial cystitis.

B Cigarette smoking is a risk factor for bladder cancer. The patient's risk for developing interstitial cystitis, urinary tract infection, or kidney stones will not be reduced by quitting smoking. DIF: Cognitive Level: Apply (application) REF: 1054 TOP: Nursing Process: Planning

2. Which item should the nurse offer to the patient who is to restart oral intake after being NPO due to nausea and vomiting? a. Glass of orange juice c. Cup of coffee with cream b. Dish of lemon gelatin d. Bowl of hot chicken broth

B Clear cool liquids are usually the first foods started after a patient has been nauseated. Acidic foods such as orange juice, very hot foods, and coffee are poorly tolerated when patients have been nauseated. DIF: Cognitive Level: Apply (application) REF: 894 TOP: Nursing Process: Implementation

2. A patient who has been NPO during treatment for nausea and vomiting caused by gastric irritation is to start oral intake. Which of these should the nurse offer to the patient? a. A glass of orange juice b. A dish of lemon gelatin c. A cup of coffee with cream d. A bowl of hot chicken broth

B Clear liquids are usually the first foods started after a patient has been nauseated. Acidic foods such as orange juice, very hot foods, and coffee are poorly tolerated when patients have been nauseated. DIF: Cognitive Level: Application REF: 965 | 966-967 TOP: Nursing Process: Implementation

6. A client is scheduled for a traditional esophagogastrostomy. All preoperative teaching has been completed and the client and family show good understanding. What action by the nurse is best? a. Arrange an intensive care unit tour. b. Assess the client's psychosocial status. c. Document the teaching and response. d. Have the client begin nutritional supplements.

B Clients facing this long, difficult procedure are often anxious and fearful. The nurse should now assess the client's psychosocial status and provide the care and teaching required based on this assessment. An intensive care unit tour may help decrease stress but is too limited in scope to be the best response. Documentation should be thorough, but the nurse needs to do more than document. The client should begin nutritional supplements prior to the operation, but again this response is too limited in scope. DIF: Applying/Application REF: 1120 KEY: Gastrointestinal disorders| psychosocial response| nursing assessment| coping

1. After teaching a client with irritable bowel syndrome (IBS), a nurse assesses the client's understanding. Which menu selection indicates that the client correctly understands the dietary teaching? a. Ham sandwich on white bread, cup of applesauce, glass of diet cola b. Broiled chicken with brown rice, steamed broccoli, glass of apple juice c. Grilled cheese sandwich, small banana, cup of hot tea with lemon d. Baked tilapia, fresh green beans, cup of coffee with low-fat milk

B Clients with IBS are advised to eat a high-fiber diet (30 to 40 g/day), with 8 to 10 cups of liquid daily. Chicken with brown rice, broccoli, and apple juice has the highest fiber content. They should avoid alcohol, caffeine, and other gastric irritants. DIF: Applying/Application REF: 1145 KEY: Irritable bowel| nutritional requirements

9. A client had an upper gastrointestinal hemorrhage and now has a nasogastric (NG) tube. What comfort measure may the nurse delegate to the unlicensed assistive personnel (UAP)? a. Lavaging the tube with ice water b. Performing frequent oral care c. Re-positioning the tube every 4 hours d. Taking and recording vital signs

B Clients with NG tubes need frequent oral care both for comfort and to prevent infection. Lavaging the tube is done by the nurse. Re-positioning the tube, if needed, is also done by the nurse. The UAP can take vital signs, but this is not a comfort measure. DIF: Applying/Application REF: 1136 KEY: Gastrointestinal disorders| nasogastric tubes| comfort measures| delegation| unlicensed assistive personnel (UAP)

24. A patient who has had a transurethral resection with fulguration for bladder cancer 3 days previously calls the nurse at the urology clinic. Which information given by the patient is most important to report to the health care provider? a. The patient is using opioids for pain. b. The patient has noticed clots in the urine. c. The patient is very anxious about the cancer. d. The patient is voiding every 4 hours at night.

B Clots in the urine are not expected and require further follow-up. Voiding every 4 hours, use of opioids for pain, and anxiety are typical after this procedure. DIF: Cognitive Level: Application REF: 1145-1146 TOP: Nursing Process: Assessment

16. The nurse determines that teaching regarding cobalamin injections has been effective when the patient with chronic atrophic gastritis states a. "The cobalamin injections will prevent gastric inflammation." b. "The cobalamin injections will prevent me from becoming anemic." c. "These injections will increase the hydrochloric acid in my stomach." d. "These injections will decrease my risk for developing stomach cancer."

B Cobalamin supplementation prevents the development of pernicious anemia. Chronic gastritis may cause achlorhydria, but cobalamin does not correct this. The loss of intrinsic factor secretion with chronic gastritis is permanent, and the patient will need lifelong supplementation with cobalamin. The incidence of stomach cancer is higher in patients with chronic gastritis, but cobalamin does not reduce the risk for stomach cancer. DIF: Cognitive Level: Apply (application) REF: 910 TOP: Nursing Process: Evaluation

17. Which information will the nurse prioritize in planning preoperative teaching for a patient undergoing a Roux-en- gastric bypass? a. Educating the patient about the nasogastric (NG) tube b. Instructing the patient on coughing and breathing techniques c. Discussing necessary postoperative modifications in lifestyle d. Demonstrating passive range-of-motion exercises for the legs

B Coughing and deep breathing can prevent major postoperative complications such as carbon monoxide retention and hypoxemia. Information about passive range of motion, the NG tube, and postoperative modifications in lifestyle will also be discussed, but avoidance of respiratory complications is the priority goal after surgery. DIF: Cognitive Level: Analyze (analysis) REF: 888 OBJ: Special Questions: Prioritization TOP: Nursing Process: Planning

38. Which of these assessment findings in a patient with a hiatal hernia who returned from a laparoscopic Nissen fundoplication 4 hours ago is most important for the nurse to address immediately? a. The patient is experiencing intermittent waves of nausea. b. The patient has absent breath sounds throughout the left lung. c. The patient has decreased bowel sounds in all four quadrants. d. The patient complains of 6/10 (0 to 10 scale) abdominal pain.

B Decreased breath sounds on one side may indicate a pneumothorax, which requires rapid diagnosis and treatment. The abdominal pain and nausea also should be addressed but they are not as high priority as the patient's respiratory status. The patient's decreased bowel sounds are expected after surgery and require ongoing monitoring but no other action. DIF: Cognitive Level: Application REF: 974-975 OBJ: Special Questions: Prioritization TOP: Nursing Process: Assessment

38. A patient returned from a laparoscopic Nissen fundoplication for hiatal hernia 4 hours ago. Which assessment finding is most important for the nurse to address immediately? a. The patient is experiencing intermittent waves of nausea. b. The patient has no breath sounds in the left anterior chest. c. The patient complains of 7/10 (0 to 10 scale) abdominal pain. d. The patient has hypoactive bowel sounds in all four quadrants.

B Decreased breath sounds on one side may indicate a pneumothorax, which requires rapid diagnosis and treatment. The nausea and abdominal pain should also be addressed, but they are not as high priority as the patient's respiratory status. The patient's decreased bowel sounds are expected after surgery and require ongoing monitoring but no other action. DIF: Cognitive Level: Analyze (analysis) REF: 904 OBJ: Special Questions: Prioritization TOP: Nursing Process: Assessment

18. A patient hospitalized with chronic heart failure eats only about 50% of each meal and reports "feeling too tired to eat." Which action should the nurse take first? a. Teach the patient about the importance of good nutrition. b. Serve multiple small feedings of high-calorie, high-protein foods. c. Consult with the health care provider about parenteral nutrition (PN). d. Obtain an order for enteral feedings of liquid nutritional supplements.

B Eating small amounts of food frequently throughout the day is less fatiguing and will improve the patient's ability to take in more nutrients. Teaching the patient may be appropriate, but will not address the patient's inability to eat more because of fatigue. Tube feedings or PN may be needed if the patient is unable to take in enough nutrients orally, but increasing the oral intake should be attempted first. DIF: Cognitive Level: Analyze (analysis) REF: 862 OBJ: Special Questions: Prioritization TOP: Nursing Process: Planning

18. Which action should the nurse take first in order to improve calorie and protein intake for a patient who eats only about 50% of each meal because of "feeling too tired to eat much." a. Teach the patient about the importance of good nutrition. b. Serve multiple small feedings of high-calorie, high-protein foods. c. Obtain an order for enteral feedings of liquid nutritional supplements. d. Consult with the health care provider about providing parenteral nutrition (PN).

B Eating small amounts of food frequently throughout the day is less fatiguing and will improve the patient's ability to take in more nutrients. Teaching the patient may be appropriate, but will not address the patient's inability to eat more because of fatigue. Tube feedings or PN may be needed if the patient is unable to take in enough nutrients orally, but increasing the oral intake should be attempted first. DIF: Cognitive Level: Application REF: 929 OBJ: Special Questions: Prioritization TOP: Nursing Process: Planning

29. A 58-yr-old male patient who is diagnosed with nephrotic syndrome has ascites and 4+ leg edema. Which patient problem is present based on these findings? a. Activity intolerance c. Disturbed body image b. Excess fluid volume d. Altered nutrition: less than required

B Edema and ascites are evidence of the excess fluid volume. There are no data provided to support the other problems. DIF: Cognitive Level: Apply (application) REF: 1043 TOP: Nursing Process: Analysis

10. Which assessment will the nurse do to help determine if an obese patient seen in the clinic has metabolic syndrome? a. Take the patient's apical pulse. b. Check the patient's blood pressure. c. Ask the patient about dietary intake. d. Dipstick the patient's urine for protein.

B Elevated blood pressure is one of the characteristics of metabolic syndrome. The other information also may be obtained by the nurse, but it will not assist with the diagnosis of metabolic syndrome. DIF: Cognitive Level: Application REF: 959-960 TOP: Nursing Process: Assessment

8. Which assessment action will help the nurse determine if an obese patient has metabolic syndrome? a. Take the patient's apical pulse. b. Check the patient's blood pressure. c. Ask the patient about dietary intake. d. Dipstick the patient's urine for protein.

B Elevated blood pressure is one of the characteristics of metabolic syndrome. The other information will not assist with the diagnosis of metabolic syndrome. DIF: Cognitive Level: Apply (application) REF: 890 TOP: Nursing Process: Assessment

12. Which information will the nurse include for a patient with newly diagnosed gastroesophageal reflux disease (GERD)? a. "Peppermint tea may reduce your symptoms." b. "Keep the head of your bed elevated on blocks." c. "You should avoid eating between meals to reduce acid secretion." d. "Vigorous physical activities may increase the incidence of reflux."

B Elevating the head of the bed will reduce the incidence of reflux while the patient is sleeping. Peppermint will decrease lower esophageal sphincter (LES) pressure and increase the chance for reflux. Small, frequent meals are recommended to avoid abdominal distention. There is no need to make changes in physical activities because of GERD. DIF: Cognitive Level: Apply (application) REF: 902 TOP: Nursing Process: Implementation

13. Which nursing action should be included in the postoperative plan of care for a patient after a laparoscopic esophagectomy? a. Reposition the NG tube if drainage stops. b. Elevate the head of the bed to at least 30 degrees. c. Start oral fluids when the patient has active bowel sounds. d. Notify the doctor for any bloody nasogastric (NG) drainage.

B Elevation of the head of the bed decreases the risk for reflux and aspiration of gastric secretions. The NG tube should not be repositioned without consulting with the health care provider. Bloody NG drainage is expected for the first 8 to 12 hours. A swallowing study is needed before oral fluids are started. DIF: Cognitive Level: Apply (application) REF: 907 TOP: Nursing Process: Planning

14. After a total proctocolectomy and permanent ileostomy, the patient tells the nurse, "I cannot manage all these changes. I don't want to look at the stoma." What is the best action by the nurse? a. Reassure the patient that ileostomy care will become easier. b. Ask the patient about the concerns with stoma management. c. Postpone any teaching until the patient adjusts to the ileostomy. d. Develop a detailed written list of ostomy care tasks for the patient.

B Encouraging the patient to share concerns assists in helping the patient adjust to the body changes. Acknowledgment of the patient's feelings and concerns is important rather than offering false reassurance. Because the patient indicates that the feelings about the ostomy are the reason for the difficulty with the many changes, development of a detailed ostomy care plan will not improve the patient's ability to manage the ostomy. Although detailed ostomy teaching may be postponed, the nurse should offer teaching about some aspects of living with an ostomy. DIF: Cognitive Level: Analyze (analysis) REF: 959 TOP: Nursing Process: Implementation

14. A patient who has had a total proctocolectomy and permanent ileostomy tells the nurse, "I cannot bear to even look at the stoma. I do not think I can manage all these changes." Which is the best action by the nurse? a. Develop a detailed written plan for ostomy care for the patient. b. Ask the patient more about the concerns with stoma management. c. Reassure the patient that care for the ileostomy will become easier. d. Postpone any patient teaching until the patient adjusts to the ileostomy.

B Encouraging the patient to share concerns assists in helping the patient adjust to the body changes. Acknowledgment of the patient's feelings and concerns is important rather than offering false reassurance. Because the patient indicates that the feelings about the ostomy are the reason for the difficulty with the many changes, development of a detailed ostomy care plan will not improve the patient's ability to manage the ostomy. Although detailed ostomy teaching may be postponed, the nurse should offer teaching about some aspects of living with an ostomy. DIF: Cognitive Level: Application REF: 1042 | 1044-1046 TOP: Nursing Process: Implementation

5. A client with an esophageal tumor is having extreme difficulty swallowing. For what procedure does the nurse prepare this client? a. Enteral tube feeding b. Esophageal dilation c. Nissen fundoplication d. Photodynamic therapy

B Esophageal dilation can provide immediate relief of esophageal strictures that impair swallowing. Enteral tube feeding is a method of providing nutrition when dysphagia is severe, but esophageal dilation would be attempted before this measure is taken. Nissen fundoplication is performed for severe gastroesophageal reflux disease. Photodynamic therapy is performed for esophageal cancer. DIF: Understanding/Comprehension REF: 1120 KEY: Gastrointestinal disorders| patient education

16. Several nurses have just helped a morbidly obese client get out of bed. One nurse accesses the client's record because "I just have to know how much she weighs!" What action by the client's nurse is most appropriate? a. Make an anonymous report to the charge nurse. b. State "That is a violation of client confidentiality." c. Tell the nurse "Don't look; I'll tell you her weight." d. Walk away and ignore the other nurse's behavior.

B Ethical practice requires the nurse to speak up and tell the other nurse that he or she is violating client confidentiality rules. The other responses do not address this concern. DIF: Applying/Application REF: 1248 KEY: Ethics| confidentiality

22. A nurse plans care for clients with urinary incontinence. Which client is correctly paired with the appropriate intervention? a. A 29-year-old client after a difficult vaginal delivery - Habit training b. A 58-year-old postmenopausal client who is not taking estrogen therapy - Electrical stimulation c. A 64-year-old female with Alzheimer's-type senile dementia - Bladder training d. A 77-year-old female who has difficulty ambulating - Exercise therapy

B Exercise therapy and electrical stimulation are used for clients with stress incontinence related to childbirth or low levels of estrogen after menopause. Exercise therapy increases pelvic wall strength; it does not improve ambulation. Physical therapy and a bedside commode would be appropriate interventions for the client who has difficulty ambulating. Habit training is the type of bladder training that will be most effective with cognitively impaired clients. Bladder training can be used only with a client who is alert, aware, and able to resist the urge to urinate. DIF: Applying/Application REF: 1373 KEY: Urinary incontinence

22. The family member of a patient who has suffered massive abdominal trauma in an automobile accident asks the nurse why the patient is receiving famotidine (Pepcid). The nurse will explain that the medication will a. prevent aspiration of gastric contents. b. inhibit the development of stress ulcers. c. lower the chance for H. pylori infection. d. decrease the risk for nausea and vomiting.

B Famotidine is administered to prevent the development of physiologic stress ulcers, which are associated with a major physiologic insult such as massive trauma. Famotidine does not decrease nausea or vomiting, prevent aspiration, or prevent H. pylori infection. DIF: Cognitive Level: Application REF: 981 TOP: Nursing Process: Implementation

22. A young adult patient is hospitalized with massive abdominal trauma from a motor vehicle crash. The patient asks the nurse about the purpose of receiving famotidine (Pepcid). The nurse will explain that the medication will a. decrease nausea and vomiting. b. inhibit development of stress ulcers. c. lower the risk for H. pylori infection. d. prevent aspiration of gastric contents.

B Famotidine is administered to prevent the development of physiologic stress ulcers, which are associated with a major physiologic insult such as massive trauma. Famotidine does not decrease nausea or vomiting, prevent aspiration, or prevent Helicobacter pylori infection. DIF: Cognitive Level: Apply (application) REF: 903 TOP: Nursing Process: Implementation

32. A critically ill patient with sepsis is frequently incontinent of watery stools. What action by the nurse will prevent complications associated with ongoing incontinence? a. Apply incontinence briefs. b. Use a fecal management system c. Insert a rectal tube with a drainage bag. d. Assist the patient to a commode frequently.

B Fecal management systems are designed to contain loose stools and can be in place for as long as 4 weeks without causing damage to the rectum or anal sphincters. Although incontinence briefs may be helpful, unless they are changed frequently, they are likely to increase the risk for skin breakdown. Rectal tubes are avoided because of possible damage to the anal sphincter and ulceration of the rectal mucosa. A critically ill patient will not be able to tolerate getting up frequently to use the commode or bathroom. DIF: Cognitive Level: Apply (application) REF: 934 TOP: Nursing Process: Implementation

16. A young woman who has Crohn's disease develops a fever and symptoms of a urinary tract infection (UTI) with tan, fecal-smelling urine. What information will the nurse add to a general teaching plan about UTIs in order to individualize the teaching for this patient? a. Bacteria in the perianal area can enter the urethra. b. Fistulas can form between the bowel and bladder. c. Drink adequate fluids to maintain normal hydration. d. Empty the bladder before and after sexual intercourse.

B Fistulas between the bowel and bladder occur in Crohn's disease and can lead to UTI. Teaching for UTI prevention in general includes good hygiene, adequate fluid intake, and voiding before and after intercourse. DIF: Cognitive Level: Apply (application) REF: 963 TOP: Nursing Process: Implementation

16. A patient with Crohn's disease develops a fever and symptoms of a urinary tract infection (UTI) with tan, fecal-smelling urine. The nurse will teach the patient a. to clean the perianal area carefully after any stools. b. about fistula formation between the bowel and bladder. c. to empty the bladder before and after sexual intercourse. d. about the effects of corticosteroid use on immune function.

B Fistulas between the bowel and bladder occur in Crohn's disease and can lead to UTI. There is no information indicating that the patient's risk for UTI is caused by poor cleaning or not voiding before and after intercourse. Steroid use may increase the risk for infection, but the characteristics of the patient's urine indicate that a fistula has occurred. DIF: Cognitive Level: Application REF: 1023-1024 TOP: Nursing Process: Implementation

8. A patient with nephrotic syndrome develops flank pain. The nurse will anticipate teaching the patient about treatment with a. antibiotics. b. anticoagulants. c. corticosteroids. d. antihypertensives.

B Flank pain in a patient with nephrosis suggests a renal vein thrombosis, and anticoagulation is needed. Antibiotics are used to treat a patient with flank pain caused by pyelonephritis. Antihypertensives are used if the patient has high blood pressure. Corticosteroids may be used to treat nephrotic syndrome but will not resolve a thrombosis. DIF: Cognitive Level: Application REF: 1133-1134 TOP: Nursing Process: Planning

39. A patient seen in the clinic for a bladder infection describes the following symptoms. Which information is most important for the nurse to report to the health care provider? a. Urinary urgency c. Intermittent hematuria b. Left-sided flank pain d. Burning with urination

B Flank pain indicates that the patient may have developed pyelonephritis as a complication of the bladder infection. The other clinical manifestations are consistent with a lower urinary tract infection. DIF: Cognitive Level: Analyze (analysis) REF: 1035 OBJ: Special Questions: Prioritization TOP: Nursing Process: Assessment

4. A client is receiving bolus feedings through a Dobhoff tube. What action by the nurse is most important? a. Auscultate lung sounds after each feeding. b. Check tube placement before each feeding. c. Check tube placement every 8 hours. d. Weigh the client daily on the same scale.

B For bolus feedings, the nurse checks placement of the tube per institutional policy prior to each feeding, which is more often than every 8 hours during the day. Auscultating lung sounds is also important, but this will indicate a complication that has already occurred. Weighing the client is important to determine if nutritional goals are being met. DIF: Applying/Application REF: 1243 KEY: Nutritional disorders| tube feedings| equipment safety

1. A client has been taught about alginic acid and sodium bicarbonate (Gaviscon). What statement by the client indicates that teaching has been effective? a. "I can only take this medicine at night.." b. "I should take this on a full stomach." c. "This drug decreases stomach acid." d. "This should be taken 1 hour before meals."

B Gaviscon should be taken with food in the stomach. It can be taken with meals at any time. Its mechanism of action is not to decrease stomach acid. DIF: Evaluating/Synthesis REF: 1113 KEY: Gastrointestinal disorders| antacids| patient education

7. After delegating care to an unlicensed assistive personnel (UAP) for a client who is prescribed habit training to manage incontinence, a nurse evaluates the UAP's understanding. Which action indicates the UAP needs additional teaching? a. Toileting the client after breakfast b. Changing the client's incontinence brief when wet c. Encouraging the client to drink fluids d. Recording the client's incontinence episodes

B Habit training is undermined by the use of absorbent incontinence briefs or pads. The nurse should re-educate the UAP on the technique of habit training. The UAP should continue to toilet the client after meals, encourage the client to drink fluids, and record incontinent episodes. DIF: Applying/Application REF: 1381 KEY: Urinary incontinence| delegation| supervision| unlicensed assistive personnel (UAP)

23. A patient who has ulcerative colitis has a proctocolectomy and ileostomy. Which information will the nurse include in patient teaching? a. Restrict fluid intake to prevent constant liquid drainage from the stoma. b. Use care when eating high-fiber foods to avoid obstruction of the ileum. c. Irrigate the ileostomy daily to avoid having to wear a drainage appliance. d. Change the pouch every day to prevent leakage of contents onto the skin.

B High-fiber foods are introduced gradually and should be well chewed to avoid obstruction of the ileostomy. Patients with ileostomies lose the absorption of water in the colon and need to take in increased amounts of fluid. The pouch should be drained frequently but is changed every 5 to 7 days. The drainage from an ileostomy is liquid and continuous, so control by irrigation is not possible. DIF: Cognitive Level: Application REF: 1042 | 1044-1045 TOP: Nursing Process: Implementation

23. Which information will the nurse include in teaching a patient who had a proctocolectomy and ileostomy for ulcerative colitis? a. Restrict fluid intake to prevent constant liquid drainage from the stoma. b. Use care when eating high-fiber foods to avoid obstruction of the ileum. c. Irrigate the ileostomy daily to avoid having to wear a drainage appliance. d. Change the pouch every day to prevent leakage of contents onto the skin.

B High-fiber foods are introduced gradually and should be well chewed to avoid obstruction of the ileostomy. Patients with ileostomies lose the absorption of water in the colon and need to take in increased amounts of fluid. The pouch should be drained frequently but is changed every 5 to 7 days. The drainage from an ileostomy is liquid and continuous, so control by irrigation is not possible. DIF: Cognitive Level: Apply (application) REF: 962 TOP: Nursing Process: Implementation

5. After teaching a patient with interstitial cystitis about management of the condition, the nurse determines that further instruction is needed when the patient says, a. "I will have to stop having coffee and orange juice for breakfast." b. "I should start taking a high potency multiple vitamin every morning." c. "I will buy some calcium glycerophosphate (Prelief) at the pharmacy." d. "I should call the doctor about increased bladder pain or odorous urine."

B High-potency multiple vitamins may irritate the bladder and increase symptoms. The other patient statements indicate good understanding of the teaching. DIF: Cognitive Level: Application REF: 1130-1131 TOP: Nursing Process: Evaluation

12. When planning teaching for a patient with benign nephrosclerosis, the nurse should include instructions regarding a. preventing bleeding with anticoagulants. b. monitoring and recording blood pressure. c. obtaining and documenting daily weights. d. measuring daily intake and output volumes.

B Hypertension is the major manifestation of nephrosclerosis. Measurements of intake and output and daily weights are not necessary unless the patient develops renal insufficiency. Anticoagulants are not used to treat nephrosclerosis. DIF: Cognitive Level: Apply (application) REF: 1050 TOP: Nursing Process: Planning

15. Which action for a patient receiving tube feedings through a percutaneous endoscopic gastrostomy (PEG) may be delegated to a licensed practical/vocational nurse (LPN/LVN)? a. Assessing the patient's nutritional status weekly b. Providing skin care to the area around the tube site c. Teaching the patient how to administer tube feedings d. Determining the need for adding water to the feedings

B LPN/LVN education and scope of practice include actions such as dressing changes and wound care. Patient teaching and complex assessments (such as patient nutrition and hydration status) require registered nurse (RN)-level education and scope of practice. DIF: Cognitive Level: Apply (application) REF: 867 OBJ: Special Questions: Delegation TOP: Nursing Process: Planning

53. Which information will the nurse teach a patient with lactose intolerance? a. Ice cream is relatively low in lactose. b. Live-culture yogurt is usually tolerated. c. Heating milk will break down the lactose. d. Nonfat milk is tolerated better than whole milk.

B Lactose-intolerant individuals can usually eat yogurt without experiencing discomfort. Ice cream, nonfat milk, and milk that has been heated are all high in lactose. DIF: Cognitive Level: Understand (comprehension) REF: 949 TOP: Nursing Process: Planning

3. A nurse cares for a postmenopausal client who has had two episodes of bacterial urethritis in the last 6 months. The client asks, "I never have urinary tract infections. Why is this happening now?" How should the nurse respond? a. "Your immune system becomes less effective as you age." b. "Low estrogen levels can make the tissue more susceptible to infection." c. "You should be more careful with your personal hygiene in this area." d. "It is likely that you have an untreated sexually transmitted disease."

B Low estrogen levels decrease moisture and secretions in the perineal area and cause other tissue changes, predisposing it to the development of infection. Urethritis is most common in postmenopausal women for this reason. Although immune function does decrease with aging and sexually transmitted diseases are a known cause of urethritis, the most likely reason in this client is low estrogen levels. Personal hygiene usually does not contribute to this disease process. DIF: Applying/Application REF: 1367 KEY: Cystitis| patient education

17. A nurse cares for a middle-aged male client who has irritable bowel syndrome (IBS). The client states, "I have changed my diet and take bulk-forming laxatives, but my symptoms have not gotten better. I heard about a drug called Amitiza. Do you think it might help?" How should the nurse respond? a. "This drug is still in the research phase and is not available for public use yet." b.. "Unfortunately, lubiprostone is approved only for use in women." c. "Lubiprostone works well. I will recommend this prescription to your provider." d. "This drug should not be used with bulk-forming laxatives."

B Lubiprostone (Amitiza) is a new drug for IBS with constipation that works by simulating receptors in the intestines to increase fluid and promote bowel transit time. Lubiprostone is currently approved only for use in women. Trials with increased numbers of male participants are needed prior to Food and Drug Administration approval for men. DIF: Applying/Application REF: 1146 KEY: Irritable bowel| medications

13. A morbidly obese client is admitted to a community hospital that does not typically care for bariatric-sized clients.. What action by the nurse is most appropriate? a.. Assess the client's readiness to make lifestyle changes. b. Ensure adequate staff when moving the client. c. Leave siderails down to prevent pressure ulcers. d. Reinforce the need to be sensitive to the client.

B Many hospitals that see bariatric-sized clients have appropriate equipment for this population. A hospital that does not typically see these clients is less likely to have appropriate equipment, putting staff and client safety at risk. The nurse ensures enough staffing is available to help with all aspects of mobility. It may or may not be appropriate to assess the client's willingness to make lifestyle changes. Leaving the siderails down may present a safety hazard. The staff should be sensitive to this client's situation, but safety takes priority. DIF: Applying/Application REF: 1250 KEY: Nutritional disorders| obesity| patient safety| staff safety

19. A client is awaiting bariatric surgery in the morning. What action by the nurse is most important? a. Answering questions the client has about surgery b. Beginning venous thromboembolism prophylaxis c. Informing the client that he or she will be out of bed tomorrow d. Teaching the client about needed dietary changes

B Morbidly obese clients are at high risk of venous thromboembolism and should be started on a regimen to prevent this from occurring as a priority. Answering questions about the surgery is done by the surgeon. Teaching is important, but safety comes first. DIF: Applying/Application REF: 1251 KEY: Nutritional disorders| obesity| venous thromboembolism

1. Which statement by the nurse is most likely to help a 22-yr-old patient with extreme obesity in losing weight on a 1000-calorie diet? a. "It will be necessary to change lifestyle habits permanently to maintain weight loss." b. "You are likely to notice changes in how you feel after a few weeks of diet and exercise." c. "You will decrease your risk for future health problems such as diabetes by losing weight now." d. "Most of the weight that you lose during the first weeks of dieting is water weight rather than fat."

B Motivation is a key factor in successful weight loss and a short-term outcome provides a higher motivation. A 22-yr-old patient is unlikely to be motivated by future health problems. Telling a patient that the initial weight loss is water will be discouraging, although this may be correct. Changing lifestyle habits is necessary, but this process occurs over time, and discussing this is not likely to motivate the patient. DIF: Cognitive Level: Analyze (analysis) REF: 881 TOP: Nursing Process: Implementation

42. The RN and nursing assistive personnel (NAP) are caring for a patient with a paralytic ileus. Which of these nursing activities is appropriate for the nurse to delegate to NAP? a. Auscultation for bowel sounds b. Applying petroleum jelly to the lips c. Irrigation of the nasogastric (NG) tube with saline d. Assessment of the nose for irritation

B NAP education and scope of practice include patient hygiene such as oral care. The other actions require education and scope of practice appropriate to the RN. DIF: Cognitive Level: Application REF: 1033-1034 OBJ: Special Questions: Delegation TOP: Nursing Process: Implementation

10. An emergency room nurse assesses a client after a motor vehicle crash and notes ecchymotic areas across the client's lower abdomen. Which action should the nurse take first? a. Measure the client's abdominal girth. b. Assess for abdominal guarding or rigidity. c. Check the client's hemoglobin and hematocrit. d. Obtain the client's complete health history.

B On noticing the ecchymotic areas, the nurse should check to see if abdominal guarding or rigidity is present, because this could indicate major organ injury. The nurse should then notify the provider. Measuring abdominal girth or obtaining a complete health history is not appropriate at this time. Laboratory test results can be checked after assessment for abdominal guarding or rigidity. DIF: Applying/Application REF: 1162 KEY: Gastrointestinal trauma| hemorrhage

10. To prevent recurrence of uric acid renal calculi, the nurse teaches the patient to avoid eating a. milk and cheese. c. spinach and chocolate. b. sardines and liver. d. legumes and dried fruit.

B Organ meats and fish such as sardines increase purine levels and uric acid. Spinach, chocolate, and tomatoes should be avoided in patients who have oxalate stones. Milk, dairy products, legumes, and dried fruits may increase the incidence of calcium-containing stones. DIF: Cognitive Level: Apply (application) REF: 1046 TOP: Nursing Process: Implementation

51. After change-of-shift report, which patient should the nurse assess first? a. A 40-yr-old male patient with celiac disease who has frequent frothy diarrhea b. A 30-yr-old female patient with a femoral hernia who has abdominal pain and vomiting c. A 30-yr-old male patient with ulcerative colitis who has severe perianal skin breakdown d. A 40-yr-old female patient with a colostomy bag that is pulling away from the adhesive wafer

B Pain and vomiting with a femoral hernia suggest possible strangulation, which will necessitate emergency surgery. The other patients have less urgent problems. DIF: Cognitive Level: Analyze (analysis) REF: 952 OBJ: Special Questions: Multiple Patients | Special Questions: Prioritization TOP: Nursing Process: Assessment

20. After the home health nurse teaches a patient with a neurogenic bladder how to use intermittent catheterization for bladder emptying, which patient statement indicates that the teaching has been effective? a. "I will use a sterile catheter and gloves for each time I self-catheterize." b. "I will clean the catheter carefully before and after each catheterization." c. "I will need to buy seven new catheters weekly and use a new one every day." d. "I will need to take prophylactic antibiotics to prevent any urinary tract infections."

B Patients who are at home can use a clean technique for intermittent self-catheterization and change the catheter every 7 days. There is no need to use a new catheter every day, to use sterile catheters, or to take prophylactic antibiotics. DIF: Cognitive Level: Application REF: 1154 TOP: Nursing Process: Evaluation

49. A new 19-yr-old male patient has familial adenomatous polyposis (FAP). Which action will the nurse in the gastrointestinal clinic include in the plan of care? a. Obtain blood samples for DNA analysis. b. Schedule the patient for yearly colonoscopy. c. Provide preoperative teaching about total colectomy. d. Discuss lifestyle modifications to decrease cancer risk.

B Patients with FAP should have annual colonoscopy starting at age 16 years and usually have total colectomy by age 25 years to avoid developing colorectal cancer. DNA analysis is used to make the diagnosis but is not needed now for this patient. Lifestyle modifications will not decrease cancer risk for this patient. DIF: Cognitive Level: Apply (application) REF: 953 TOP: Nursing Process: Planning

2. A client has a pyloric obstruction and reports sudden muscle weakness. What action by the nurse takes priority? a. Document the findings in the chart. b. Request an electrocardiogram (ECG). c. Facilitate a serum potassium test. d. Place the client on bedrest.

B Pyloric stenosis can lead to hypokalemia, which is manifested by muscle weakness. The nurse first obtains an ECG because potassium imbalances can lead to cardiac dysrhythmias. A potassium level is also warranted, as is placing the client on bedrest for safety. Documentation should be thorough, but none of these actions takes priority over the ECG. DIF: Analyzing/Analysis REF: 1132 KEY: Gastrointestinal disorders| electrolyte imbalances| cardiac system

21. Which information will the nurse include when teaching a patient with peptic ulcer disease about the effect of ranitidine (Zantac)? a. "Ranitidine absorbs the excess gastric acid." b. "Ranitidine decreases gastric acid secretion." c. "Ranitidine constricts the blood vessels near the ulcer." d. "Ranitidine covers the ulcer with a protective material."

B Ranitidine is a histamine-2 (H2) receptor blocker that decreases the secretion of gastric acid. The response beginning, "Ranitidine constricts the blood vessels" describes the effect of vasopressin. The response "Ranitidine absorbs the gastric acid" describes the effect of antacids. The response beginning "Ranitidine covers the ulcer" describes the action of sucralfate (Carafate). DIF: Cognitive Level: Understand (comprehension) REF: 903 TOP: Nursing Process: Implementation

6. A 58-yr-old patient with blunt abdominal trauma from a motor vehicle crash undergoes peritoneal lavage. If the lavage returns brown fecal drainage, which action will the nurse plan to take next? a. Auscultate the bowel sounds. b. Prepare the patient for surgery. c. Check the patient's oral temperature. d. Obtain information about the accident.

B Return of brown drainage and fecal material suggests perforation of the bowel and the need for immediate surgery. Auscultation of bowel sounds, checking the temperature, and obtaining information about the accident are appropriate actions, but the priority is to prepare to send the patient for emergency surgery. DIF: Cognitive Level: Analyze (analysis) REF: 941 OBJ: Special Questions: Prioritization TOP: Nursing Process: Planning

6. A patient who has blunt abdominal trauma after an automobile accident is complaining of severe pain. A peritoneal lavage returns brown drainage with fecal material. Which action will the nurse plan to take next? a. Auscultate the bowel sounds. b. Prepare the patient for surgery. c. Check the patient's oral temperature. d. Obtain information about the accident.

B Return of brown drainage and fecal material suggests perforation of the bowel and the need for immediate surgery. Auscultation of bowel sounds, checking the temperature, and obtaining information about the accident are appropriate actions, but the priority is to prepare to send the patient for emergency surgery. DIF: Cognitive Level: Application REF: 1018-1019 TOP: Nursing Process: Planning

10. Which nursing action will the nurse include in the plan of care when admitting a patient with an exacerbation of inflammatory bowel disease (IBD)? a. Restrict oral fluid intake. b. Monitor stools for blood. c. Increase dietary fiber intake. d. Ambulate four times daily.

B Since anemia or hemorrhage may occur with IBD, stools should be assessed for the presence of blood. The other actions would not be appropriate for the patient with IBD. Because dietary fiber may increase gastrointestinal (GI) motility and exacerbate the diarrhea, severe fatigue is common with IBD exacerbations, and dehydration may occur. DIF: Cognitive Level: Application REF: 1024-1025 | 1028-1029 TOP: Nursing Process: Planning

35. Which of these prescribed interventions will the nurse implement first when caring for a patient who has just been diagnosed with peritonitis caused by a ruptured diverticulum? a. Administer morphine sulfate 4 mg IV. b. Infuse metronidazole (Flagyl) 500 mg IV. c. Send the patient for a computerized tomography scan. d. Insert a nasogastric (NG) tube and connect it to intermittent low suction.

B Since peritonitis can be fatal if treatment is delayed, the initial action should be to start antibiotic therapy (after any ordered cultures are obtained). The other actions can be done after antibiotic therapy is initiated. DIF: Cognitive Level: Application REF: 1021-1022 OBJ: Special Questions: Prioritization TOP: Nursing Process: Implementation

1. A patient returns to the clinic with recurrent dysuria after being treated with trimethoprim and sulfamethoxazole (Bactrim) for 3 days. Which action will the nurse plan to take? a. Remind the patient about the need to drink 1000 mL of fluids daily. b. Obtain a midstream urine specimen for culture and sensitivity testing. c. Teach the patient to take the prescribed Bactrim for at least 3 more days. d. Suggest that the patient use acetaminophen (Tylenol) to treat the symptoms.

B Since uncomplicated urinary tract infections (UTIs) are usually successfully treated with 3 days of antibiotic therapy, this patient will need a urine culture and sensitivity to determine appropriate antibiotic therapy. Tylenol would not be as effective as other over-the-counter (OTC) medications such as phenazopyridine (Pyridium) in treating dysuria. The fluid intake should be increased to at least 1800 mL/day. Since the UTI has persisted after treatment with Bactrim, the patient is likely to need a different antibiotic. DIF: Cognitive Level: Application REF: 1123-1125 TOP: Nursing Process: Planning

34. Following an open loop resection and fulguration of the bladder, a patient is unable to void. Which nursing action should be implemented first? a. Insert a straight catheter and drain the bladder. b. Assist the patient to take a 15-minute sitz bath. c. Encourage the patient to drink several glasses of water. d. Teach the patient how to do isometric perineal exercises.

B Sitz baths will relax the perineal muscles and promote voiding. Although the patient should be encouraged to drink fluids and Kegel exercises are helpful in the prevention of incontinence, these activities would not be helpful for a patient experiencing retention. Catheter insertion increases the risk for urinary tract infection (UTI) and should be avoided when possible DIF: Cognitive Level: Application REF: 1146 OBJ: Special Questions: Prioritization TOP: Nursing Process: Implementation

32. A 26-yr-old patient with a family history of stomach cancer asks the nurse about ways to decrease the risk for developing stomach cancer. The nurse will teach the patient to avoid a. emotionally stressful situations. b. smoked foods such as ham and bacon. c. foods that cause distention or bloating. d. chronic use of H2 blocking medications.

B Smoked foods such as bacon, ham, and smoked sausage increase the risk for stomach cancer. Stressful situations, abdominal distention, and use of H2 blockers are not associated with an increased incidence of stomach cancer. DIF: Cognitive Level: Apply (application) REF: 919 TOP: Nursing Process: Implementation

20. An emergency department nurse assesses a client with a history of urinary incontinence who presents with extreme dry mouth, constipation, and an inability to void. Which question should the nurse ask first? a. "Are you drinking plenty of water?" b. "What medications are you taking?" c. "Have you tried laxatives or enemas?" d. "Has this type of thing ever happened before?"

B Some types of incontinence are treated with anticholinergic medications such as propantheline (Pro-Banthine). Anticholinergic side effects include dry mouth, constipation, and urinary retention. The nurse needs to assess the client's medication list to determine whether the client is taking an anticholinergic medication. If he or she is taking anticholinergics, the nurse should further assess the client's manifestations to determine if they are related to a simple side effect or an overdose. The other questions are not as helpful to understanding the current situation. DIF: Applying/Application REF: 1378 KEY: Urinary incontinence| medication safety

11. Which patient statement indicates that the nurse's teaching about sulfasalazine (Azulfidine) for ulcerative colitis has been effective? a. "The medication will be tapered if I need surgery." b. "I will need to use a sunscreen when I am outdoors." c. "I will need to avoid contact with people who are sick." d. "The medication prevents the infections that cause diarrhea."

B Sulfasalazine may cause photosensitivity in some patients. It is not used to treat infections. Sulfasalazine does not reduce immune function. Unlike corticosteroids, tapering of sulfasalazine is not needed. DIF: Cognitive Level: Apply (application) REF: 947 TOP: Nursing Process: Evaluation

22. A patient who has bladder cancer had a cystectomy with creation of an Indiana pouch. Which topic will be included in patient teaching? a. Application of ostomy appliances b. Catheterization technique and schedule c. Analgesic use before emptying the pouch d. Use of barrier products for skin protection

B The Indiana pouch enables the patient to self-catheterize every 4 to 6 hours. There is no need for an ostomy device or barrier products. Catheterization of the pouch is not painful. DIF: Cognitive Level: Application REF: 1155-1156 TOP: Nursing Process: Implementation

30. Which information about dietary management should the nurse include when teaching a patient with peptic ulcer disease (PUD)? a. "You will need to remain on a bland diet." b. "Avoid foods that cause pain after you eat them." c. "High-protein foods are least likely to cause you pain." d. "You should avoid eating any raw fruits and vegetables."

B The best information is that each individual should choose foods that are not associated with postprandial discomfort. Raw fruits and vegetables may irritate the gastric mucosa, but chewing well seems to decrease this problem and some patients may tolerate these foods well. High-protein foods help neutralize acid, but they also stimulate hydrochloric (HCl) acid secretion and may increase discomfort for some patients. Bland diets may be recommended during an acute exacerbation of PUD, but there is little scientific evidence to support their use. DIF: Cognitive Level: Apply (application) REF: 918 TOP: Nursing Process: Implementation

21. A nurse teaches a client who is starting urinary bladder training. Which statement should the nurse include in this client's teaching? a. "Use the toilet when you first feel the urge, rather than at specific intervals." b. "Try to consciously hold your urine until the scheduled toileting time." c. "Initially try to use the toilet at least every half hour for the first 24 hours." d. "The toileting interval can be increased once you have been continent for a week."

B The client should try to hold the urine consciously until the next scheduled toileting time. Toileting should occur at specific intervals during the training. The toileting interval should be no less than every hour. The interval can be increased once the client becomes comfortable with the interval. DIF: Understanding/Comprehension REF: 1380 KEY: Urinary incontinence| patient education

17. A patient with peptic ulcer disease associated with the presence of Helicobacter pylori is treated with triple drug therapy. The nurse will plan to teach the patient about a. sucralfate (Carafate), nystatin (Mycostatin), and bismuth (Pepto-Bismol). b. amoxicillin (Amoxil), clarithromycin (Biaxin), and omeprazole (Prilosec). c. famotidine (Pepcid), magnesium hydroxide (Mylanta), and pantoprazole (Protonix). d. metoclopramide (Reglan), bethanechol (Urecholine), and promethazine (Phenergan).

B The drugs used in triple drug therapy include a proton pump inhibitor such as omeprazole and the antibiotics amoxicillin and clarithromycin. The other combinations listed are not included in the protocol for H. pylori infection. DIF: Cognitive Level: Comprehension REF: 986 TOP: Nursing Process: Planning

15. When preparing to teach an 82-year-old Hispanic patient who lives with an adult daughter about ways to improve nutrition, which action should the nurse take first? a. Ask the daughter about the patient's food preferences. b. Determine who shops for groceries and prepares the meals. c. Question the patient about how many meals per day are eaten. d. Assure the patient that culturally appropriate foods will be included.

B The family member who shops for groceries and cooks will be in control of the patient's diet, so the nurse will need to ensure that this family member is involved in any teaching or discussion about the patient's nutritional needs. The other information also will be assessed and used but will not be useful in meeting the patient's nutritional needs unless nutritionally appropriate foods are purchased and prepared. DIF: Cognitive Level: Application REF: 923-924 OBJ: Special Questions: Prioritization TOP: Nursing Process: Planning

16. Which action should the nurse take first when preparing to teach a frail 79-yr-old Hispanic man who lives with an adult daughter about ways to improve nutrition? a. Ask the daughter about the patient's food preferences. b. Determine who shops for groceries and prepares the meals. c. Question the patient about how many meals per day are eaten. d. Assure the patient that culturally preferred foods will be included.

B The family member who shops for groceries and cooks will be in control of the patient's diet, so the nurse will need to ensure that this family member is involved in any teaching or discussion about the patient's nutritional needs. The other information will also be assessed and used but will not be useful in meeting the patient's nutritional needs unless nutritionally appropriate foods are purchased and prepared. DIF: Cognitive Level: Analyze (analysis) REF: 864 OBJ: Special Questions: Prioritization TOP: Nursing Process: Planning

12. A nurse cares for a client who is recovering from a hemorrhoidectomy. The client states, "I need to have a bowel movement." Which action should the nurse take? a. Obtain a bedside commode for the client to use. b. Stay with the client while providing privacy. c. Make sure the call light is in reach to signal completion. d. Gather supplies to collect a stool sample for the laboratory.

B The first bowel movement after hemorrhoidectomy can be painful enough to induce syncope. The nurse should stay with the client. The nurse should instruct clients who are discharged the same day to have someone nearby when they have their first postoperative bowel movement. Making sure the call light is within reach is an important nursing action too, but it does not take priority over client safety. Obtaining a bedside commode and taking a stool sample are not needed in this situation. DIF: Applying/Application REF: 1165 KEY: Postoperative care| syncope

6. A young adult with extensive facial injuries from a motor vehicle crash is receiving tube feedings through a percutaneous endoscopic gastrostomy (PEG). Which action will the nurse include in the plan of care? a. Keep the patient positioned on the left side. b. Check the gastric residual volume every 4 to 6 hours. c. Avoid giving bolus tube feedings through the PEG tube. d. Obtain a daily abdominal radiographs to verify tube placement.

B The gastric residual volume is assessed every 4 to 6 hours to decrease the risk for aspiration. The patient does not need to be positioned on the left side. Bolus feedings can be administered through a PEG tube. An x-ray is obtained immediately after placement of the PEG tube to check position, but daily are not needed. DIF: Cognitive Level: Apply (application) REF: 866 TOP: Nursing Process: Planning

35. A 25-yr-old male patient calls the clinic complaining of diarrhea for 24 hours. Which action should the nurse take first? a. Inform the patient that laboratory testing of blood and stools will be necessary. b. Ask the patient to describe the character of the stools and any associated symptoms. c. Suggest that the patient drink clear liquid fluids with electrolytes, such as Gatorade or Pedialyte. d. Advise the patient to use over-the-counter loperamide (Imodium) to slow gastrointestinal (GI) motility.

B The initial response by the nurse should be further assessment of the patient. The other responses may be appropriate, depending on what is learned in the assessment. DIF: Cognitive Level: Analyze (analysis) REF: 932 OBJ: Special Questions: Prioritization TOP: Nursing Process: Assessment

28. Which assessment finding for a patient who has just been admitted with acute pyelonephritis is most important for the nurse to report to the health care provider? a. Complaint of flank pain c. Cloudy and foul-smelling urine b. Blood pressure 90/48 mm Hg d. Temperature 100.1° F (57.8° C)

B The low blood pressure indicates that urosepsis and septic shock may be occurring and should be immediately reported. The other findings are typical of pyelonephritis. DIF: Cognitive Level: Analyze (analysis) REF: 1038 OBJ: Special Questions: Prioritization TOP: Nursing Process: Assessment

24. A nurse assesses a male client who is recovering from a urologic procedure. Which assessment finding indicates an obstruction of urine flow? a. Severe pain b. Overflow incontinence c. Hypotension d. Blood-tinged urine

B The most common manifestation of urethral stricture after a urologic procedure is obstruction of urine flow. This rarely causes pain and has no impact on blood pressure. The client may experience overflow incontinence with the involuntary loss of urine when the bladder is distended. Blood in the urine is not a manifestation of the obstruction of urine flow. DIF: Applying/Application REF: 1373 KEY: Urethral strictures| urinary incontinence| postoperative nursing

11. A client has a recurrence of gastric cancer and is in the gastrointestinal clinic crying. What response by the nurse is most appropriate? a. "Do you have family or friends for support?" b. "I'd like to know what you are feeling now." c. "Well, we knew this would probably happen." d. "Would you like me to refer you to hospice?"

B The nurse assesses the client's emotional state with open-ended questions and statements and shows a willingness to listen to the client's concerns. Asking about support people is very limited in nature, and "yes-or-no" questions are not therapeutic. Stating that this was expected dismisses the client's concerns. The client may or may not be ready to hear about hospice, and this is another limited, yes-or-no question. DIF: Applying/Application REF: 1139 KEY: Gastrointestinal disorders| cancer| therapeutic communication| psychosocial response

8. A nurse cares for a client newly diagnosed with colon cancer who has become withdrawn from family members. Which action should the nurse take? a. Contact the provider and recommend a psychiatric consult for the client. b. Encourage the client to verbalize feelings about the diagnosis. c. Provide education about new treatment options with successful outcomes. d. Ask family and friends to visit the client and provide emotional support.

B The nurse recognizes that the client may be expressing feelings of grief. The nurse should encourage the client to verbalize feelings and identify fears to move the client through the phases of the grief process. A psychiatric consult is not appropriate for the client. The nurse should not brush aside the client's feelings with discussions related to cancer prognosis and treatment. The nurse should not assume that the client desires family or friends to visit or provide emotional support. DIF: Applying/Application REF: 1155 KEY: Colorectal cancer| coping

16. An older client has gastric cancer and is scheduled to have a partial gastrectomy. The family does not want the client told about her diagnosis. What action by the nurse is best? a. Ask the family why they feel this way. b. Assess family concerns and fears. c. Refuse to go along with the family's wishes. d. Tell the family that such secrets cannot be kept.

B The nurse should use open-ended questions and statements to fully assess the family's concerns and fears. Asking "why" questions often puts people on the defensive and is considered a barrier to therapeutic communication. Refusing to follow the family's wishes or keep their confidence will not help move this family from their position and will set up an adversarial relationship. DIF: Applying/Application REF: 1142 KEY: Gastrointestinal disorders| ethics| communication

22. A severely malnourished patient reports that he is Jewish. The nurse's initial action to meet his nutritional needs will be to a. have family members bring in food. b. ask the patient about food preferences. c. teach the patient about nutritious Kosher foods. d. order nutrition supplements that are manufactured Kosher.

B The nurse's first action should be further assessment whether or not the patient follows any specific religious guidelines that impact nutrition. The other actions may also be appropriate, based on the information obtained during the assessment. DIF: Cognitive Level: Analyze (analysis) REF: 856 OBJ: Special Questions: Prioritization TOP: Nursing Process: Assessment

2. A 74-yr-old male patient tells the nurse that growing old causes constipation so he has been using a suppository for constipation every morning. Which action should the nurse take first? a. Encourage the patient to increase oral fluid intake. b. Question the patient about risk factors for constipation. c. Suggest that the patient increase intake of high-fiber foods. d. Teach the patient that a daily bowel movement is unnecessary.

B The nurse's initial action should be further assessment of the patient for risk factors for constipation and for his usual bowel pattern. The other actions may be appropriate but will be based on the assessment. DIF: Cognitive Level: Analyze (analysis) REF: 933 OBJ: Special Questions: Prioritization TOP: Nursing Process: Implementation

17. After change-of-shift report, which patient will the nurse assess first? a. A 40-yr-old woman whose parenteral nutrition infusion bag has 30 minutes of solution left b. A 40-yr-old man with continuous enteral feedings who has developed pulmonary crackles c. A 30-yr-old man with 4+ generalized pitting edema and severe protein-calorie malnutrition d. A 30-yr-old woman whose gastrostomy tube is plugged after crushed medications were administered

B The patient data suggest aspiration has occurred, and rapid assessment and intervention are needed. The other patients should also be assessed soon, but the data about them do not suggest any immediately life-threatening complications. DIF: Cognitive Level: Analyze (analysis) REF: 865 OBJ: Special Questions: Prioritization | Special Questions: Multiple Patients TOP: Nursing Process: Planning

6. A patient has been on a 1000-calorie diet with a daily exercise routine and a prescription for sibutramine (Meridia) for 10 weeks. Which information obtained by the nurse is important to report to the health care provider? a. The patient has not lost any weight for the last 2 weeks. b. The patient tells the nurse about occasional palpitations. c. The patient complains about having chronic constipation. d. The patient reports walking only 3 days during the last week.

B The patient may be experiencing an increase in heart rate caused by the sibutramine (Meridia) that should be evaluated further by the health care provider. Plateaus during weight loss programs are common. Chronic constipation may be a side effect of the sibutramine, and the nurse should instruct the patient in measures such as eating more high fiber foods and increasing fluid intake. The nurse should reinforce the need to exercise more frequently, but no additional intervention by the health care provider is necessary regarding the patient's activity level. DIF: Cognitive Level: Application REF: 953-954 TOP: Nursing Process: Assessment

9. A 56-yr-old female patient is admitted to the hospital with new-onset nephrotic syndrome. Which assessment data will the nurse expect? a. Poor skin turgor c. Elevated urine ketones b. Recent weight gain d. Decreased blood pressure

B The patient with a nephrotic syndrome will have weight gain associated with edema. Hypertension is a clinical manifestation of nephrotic syndrome. Skin turgor is normal because of the edema. Urine protein is high. Ketones are not related to nephrotic syndrome. DIF: Cognitive Level: Understand (comprehension) REF: 1044 TOP: Nursing Process: Assessment

4. A 66-year-old patient has a body mass index (BMI) of 31 kg/m2, a normal C-reactive protein level, and low transferrin and albumin levels. The nurse will plan patient teaching to increase the patient's intake of foods that are high in a. iron. b. protein. c. calories. d. carbohydrate.

B The patient's C-reactive protein and transferrin levels indicate low protein stores. The BMI is in the obese range, so increasing caloric intake is not indicated. The data do not indicate a need for increased carbohydrate or iron intake. DIF: Cognitive Level: Application REF: 925-927 TOP: Nursing Process: Planning

4. A patient has a body mass index (BMI) of 31 kg/m2, a normal C-reactive protein level, and low serum transferrin and albumin levels. The nurse will plan patient teaching to increase the patient's intake of foods that are high in a. iron. c. calories. b. protein. d. carbohydrate.

B The patient's C-reactive protein and transferrin levels indicate low protein stores. The BMI is in the obese range, so increasing caloric intake is not indicated. The data do not indicate a need for increased carbohydrate or iron intake. DIF: Cognitive Level: Apply (application) REF: 860 TOP: Nursing Process: Planning

44. The nurse and a licensed practical/vocational nurse (LPN/LVN) are working together to care for a patient who had an esophagectomy 2 days ago. Which action by the LPN/LVN requires that the nurse intervene? a. The LPN/LVN uses soft swabs to provide oral care. b. The LPN/LVN positions the head of the bed in the flat position. c. The LPN/LVN includes the enteral feeding volume when calculating intake. d. The LPN/LVN encourages the patient to use pain medications before coughing.

B The patient's bed should be in Fowler's position to prevent reflux and aspiration of gastric contents. The other actions by the LPN/LVN are appropriate. DIF: Cognitive Level: Apply (application) REF: 907 OBJ: Special Questions: Delegation TOP: Nursing Process: Evaluation

14. A young adult male patient seen at the primary care clinic complains of feeling continued fullness after voiding and a split, spraying urine stream. The nurse will ask about a history of a. recent kidney trauma. c. recurrent bladder infection. b. gonococcal urethritis. d. benign prostatic hyperplasia.

B The patient's clinical manifestations are consistent with urethral strictures, a possible complication of gonococcal urethritis. These symptoms are not consistent with benign prostatic hyperplasia, kidney trauma, or bladder infection. DIF: Cognitive Level: Apply (application) REF: 1039 TOP: Nursing Process: Assessment

20. A 22-year-old who is hospitalized with anorexia nervosa is 5 ft 5 in (163 cm) tall and weighs 90 pounds (41 kg). Laboratory tests reveal hypokalemia and iron-deficiency anemia. Which nursing diagnosis has the highest priority for the patient? a. Risk for activity intolerance related to anemia and weakness b. Risk for electrolyte imbalance related to poor eating patterns c. Ineffective health maintenance related to obsession with body image d. Imbalanced nutrition: less than body requirements related to refusal to eat

B The patient's hypokalemia may lead to life-threatening cardiac dysrhythmias. The other diagnoses also are appropriate for this patient but are not associated with immediate risk for fatal complications. DIF: Cognitive Level: Application REF: 940 OBJ: Special Questions: Prioritization TOP: Nursing Process: Diagnosis

20. A 19-yr-old woman admitted with anorexia nervosa is 5 ft, 6 in (163 cm) tall and weighs 88 lb (41 kg). Laboratory tests reveal hypokalemia and iron-deficiency anemia. Which patient problem has the highest priority? a. Risk for activity intolerance b. Risk for electrolyte imbalance c. Ineffective health maintenance d. Imbalanced nutrition: less than body requirements

B The patient's hypokalemia may lead to life-threatening cardiac dysrhythmias. The other diagnoses are also appropriate for this patient but are not associated with immediate risk for fatal complications. DIF: Cognitive Level: Analyze (analysis) REF: 871 OBJ: Special Questions: Prioritization TOP: Nursing Process: Diagnosis

41. The nurse is administering IV fluid boluses and nasogastric irrigation to a patient with acute gastrointestinal (GI) bleeding. Which assessment finding is most important for the nurse to communicate to the health care provider? a. The bowel sounds are hyperactive in all four quadrants. b. The patient's lungs have crackles audible to the midchest. c. The nasogastric (NG) suction is returning coffee-ground material. d. The patient's blood pressure (BP) has increased to 142/84 mm Hg.

B The patient's lung sounds indicate that pulmonary edema may be developing as a result of the rapid infusion of IV fluid and that the fluid infusion rate should be slowed. The return of coffee-ground material in an NG tube is expected for a patient with upper GI bleeding. The BP is slightly elevated but would not be an indication to contact the health care provider immediately. Hyperactive bowel sounds are common when a patient has GI bleeding. DIF: Cognitive Level: Analyze (analysis) REF: 924 OBJ: Special Questions: Prioritization TOP: Nursing Process: Assessment

24. A patient admitted with a peptic ulcer has a nasogastric (NG) tube in place. When the patient develops sudden, severe upper abdominal pain, diaphoresis, and a firm abdomen, which action should the nurse take? a. Irrigate the NG tube. c. Give the ordered antacid. b. Check the vital signs. d. Elevate the foot of the bed.

B The patient's symptoms suggest acute perforation, and the nurse should assess for signs of hypovolemic shock. Irrigation of the NG tube, administration of antacids, or both would be contraindicated because any material in the stomach will increase the spillage into the peritoneal cavity. Elevating the foot of the bed may increase abdominal pressure and discomfort, as well as making it more difficult for the patient to breathe. DIF: Cognitive Level: Apply (application) REF: 924 TOP: Nursing Process: Implementation

24. A patient with a peptic ulcer who has a nasogastric (NG) tube develops sudden, severe upper abdominal pain, diaphoresis, and a very firm abdomen. Which action should the nurse take next? a. Irrigate the NG tube. b. Obtain the vital signs. c. Listen for bowel sounds. d. Give the ordered antacid.

B The patient's symptoms suggest acute perforation, and the nurse should assess for signs of hypovolemic shock. Irrigation of the NG tube, administration of antacids, or both would be contraindicated because any material in the stomach will increase the spillage into the peritoneal cavity. The nurse should assess the bowel sounds, but this is not the first action that should be taken. DIF: Cognitive Level: Application REF: 989-990 | 992-993 TOP: Nursing Process: Implementation

23. Two days after surgery for an ileal conduit, the patient will not look at the stoma or participate in care. The patient insists that no one but the ostomy nurse specialist care for the stoma. The nurse identifies a nursing diagnosis of a. anxiety related to effects of procedure on lifestyle. b. disturbed body image related to change in body function. c. readiness for enhanced coping related to need for information. d. self-care deficit, toileting, related to denial of altered body function.

B The patient's unwillingness to look at the stoma or participate in care indicates that disturbed body image is the best diagnosis. No data suggest that the impact on lifestyle is a concern for the patient. The patient does not appear to be ready for enhanced coping. The patient's insistence that only the ostomy nurse care for the stoma indicates that denial is not present. DIF: Cognitive Level: Application REF: 1157 | 1159-1160 | 1158-1159 TOP: Nursing Process: Diagnosis

23. A patient who had surgery for creation of an ileal conduit 3 days ago will not look at the stoma and requests that only the ostomy nurse specialist does the stoma care. The nurse identifies a nursing diagnosis of a. anxiety related to effects of procedure on lifestyle. b. disturbed body image related to change in function. c. readiness for enhanced coping related to need for information. d. self-care deficit (toileting) related to denial of altered body function.

B The patient's unwillingness to look at the stoma or participate in care indicates that disturbed body image is the best way to describe the problem. No data suggest that the impact on lifestyle is a concern for the patient. The patient does not appear to be ready for enhanced coping. The patient's insistence that only the ostomy nurse care for the stoma indicates that denial is not present. DIF: Cognitive Level: Apply (application) REF: 1065 TOP: Nursing Process: Analysis

23. An older patient with a bleeding duodenal ulcer has a nasogastric (NG) tube in place. The health care provider prescribes 30 mL of aluminum hydroxide/magnesium hydroxide (Maalox) to be instilled through the tube every hour. To evaluate the effectiveness of this treatment, the nurse a. monitors arterial blood gas values daily. b. periodically aspirates and tests gastric pH. c. checks each stool for the presence of occult blood. d. measures the volume of residual stomach contents.

B The purpose for antacids is to increase gastric pH. Checking gastric pH is the most direct way of evaluating the effectiveness of the medication. Arterial blood gases may change slightly, but this does not directly reflect the effect of antacids on gastric pH. Because the patient has upper gastrointestinal bleeding, occult blood in the stools will appear even after the acute bleeding has stopped. The amount of residual stomach contents is not a reflection of resolution of bleeding or of gastric pH. DIF: Cognitive Level: Apply (application) REF: 903 TOP: Nursing Process: Evaluation

16. How many grams of protein will the nurse recommend to meet the minimum daily requirement for a patient who weighs 145 pounds (66 kg)? a. 36 b. 53 c. 75 d. 98

B The recommended daily protein intake is 0.8 to 1 g/kg of body weight, which for this patient is 66 kg × 0.8 g = 52.8 or 53 g/day. DIF: Cognitive Level: Application REF: 922-923 OBJ: Special Questions: Alternate Item Format TOP: Nursing Process: Implementation

12. A nurse assesses a client who is recovering from extracorporeal shock wave lithotripsy for renal calculi. The nurse notes an ecchymotic area on the client's right lower back. Which action should the nurse take? a. Administer fresh-frozen plasma. b. Apply an ice pack to the site. c. Place the client in the prone position. d. Obtain serum coagulation test results.

B The shock waves from lithotripsy can cause bleeding into the tissues through which the waves pass. Application of ice can reduce the extent and discomfort of the bruising. Although coagulation test results and fresh-frozen plasma are used to assess and treat bleeding disorders, ecchymosis after this procedure is not unusual and does not warrant a higher level of intervention. Changing the client's position will not decrease bleeding. DIF: Applying/Application REF: 1386 KEY: Urolithiasis| postoperative nursing

11. When using a soft, silicone nasogastric tube for enteral feedings, the nurse will need to a. avoid giving medications through the feeding tube. b. flush the tubing after checking for residual volumes. c. administer continuous feedings using an infusion pump. d. replace the tube every 3 to 5 days to avoid mucosal damage.

B The soft silicone feeding tubes are small in diameter and can easily become clogged unless they are flushed after the nurse checks the residual volume. Either intermittent or continuous feedings can be given. The tubes are less likely to cause mucosal damage than the stiffer polyvinyl chloride tubes used for nasogastric suction and do not need to be replaced at certain intervals. Medications can be given through these tubes, but flushing after medication administration is important to avoid clogging. DIF: Cognitive Level: Application REF: 931-934 TOP: Nursing Process: Implementation

11. When caring for a patient with a soft, silicone nasogastric tube in place for enteral feedings, the nurse will a. avoid giving medications through the feeding tube. b. flush the tubing after checking for residual volumes. c. replace the tube every 3 days to avoid mucosal damage. d. administer continuous feedings using an infusion pump.

B The soft silicone feeding tubes are small in diameter and can easily become clogged unless they are flushed after the nurse checks the residual volume. Either intermittent or continuous feedings can be given. The tubes are less likely to cause mucosal damage than the stiffer polyvinyl chloride tubes used for nasogastric suction and do not need to be replaced at certain intervals. Medications can be given through these tubes, but flushing after medication administration is important to avoid clogging. DIF: Cognitive Level: Apply (application) REF: 865 TOP: Nursing Process: Implementation

41. Which action will the nurse anticipate taking for an otherwise healthy 50-yr-old who has just been diagnosed with stage 1 renal cell carcinoma? a. Prepare patient for a renal biopsy. b. Provide preoperative teaching about nephrectomy. c. Teach the patient about chemotherapy medications. d. Schedule for a follow-up appointment in 3 months.

B The treatment of choice in patients with localized renal tumors who have no co-morbid conditions is partial or total nephrectomy. A renal biopsy will not be needed in a patient who has already been diagnosed with renal cancer. Chemotherapy is used for metastatic renal cancer. Because renal cell cancer frequently metastasizes, treatment will be started as soon as possible after the diagnosis. DIF: Cognitive Level: Apply (application) REF: 1053 TOP: Nursing Process: Planning

12. A patient is receiving continuous enteral nutrition through a small-bore silicone feeding tube. What should the nurse plan for when this patient has a computed tomography (CT) scan ordered? a. Ask the health care provider to reschedule the scan. b. Shut the feeding off 30 to 60 minutes before the scan. c. Connect the feeding tube to continuous suction before and during the scan. d. Send a suction catheter with the patient in case of aspiration during the scan.

B The tube feeding should be shut off 30 to 60 minutes before any procedure requiring the patient to lie flat. Because the CT scan is ordered for diagnosis of patient problems, rescheduling is not usually an option. Prevention, rather than treatment, of aspiration is needed. Small-bore feeding tubes are soft and collapse easily with aspiration or suction, making nasogastric suction of gastric contents unreliable. DIF: Cognitive Level: Apply (application) REF: 866 TOP: Nursing Process: Planning

11. Which adult will the nurse plan to teach about risks associated with obesity? a. Man who has a BMI of 18 kg/m2 b. Man with a 42 in waist and 44 in hips c. Woman who has a body mass index (BMI) of 24 kg/m2 d. Woman with a waist circumference of 34 inches (86 cm)

B The waist-to-hip ratio for this patient is 0.95, which exceeds the recommended level of less than 0.80. A patient with a BMI of 18 kg/m2 is considered underweight. A BMI of 24 kg/m2 is normal. Health risks associated with obesity increase in women with a waist circumference larger than 35 in (89 cm) and men with a waist circumference larger than 40 in (102 cm). DIF: Cognitive Level: Understand (comprehension) REF: 875 TOP: Nursing Process: Planning

12. A client with peptic ulcer disease asks the nurse about taking slippery elm supplements. What response by the nurse is best? a. "Slippery elm has no benefit for this problem." b. "Slippery elm is often used for this disorder." c. "There is no evidence that this will work." d. "You should not take any herbal remedies."

B There are several complementary and alternative medicine regimens that are used for gastritis and peptic ulcer disease. Most have been tested on animals but not humans. Slippery elm is a common supplement used for this disorder. DIF: Understanding/Comprehension REF: 1131 KEY: Gastrointestinal disorders| complementary therapy| patient education

1. The nurse is caring for a client with peptic ulcer disease who reports sudden onset of sharp abdominal pain. On palpation, the client's abdomen is tense and rigid. What action takes priority? a. Administer the prescribed pain medication. b. Notify the health care provider immediately. c. Percuss all four abdominal quadrants. d. Take and document a set of vital signs.

B This client has manifestations of a perforated ulcer, which is an emergency. The priority is to get the client medical attention.. The nurse can take a set of vital signs while someone else calls the provider. The nurse should not percuss the abdomen or give pain medication since the client may need to sign consent for surgery. DIF: Applying/Application REF: 1132 KEY: Gastrointestinal disorders| nursing assessment| communication

2. After the nurse teaches a patient about the recommended amounts of foods from animal and plant sources, which menu selections indicate that the initial instructions about diet have been understood? a. 3 oz of lean beef, 2 oz of low-fat cheese, and a sliced tomato b. 3 oz of roasted pork, a cup of corn, and a cup of carrot sticks c. Cup of tossed salad and nonfat dressing topped with a chicken breast d. Half cup of tuna mixed with nonfat mayonnaise and a half cup of celery

B This selection is most consistent with the recommendation of the American Institute for Cancer Research that one third of the diet should be from animal sources and two thirds from plant source foods. The other choices all have higher ratios of animal origin foods to plant source foods than would be recommended. DIF: Cognitive Level: Apply (application) REF: 883 TOP: Nursing Process: Evaluation

18. After bariatric surgery, a patient who is being discharged tells the nurse, "I prefer to be independent. I am not interested in any support groups." Which response by the nurse is best? a. "I hope you change your mind so that I can suggest a group for you." b. "Tell me what types of resources you think you might use after this surgery." c. "Support groups have been found to lead to more successful weight loss after surgery." d. "Because there are many lifestyle changes after surgery, we recommend support groups."

B This statement allows the nurse to assess the individual patient's potential needs and preferences. The other statements offer the patient more information about the benefits of support groups but fail to acknowledge the patient's preferences. DIF: Cognitive Level: Analyze (analysis) REF: 884 TOP: Nursing Process: Implementation

10. A client has a nasogastric (NG) tube after a Nissen fundoplication. The nurse answers the call light and finds the client vomiting bright red blood with the NG tube lying on the floor. What action should the nurse take first? a. Notify the surgeon. b. Put on a pair of gloves. c. Reinsert the NG tube. d. Take a set of vital signs.

B To avoid exposure to blood and body fluids, the nurse first puts on a pair of gloves. Taking vital signs and notifying the surgeon are also appropriate, but the nurse must protect himself or herself first. The surgeon will reinsert the NG tube either at the bedside or in surgery if the client needs to go back to the operating room. DIF: Applying/Application REF: 1114 KEY: Gastrointestinal disorders| Standard Precautions

19. A patient's peripheral parenteral nutrition (PN) bag is nearly empty, and a new PN bag has not arrived yet from the pharmacy. Which intervention by the nurse is appropriate? a. Monitor the patient's capillary blood glucose every 6 hours. b. Infuse 5% dextrose in water until a new PN bag is delivered. c. Decrease the PN infusion rate to 10 mL/hr until a new bag arrives. d. Flush the peripheral line with saline until a new PN bag is available.

B To prevent hypoglycemia, the nurse should infuse a 5% dextrose solution until the next peripheral PN bag can be started. Decreasing the rate of the ordered PN infusion is beyond the nurse's scope of practice. Flushing the line and then waiting for the next bag may lead to hypoglycemia. Monitoring the capillary blood glucose every 6 hours would not identify hypoglycemia while awaiting the new PN bag. DIF: Cognitive Level: Apply (application) REF: 870 TOP: Nursing Process: Implementation

12. A 22-yr-old female patient with an exacerbation of ulcerative colitis is having 15 to 20 stools daily and has excoriated perianal skin. Which patient behavior indicates that teaching regarding maintenance of skin integrity has been effective? a. The patient uses incontinence briefs to contain loose stools. b. The patient uses witch hazel compresses to soothe irritation. c. The patient asks for antidiarrheal medication after each stool. d. The patient cleans the perianal area with soap after each stool.

B Witch hazel compresses are suggested to reduce anal irritation and discomfort. Incontinence briefs may trap diarrhea and increase the incidence of skin breakdown. Antidiarrheal medications are not given 15 to 20 times a day. The perianal area should be washed with plain water or pH balanced cleanser after each stool. DIF: Cognitive Level: Apply (application) REF: 950 TOP: Nursing Process: Evaluation

1. Which information in this male patient's electronic health record as shown in the accompanying figure will the nurse use to confirm that the patient has metabolic syndrome (select all that apply)? a. Weight b. Waist size c. Blood glucose d. Blood pressure e. Triglyceride level f. Total cholesterol level

B, C The patient's waist circumference, high-density lipoprotein level, and fasting blood glucose level indicate that he has metabolic syndrome. The other data are not used in making a metabolic syndrome diagnosis or do not meet the criteria for this diagnosis. DIF: Cognitive Level: Analyze (analysis) REF: 890 TOP: Nursing Process: Assessment

5. The nurse is working with clients who have esophageal disorders. The nurse should assess the clients for which manifestations? (Select all that apply.) a. Aphasia b. Dysphagia c. Eructation d. Halitosis e. Weight gain

B, C, D Common signs of esophageal disorders include dysphagia, eructation, halitosis, and weight loss. Aphasia is difficulty with speech, commonly seen after stroke. DIF: Remembering/Knowledge REF: 1124 KEY: Gastrointestinal disorders| nursing assessment

2. After teaching a client who is recovering from a colon resection, the nurse assesses the client's understanding. Which statements by the client indicate a correct understanding of the teaching? (Select all that apply.) a. "I must change the ostomy appliance daily and as needed." b. "I will use warm water and a soft washcloth to clean around the stoma." c. "I might start bicycling and swimming again once my incision has healed." d. "Cutting the flange will help it fit snugly around the stoma to avoid skin breakdown." e. "I will check the stoma regularly to make sure that it stays a deep red color." f. "I must avoid dairy products to reduce gas and odor in the pouch.."

B, C, D The ostomy appliance should be changed as needed when the adhesive begins to decrease, placing the appliance at risk of leaking. Changing the appliance daily can cause skin breakdown as the adhesive will still be secured to the client's skin. The client should avoid using soap to clean around the stoma because it might prevent effective adhesion of the ostomy appliance. The client should use warm water and a soft washcloth instead. The tissue of the stoma is very fragile, and scant bleeding may occur when the stoma is cleaned. The flange should be cut to fit snugly around the stoma to reduce contact between excretions and the client's skin. Exercise (other than some contact sports) is important for clients with an ostomy. The stoma should remain a soft pink color. A deep red or purple hue indicates ischemia and should be reported to the surgeon right away. Yogurt and buttermilk can help reduce gas in the pouch, so the client need not avoid dairy products. DIF: Applying/Application REF: 1154 KEY: Colorectal cancer| postoperative care

1. Which information will the nurse include when teaching a patient how to avoid chronic constipation (select all that apply)? a. Stimulant and saline laxatives can be used regularly. b. Bulk-forming laxatives are an excellent source of fiber. c. Walking or cycling frequently will help bowel motility. d. A good time for a bowel movement may be after breakfast. e. Some over-the-counter (OTC) medications cause constipation.

B, C, D, E Stimulant and saline laxatives should be used infrequently. Use of bulk-forming laxatives, regular early morning timing of defecation, regular exercise, and avoiding many OTC medications will help the patient avoid constipation. DIF: Cognitive Level: Understand (comprehension) REF: 935 TOP: Nursing Process: Planning

4. A client's small-bore feeding tube has become occluded after the nurse administered medications. What actions by the nurse are best? (Select all that apply.) a. Attempt to dissolve the clog by instilling a cola product. b. Determine if any of the medications come in liquid form. c. Flush the tube before and after administering medications. d. Mix all medications in the formula and use a feeding pump. e. Try to flush the tube with 30 mL of water and gentle pressure.

B, C, E If the tube is obstructed, use a 50-mL syringe and gentle pressure to attempt to open the tube. Cola products should not be used unless water is not effective. To prevent future problems, determine if any of the medications can be dispensed in liquid form and flush the tube with water before and after medication administration. Do not mix medications with the formula. DIF: Remembering/Knowledge REF: 1243 KEY: Nutritional disorders| tube feedings| medication administration

1. A patient has been diagnosed with urinary tract calculi that are high in uric acid. Which foods will the nurse teach the patient to avoid (select all that apply)? a. Milk b. Liver c. Spinach d. Chicken e. Cabbage f. Chocolate

B, D Meats contain purines, which are metabolized to uric acid. The other foods might be restricted in patients who have calcium or oxalate stones. DIF: Cognitive Level: Understand (comprehension) REF: 1046 TOP: Nursing Process: Planning

3. A nurse is designing a community education program to meet the Healthy People 2020 objectives for nutrition and weight status. What information about these goals does the nurse use to plan this event? (Select all that apply.) a. Decrease the amount of fruit to 1.1 cups/1000 calories. b. Increase the amount of vegetables to 1.1 cups/1000 calories. c. Increase the number of adults at a healthy weight by 25%. d. Reduce the number of adults who are obese by 10%. e. Reduce the consumption of saturated fat by nearly 10%.

B, D, E Some of the goals in this initiative include increasing fruit consumption to 0.9 cups/1000 calories, increasing vegetable intake to 1.1 cups/1000 calories, increasing the number of people at a healthy weight by 10%, decreasing the number of adults who are obese by 10%, and reducing the consumption of saturated fats by 9.5%. DIF: Remembering/Knowledge REF: 1243 KEY: Nutritional disorders| obesity| health promotion

4. A nurse assesses a client with a fungal urinary tract infection (UTI). Which assessments should the nurse complete? (Select all that apply.) a. Palpate the kidneys and bladder. b. Assess the medical history and current medical problems. c. Perform a bladder scan to assess post-void residual. d. Inquire about recent travel to foreign countries. e. Obtain a current list of medications.

B, E Clients who are severely immunocompromised or who have diabetes mellitus are more prone to fungal UTIs. The nurse should assess for these factors by asking about medical history, current medical problems, and the current medication list. A physical examination and a post-void residual may be needed, but not until further information is obtained indicating that these examinations are necessary. Travel to foreign countries probably would not be important because, even if exposed, the client needs some degree of compromised immunity to develop a fungal UTI. DIF: Applying/Application REF: 1377 KEY: Cystitis| assessment/diagnostic examination

3. A nurse teaches clients about the difference between urge incontinence and stress incontinence. Which statements should the nurse include in this education? (Select all that apply.) a. "Urge incontinence involves a post-void residual volume less than 50 mL." b. "Stress incontinence occurs due to weak pelvic floor muscles." c. "Stress incontinence usually occurs in people with dementia." d. "Urge incontinence can be managed by increasing fluid intake." e. "Urge incontinence occurs due to abnormal bladder contractions."

B, E Clients who suffer from stress incontinence have weak pelvic floor muscles or urethral sphincter and cannot tighten their urethra sufficiently to overcome the increased detrusor pressure. Stress incontinence is common after childbirth, when the pelvic muscles are stretched and weakened from pregnancy and delivery. Urge incontinence occurs in people who cannot suppress the contraction signal from the detrusor muscle. Abnormal detrusor contractions may be a result of neurologic abnormalities including dementia, or may occur with no known abnormality. Post-void residual is associated with reflex incontinence, not with urge incontinence or stress incontinence. Management of urge incontinence includes decreasing fluid intake, especially in the evening hours. DIF: Understanding/Comprehension REF: 1375 KEY: Urinary incontinence| patient education

4. A 26-yr-old woman is being evaluated for vomiting and abdominal pain. Which question from the nurse will be most useful in determining the cause of the patient's symptoms? a. "What type of foods do you eat?" b. "Is it possible that you are pregnant?" c. "Can you tell me more about the pain?" d. "What is your usual elimination pattern?"

C A complete description of the pain provides clues about the cause of the problem. Although the nurse should ask whether the patient is pregnant to determine whether the patient might have an ectopic pregnancy and before any radiology studies are done, this information is not the most useful in determining the cause of the pain. The usual diet and elimination patterns are less helpful in determining the reason for the patient's symptoms. DIF: Cognitive Level: Analyze (analysis) REF: 939 TOP: Nursing Process: Assessment

11. The nurse teaches an adult patient to prevent the recurrence of renal calculi by a. using a filter to strain all urine. b. avoiding dietary sources of calcium. c. drinking 2000 to 3000 mL of fluid each day. d. choosing diuretic fluids such as coffee and tea.

C A fluid intake of 2000 to 3000 mL/day is recommended to help flush out minerals before stones can form. Avoidance of calcium is not usually recommended for patients with renal calculi. Coffee tends to increase stone recurrence. There is no need for a patient to strain all urine routinely after a stone has passed, and this will not prevent stones. DIF: Cognitive Level: Apply (application) REF: 1048 TOP: Nursing Process: Implementation

34. A 58-yr-old patient has just been admitted to the emergency department with nausea and vomiting. Which information requires the most rapid intervention by the nurse? a. The patient has been vomiting for 4 days. b. The patient takes antacids 8 to 10 times a day. c. The patient is lethargic and difficult to arouse. d. The patient has had a small intestinal resection.

C A lethargic patient is at risk for aspiration, and the nurse will need to position the patient to decrease aspiration risk. The other information is also important to collect, but it does not require as quick action as the risk for aspiration. DIF: Cognitive Level: Analyze (analysis) REF: 896 OBJ: Special Questions: Prioritization TOP: Nursing Process: Assessment

7. A nurse prepares a client for a colonoscopy scheduled for tomorrow. The client states, "My doctor told me that the fecal occult blood test was negative for colon cancer. I don't think I need the colonoscopy and would like to cancel it." How should the nurse respond? a. "Your doctor should not have given you that information prior to the colonoscopy." b. "The colonoscopy is required due to the high percentage of false negatives with the blood test." c. "A negative fecal occult blood test does not rule out the possibility of colon cancer." d. "I will contact your doctor so that you can discuss your concerns about the procedure."

C A negative result from a fecal occult blood test does not completely rule out the possibility of colon cancer. To determine whether the client has colon cancer, a colonoscopy should be performed so the entire colon can be visualized and a tissue sample taken for biopsy. The client may want to speak with the provider, but the nurse should address the client's concerns prior to contacting the provider. DIF: Understanding/Comprehension REF: 1151 KEY: Colorectal cancer| assessment/diagnostic examination

15. A nurse assesses a client with bladder cancer who is recovering from a complete cystectomy with ileal conduit. Which assessment finding should alert the nurse to urgently contact the health care provider? a. The ileostomy is draining blood-tinged urine. b. There is serous sanguineous drainage present on the surgical dressing. c. The ileostomy stoma is pale and cyanotic in appearance. d. Oxygen saturations are 92% on room air.

C A pale or cyanotic stoma indicates impaired circulation to the stoma and must be treated to prevent necrosis. Blood-tinged urine and serous sanguineous drainage are expected after this type of surgery. Oxygen saturation of 92% on room air is at the low limit of normal. DIF: Applying/Application REF: 1390 KEY: Urothelial cancer| postoperative nursing

4. Which finding in the mouth of a patient who uses smokeless tobacco is suggestive of oral cancer? a. Bleeding during tooth brushing b. Painful blisters at the lip border c. Red, velvety patches on the buccal mucosa d. White, curdlike plaques on the posterior tongue

C A red, velvety patch suggests erythroplasia, which has a high incidence (>50%) of progression to squamous cell carcinoma. The other lesions are suggestive of acute processes (e.g., gingivitis, oral candidiasis, herpes simplex). DIF: Cognitive Level: Understand (comprehension) REF: 898 TOP: Nursing Process: Assessment

4. When the nurse is assessing the mouth of a patient who uses smokeless tobacco for signs of oral cancer, which finding will be of most concern? a. Bleeding during tooth brushing b. Painful blisters at the border of the lips c. Red, velvety patches on the buccal mucosa d. White, curdlike plaques on the posterior tongue

C A red, velvety patch suggests erythroplasia, which has a high incidence (greater than 50%) of progression to squamous cell carcinoma. The other lesions are suggestive of acute processes (gingivitis, oral candidiasis, and herpes simplex). DIF: Cognitive Level: Comprehension REF: 968-969 TOP: Nursing Process: Assessment

4. A client with a bleeding gastric ulcer is having a nuclear medicine scan. What action by the nurse is most appropriate? a. Assess the client for iodine or shellfish allergies. b. Educate the client on the side effects of sedation. c. Inform the client a second scan may be needed. d. Teach the client about bowel preparation for the scan.

C A second scan may be performed in 1 to 2 days to see if interventions have worked. The nuclear medicine scan does not use iodine-containing contrast dye or sedation. There is no required bowel preparation. DIF: Understanding/Comprehension REF: 1134 KEY: Gastrointestinal disorders| patient education| nuclear medicine

6. To determine possible causes, the nurse will ask a patient admitted with acute glomerulonephritis about a. recent bladder infection. c. recent sore throat and fever. b. history of kidney stones. d. history of high blood pressure.

C Acute glomerulonephritis frequently occurs after a streptococcal infection such as strep throat. It is not caused by kidney stones, hypertension, or urinary tract infection. DIF: Cognitive Level: Apply (application) REF: 1041 TOP: Nursing Process: Assessment

12. A client just returned to the surgical unit after a gastric bypass. What action by the nurse is the priority? a. Assess the client's pain. b. Check the surgical incision. c. Ensure an adequate airway. d. Program the morphine pump.

C All actions are appropriate care measures for this client; however, airway is always the priority. Bariatric clients tend to have short, thick necks that complicate airway management. DIF: Applying/Application REF: 1251 KEY: Nutritional disorders| obesity| nursing assessment

8. A nurse answers a client's call light and finds the client in the bathroom, vomiting large amounts of bright red blood. Which action should the nurse take first? a. Assist the client back to bed. b. Notify the provider immediately. c. Put on a pair of gloves. d. Take a set of vital signs.

C All of the actions are appropriate; however, the nurse should put on a pair of gloves first to avoid contamination with blood or body fluids. DIF: Applying/Application REF: 1128 KEY: Gastrointestinal disorders| Standard Precautions| infection control

9. A confused client with pneumonia is admitted with an indwelling catheter in place. During interdisciplinary rounds the following day, which question should the nurse ask the primary health care provider? a. "Do you want daily weights on this client?" b. "Will the client be able to return home?" c. "Can we discontinue the indwelling catheter?" d. "Should we get another chest x-ray today?"

C An indwelling catheter dramatically increases the risks of urinary tract infection and urosepsis. Nursing staff should ensure that catheters are left in place only as long as they are medically needed. The nurse should inquire about removing the catheter. All other questions might be appropriate, but because of client safety, this question takes priority. DIF: Applying/Application REF: 1368 KEY: Infection control

18. Following rectal surgery, a patient voids about 50 mL of urine every 30 to 60 minutes for the first 4 hours. Which nursing action is most appropriate? a. Monitor the patient's intake and output overnight. b. Have the patient drink small amounts of fluid frequently. c. Use an ultrasound scanner to check the postvoiding residual volume. d. Reassure the patient that this is normal after anesthesia for rectal surgery.

C An ultrasound scanner can be used to check for residual urine after the patient voids. Because the patient's history and clinical manifestations are consistent with overflow incontinence, it is not appropriate to have the patient drink small amounts. Although overflow incontinence is not unusual after surgery, the nurse should intervene to correct the physiologic problem, not just reassure the patient. The patient may develop reflux into the renal pelvis and discomfort from a full bladder if the nurse waits to address the problem for several hours. DIF: Cognitive Level: Analyze (analysis) REF: 1061 TOP: Nursing Process: Implementation

1. Which action will the nurse include in the plan of care for a patient who is being admitted with Clostridium difficile? a. Teach the patient about proper food storage. b. Order a diet without dairy products for the patient. c. Place the patient in a private room on contact isolation. d. Teach the patient about why antibiotics will not be used.

C Because C. difficile is highly contagious, the patient should be placed in a private room, and contact precautions should be used. There is no need to restrict dairy products for this type of diarrhea. Metronidazole (Flagyl) is frequently used to treat C. difficile infections. Improper food handling and storage do not cause C. difficile. DIF: Cognitive Level: Apply (application) REF: 932 TOP: Nursing Process: Planning

7. When admitting a patient with a stroke who is unconscious and unresponsive to stimuli, the nurse learns from the patient's family that the patient has a history of gastroesophageal reflux disease (GERD). The nurse will plan to do frequent assessments of the patient's a. apical pulse. b. bowel sounds. c. breath sounds. d. abdominal girth.

C Because GERD may cause aspiration, the unconscious patient is at risk for developing aspiration pneumonia. Bowel sounds, abdominal girth, and apical pulse will not be affected by the patient's stroke or GERD and do not require more frequent monitoring than the routine. DIF: Cognitive Level: Application REF: 972-973 TOP: Nursing Process: Assessment

7. A 68-yr-old male patient with a stroke is unconscious and unresponsive to stimuli. After learning that the patient has a history of gastroesophageal reflux disease (GERD), the nurse will plan to do frequent assessments of the patient's a. apical pulse. c. breath sounds. b. bowel sounds. d. abdominal girth.

C Because GERD may cause aspiration, the unconscious patient is at risk for developing aspiration pneumonia. Bowel sounds, abdominal girth, and apical pulse will not be affected by the patient's stroke or GERD and do not require more frequent monitoring than the routine. DIF: Cognitive Level: Apply (application) REF: 905 TOP: Nursing Process: Assessment

13. A 28-yr-old male patient is diagnosed with polycystic kidney disease. Which information is most appropriate for the nurse to include in teaching at this time? a. Complications of renal transplantation b. Methods for treating severe chronic pain c. Options to consider for genetic counseling d. Differences between hemodialysis and peritoneal dialysis

C Because a 28-yr-old patient may be considering having children, the nurse should include information about genetic counseling when teaching the patient. A well-managed patient will not need to choose between hemodialysis and peritoneal dialysis or know about the effects of transplantation for many years. There is no indication that the patient has chronic pain. DIF: Cognitive Level: Apply (application) REF: 1051 TOP: Nursing Process: Implementation

9. After abdominal surgery, a patient with protein calorie malnutrition is receiving parenteral nutrition (PN). Which is the best indicator that the patient is receiving adequate nutrition? a. Serum albumin level is 3.5 mg/dL. b. Fluid intake and output are balanced. c. Surgical incision is healing normally. d. Blood glucose is less than 110 mg/dL.

C Because poor wound healing is a possible complication of malnutrition for this patient, normal healing of the incision is an indicator of the effectiveness of the PN in providing adequate nutrition. Blood glucose is monitored to prevent the complications of hyperglycemia and hypoglycemia, but it does not indicate that the patient's nutrition is adequate. The intake and output will be monitored, but do not indicate that the PN is effective. The albumin level is in the low-normal range but does not reflect adequate caloric intake, which is also important for the patient. DIF: Cognitive Level: Analyze (analysis) REF: 869 TOP: Nursing Process: Evaluation

40. Which prescribed action will the nurse implement first for a patient who has vomited 1100 mL of blood? a. Give an IV H2 receptor antagonist. b. Draw blood for typing and crossmatching. c. Administer 1 L of lactated Ringer's solution. d. Insert a nasogastric (NG) tube and connect to suction.

C Because the patient has vomited a large amount of blood, correction of hypovolemia and prevention of hypovolemic shock are the priorities. The other actions also are important to implement quickly, but are not the highest priorities. DIF: Cognitive Level: Analyze (analysis) REF: 923 OBJ: Special Questions: Prioritization TOP: Nursing Process: Implementation

1. A 53-yr-old male patient with deep partial-thickness burns from a chemical spill in the workplace experiences severe pain followed by nausea during dressing changes. Which action will be most useful in decreasing the patient's nausea? a. Keep the patient NPO for 2 hours before dressing changes. b. Give the ordered prochlorperazine before dressing changes. c. Administer the prescribed morphine sulfate before dressing changes. d. Avoid performing dressing changes close to the patient's mealtimes.

C Because the patient's nausea is associated with severe pain, it is likely that it is precipitated by stress and pain. The best treatment will be to provide adequate pain medication before dressing changes. The nurse should avoid doing painful procedures close to mealtimes, but nausea or vomiting that occur at other times also should be addressed. Keeping the patient NPO does not address the reason for the nausea and vomiting and will have an adverse effect on the patient's nutrition. Administration of antiemetics is not the best choice for a patient with nausea caused by pain. However, an antiemetic may be added later if the nausea persists despite pain management. DIF: Cognitive Level: Analyze (analysis) REF: 894 TOP: Nursing Process: Implementation

1. A patient with deep partial-thickness burns experiences severe pain associated with nausea during dressing changes. Which action will be most useful in decreasing the patient's nausea? a. The patient NPO for 2 hours before and after dressing changes. b. Avoid performing dressing changes close to the patient's mealtimes. c. Administer the prescribed morphine sulfate before dressing changes. d. Give the ordered prochlorperazine (Compazine) before dressing changes.

C Because the patient's nausea is associated with severe pain, it is likely that it is precipitated by stress and pain. The best treatment will be to provide adequate pain medication before dressing changes. The nurse should avoid doing painful procedures close to mealtimes, but nausea/vomiting that occur at other times also should be addressed. Keeping the patient NPO does not address the reason for the nausea and vomiting and will have an adverse effect on the patient's nutrition. Administration of antiemetics is not the best choice for a patient with nausea caused by pain. DIF: Cognitive Level: Application REF: 963-964 TOP: Nursing Process: Implementation

21. A 71-yr-old patient had an abdominal-perineal resection for colon cancer. Which nursing action is most important to include in the plan of care for the day after surgery? a. Teach about a low-residue diet. b. Monitor output from the stoma. c. Assess the perineal drainage and incision. d. Encourage acceptance of the colostomy stoma.

C Because the perineal wound is at high risk for infection, the initial care is focused on assessment and care of this wound. Teaching about diet is best done closer to discharge from the hospital. There will be very little drainage into the colostomy until peristalsis returns. The patient will be encouraged to assist with the colostomy, but this is not the highest priority in the immediate postoperative period. DIF: Cognitive Level: Analyze (analysis) REF: 956 TOP: Nursing Process: Planning

21. Which nursing action is most important to include in the plan of care for a patient who had an abdominal-perineal resection the previous day? a. Teach about a low-residue diet. b. Monitor output from the stoma. c. Assess the perineal drainage and incision. d. Encourage acceptance of the colostomy stoma.

C Because the perineal wound is at high risk for infection, the initial care is focused on assessment and care of this wound. Teaching about diet is best done closer to discharge from the hospital. There will be very little drainage into the colostomy until peristalsis returns. The patient will be encouraged to assist with the colostomy, but this is not the highest priority in the immediate postoperative period. DIF: Cognitive Level: Application REF: 1039 TOP: Nursing Process: Planning

36. A patient undergoes a nephrectomy after having massive trauma to the kidney. Which assessment finding obtained postoperatively is most important to communicate to the surgeon? a. Blood pressure is 102/58. b. Incisional pain level is 8/10. c. Urine output is 20 mL/hr for 2 hours. d. Crackles are heard at both lung bases.

C Because the urine output should be at least 0.5 mL/kg/hr, a 40 mL output for 2 hours indicates that the patient may have decreased renal perfusion because of bleeding, inadequate fluid intake, or obstruction at the suture site. The blood pressure requires ongoing monitoring but does not indicate inadequate perfusion at this time. The patient should cough and deep breathe, but the crackles do not indicate a need for an immediate change in therapy. The incisional pain should be addressed, but this is not as potentially life threatening as decreased renal perfusion. In addition, the nurse can medicate the patient for pain. DIF: Cognitive Level: Application REF: 1154-1155 OBJ: Special Questions: Prioritization TOP: Nursing Process: Assessment

28. After a patient has had a hemorrhoidectomy at an outpatient surgical center, which instructions will the nurse include in discharge teaching? a. Maintain a low-residue diet until the surgical area is healed. b. Use ice packs on the perianal area to relieve pain and swelling. c. Take prescribed pain medications before you expect a bowel movement. d. Delay having a bowel movement for several days until you are well healed.

C Bowel movements may be very painful, and patients may avoid defecation unless pain medication is taken before the bowel movement. A high-residue diet will increase stool bulk and prevent constipation. Delay of bowel movements is likely to lead to constipation. Warm sitz baths rather than ice packs are used to relieve pain and keep the surgical area clean. DIF: Cognitive Level: Apply (application) REF: 969 TOP: Nursing Process: Implementation

29. Which instructions will the nurse include in discharge teaching for a patient who has had a hemorrhoidectomy at an outpatient surgical center? a. Maintain a low-residue diet until the surgical area is healed. b. Use ice packs on the perianal area to relieve pain and swelling. c. Take prescribed pain medications before a bowel movement is expected. d. Delay having a bowel movement for several days until healing has occurred.

C Bowel movements may be very painful, and patients may avoid defecation unless pain medication is taken before the bowel movement. Delay of bowel movements is likely to lead to constipation. A high-residue diet will increase stool bulk and prevent constipation. Sitz baths are used to relieve pain and keep the surgical area clean. DIF: Cognitive Level: Application REF: 1053 TOP: Nursing Process: Implementation

3. A 38-year old woman receiving chemotherapy for breast cancer develops a Candida albicans oral infection. The nurse will anticipate the need for a. hydrogen peroxide rinses. b. the use of antiviral agents. c. administration of nystatin tablets. d. referral to a dentist for professional tooth cleaning.

C Candida albicans infections are treated with an antifungal such as nystatin. Peroxide rinses would be painful. Oral saltwater rinses may be used but will not cure the infection. Antiviral agents are used for viral infections such as herpes simplex. Referral to a dentist is indicated for gingivitis but not for Candida infection. DIF: Cognitive Level: Apply (application) REF: 897 TOP: Nursing Process: Planning

18. A nurse teaches a client who is recovering from a colon resection. Which statement should the nurse include in this client's plan of care? a. "You may experience nausea and vomiting for the first few weeks." b. "Carbonated beverages can help decrease acid reflux from anastomosis sites." c. "Take a stool softener to promote softer stools for ease of defecation." d. "You may return to your normal workout schedule, including weight lifting."

C Clients recovering from a colon resection should take a stool softener as prescribed to keep stools a soft consistency for ease of passage. Nausea and vomiting are symptoms of intestinal obstruction and perforation and should be reported to the provider immediately. The client should be advised to avoid gas-producing foods and carbonated beverages, and avoid lifting heavy objects or straining on defecation. DIF: Applying/Application REF: 1155 KEY: Colorectal cancer| postoperative nursing| bowel care

24. Which information from a patient who had a transurethral resection with fulguration for bladder cancer 3 days ago is most important to report to the health care provider? a. The patient is voiding every 4 hours. b. The patient is using opioids for pain. c. The patient has seen clots in the urine. d. The patient is anxious about the cancer.

C Clots in the urine are not expected and require further follow-up. Voiding every 4 hours, use of opioids for pain, and anxiety are typical after this procedure. DIF: Cognitive Level: Analyze (analysis) REF: 1054 OBJ: Special Questions: Prioritization TOP: Nursing Process: Assessment

26. Which patient statement indicates that the nurse's postoperative teaching after a gastroduodenostomy has been effective? a. "I will drink more liquids with my meals." b. "I should choose high carbohydrate foods." c. "Vitamin supplements may prevent anemia." d. "Persistent heartburn is common after surgery."

C Cobalamin deficiency may occur after partial gastrectomy, and the patient may need to receive cobalamin via injections or nasal spray. Although peptic ulcer disease may recur, persistent heartburn is not expected after surgery, and the patient should call the health care provider if this occurs. Ingestion of liquids with meals is avoided to prevent dumping syndrome. Foods that have moderate fat and low carbohydrate should be chosen to prevent dumping syndrome. DIF: Cognitive Level: Apply (application) REF: 919 TOP: Nursing Process: Evaluation

26. The nurse implements discharge teaching for a patient following a gastroduodenostomy for treatment of a peptic ulcer. Which patient statement indicates that the teaching has been effective? a. "Persistent heartburn is expected after surgery." b. "I will try to drink liquids along with my meals." c. "Vitamin supplements may be needed to prevent problems with anemia." d. "I will need to choose foods that are low in fat and high in carbohydrate."

C Cobalamin deficiency may occur after partial gastrectomy, and the patient may need to receive cobalamin via injections or nasal spray. Foods that have moderate fat and low carbohydrate should be chosen to prevent dumping syndrome. Ingestion of liquids with meals is avoided to prevent dumping syndrome. Although peptic ulcer disease may recur, persistent heartburn is not expected after surgery and the patient should call the health care provider if this occurs. DIF: Cognitive Level: Application REF: 996-998 TOP: Nursing Process: Evaluation

19. To evaluate an obese patient for adverse effects of lorcaserin (Belviq), which action will the nurse take? a. Take the apical pulse rate. c. Ask about bowel movements. b. Check sclera for jaundice. d. Assess for agitation or restlessness.

C Constipation is a common side effect of lorcaserin. The other assessments would be appropriate for other weight-loss medications. DIF: Cognitive Level: Apply (application) REF: 884 TOP: Nursing Process: Evaluation

19. In planning preoperative teaching for a patient undergoing a Roux-en-Y gastric bypass as treatment for morbid obesity, the nurse places the highest priority on a. demonstrating passive range-of-motion exercises to the legs. b. discussing the necessary postoperative modifications in lifestyle. c. teaching the patient proper coughing and deep breathing techniques. d. educating the patient about the postoperative presence of a nasogastric (NG) tube.

C Coughing and deep breathing can prevent major postoperative complications such as carbon monoxide retention and hypoxemia. Information about passive range of motion, the NG tube, and postoperative modifications in lifestyle also will be discussed, but avoidance of respiratory complications is the priority goal after surgery. DIF: Cognitive Level: Application REF: 957-958 OBJ: Special Questions: Prioritization TOP: Nursing Process: Planning

42. Which information about a patient with Goodpasture syndrome requires the most rapid action by the nurse? a. Blood urea nitrogen level is 70 mg/dL. b. Urine output over the last 2 hours is 30 mL. c. Audible crackles bilaterally over the posterior chest to the midscapular level. d. Elevated level of antiglomerular basement membrane (anti-GBM) antibodies.

C Crackles heard to a high level indicate a need for rapid actions such as assessment of O2 saturation, reporting the findings to the health care provider, initiating O2 therapy, and dialysis. The other findings will also be reported but are typical of Goodpasture syndrome and do not require immediate nursing action. DIF: Cognitive Level: Analyze (analysis) REF: 1043 OBJ: Special Questions: Prioritization TOP: Nursing Process: Planning

40. Which of these nursing activities included in the care of a patient with a new colostomy should the nurse delegate to nursing assistive personnel (NAP)? a. Document the appearance of the stoma. b. Place the pouching system over the ostomy. c. Drain and measure the output from the ostomy. d. Check the skin around the ostomy for breakdown.

C Draining and measuring the output from the ostomy is included in NAP education and scope of practice. The other actions should be implemented by LPNs or RNs. DIF: Cognitive Level: Application REF: 1044 OBJ: Special Questions: Delegation TOP: Nursing Process: Planning

39. Which activity in the care of a patient with a new colostomy could the nurse delegate to unlicensed assistive personnel (UAP)? a. Document the appearance of the stoma. b. Place a pouching system over the ostomy. c. Drain and measure the output from the ostomy. d. Check the skin around the stoma for breakdown.

C Draining and measuring the output from the ostomy is included in UAP education and scope of practice. The other actions should be implemented by LPNs or RNs. DIF: Cognitive Level: Apply (application) REF: 960 OBJ: Special Questions: Delegation TOP: Nursing Process: Planning

13. After the nurse has provided patient teaching about recommended dietary choices for a patient with an acute exacerbation of inflammatory bowel disease (IBD), which diet choice by the patient indicates a need for more teaching? a. Scrambled eggs b. White toast and jam c. Oatmeal with cream d. Pancakes with syrup

C During acute exacerbations of IBD, the patient should avoid high-fiber foods such as whole grains. High-fat foods also may cause diarrhea in some patients. The other choices are low residue and would be appropriate for this patient. DIF: Cognitive Level: Application REF: 1030-1031 TOP: Nursing Process: Evaluation

13. Which diet choice by the patient with an acute exacerbation of inflammatory bowel disease (IBD) indicates a need for more teaching? a. Scrambled eggs c. Oatmeal with cream b. White toast and jam d. Pancakes with syrup

C During acute exacerbations of IBD, the patient should avoid high-fiber foods such as whole grains. High-fat foods also may cause diarrhea in some patients. The other choices are low residue and would be appropriate for this patient. DIF: Cognitive Level: Apply (application) REF: 949 TOP: Nursing Process: Evaluation

2. A patient with a body mass index (BMI) of 17 kg/m2 and a low albumin level is being admitted by the nurse. Which assessment finding will the nurse expect to find? a. Restlessness b. Hypertension c. Pitting edema d. Food allergies

C Edema occurs when serum albumin levels and plasma oncotic pressure decrease. The blood pressure and level of consciousness are not directly affected by malnutrition. Food allergies are not an indicator of nutritional status. DIF: Cognitive Level: Application REF: 923 | 925 TOP: Nursing Process: Assessment

2. A 76-yr-old woman with a body mass index (BMI) of 17 kg/m2 and a low serum albumin level is being admitted by the nurse. Which assessment finding will the nurse expect to find? a. Restlessness c. Pitting edema b. Hypertension d. Food allergies

C Edema occurs when serum albumin levels and plasma oncotic pressure decrease. The blood pressure and level of consciousness are not directly affected by malnutrition. Food allergies are not an indicator of nutritional status. DIF: Cognitive Level: Apply (application) REF: 858 TOP: Nursing Process: Assessment

14. The nurse will plan to teach the patient with newly diagnosed achalasia that a. a liquid or blenderized diet will be necessary. b. drinking fluids with meals should be avoided. c. endoscopic procedures may be used for treatment. d. lying down and resting after meals is recommended.

C Endoscopic and laparoscopic procedures are the most effective therapy for improving symptoms caused by achalasia. Patients are advised to drink fluid with meals. Keeping the head elevated after eating will improve esophageal emptying. A semisoft diet is recommended to improve esophageal emptying. DIF: Cognitive Level: Application REF: 980-981 TOP: Nursing Process: Planning

42. After the nurse has completed teaching a patient with newly diagnosed eosinophilic esophagitis about the management of the disease, which patient action indicates that the teaching has been effective? a. Patient orders nonfat milk for each meal. b. Patient uses the prescribed corticosteroid inhaler. c. Patient schedules an appointment for allergy testing. d. Patient takes ibuprofen (Advil) to control throat pain.

C Eosinophilic esophagitis is frequently associated with environmental allergens, so allergy testing is used to determine possible triggers. Corticosteroid therapy may be prescribed, but the medication will be swallowed, not inhaled. Milk is a frequent trigger for attacks. NSAIDs are not used for eosinophilic esophagitis. DIF: Cognitive Level: Apply (application) REF: 907 TOP: Nursing Process: Evaluation

33. A critically ill patient develops incontinence of watery stools. What action will be best for the nurse to take to prevent complications associated with ongoing incontinence? a. Insert a rectal tube. b. Use incontinence briefs. c. Implement fecal management system. d. Assist the patient to a bedside commode or to the bathroom at frequent intervals.

C Fecal management systems are designed to contain loose stools and can be in place for as long as 4 weeks without causing damage to the rectum or anal sphincters. Rectal tubes are avoided because of possible damage to the anal sphincter and ulceration of the rectal mucosa. Incontinence briefs may be helpful but, unless they are changed frequently, are likely to increase the risk for skin breakdown. A critically ill patient will not be able to use the commode or bathroom. DIF: Cognitive Level: Application REF: 1012 TOP: Nursing Process: Implementation

21. All of the following nursing actions are included in the plan of care for a patient who is malnourished. Which action is appropriate for the nurse to delegate to nursing assistive personnel (NAP)? a. Assist the patient to choose high nutrition items from the menu. b. Monitor the patient for skin breakdown over the bony prominences. c. Offer the patient the prescribed nutritional supplement between meals. d. Assess the patient's strength while ambulating the patient in the room.

C Feeding the patient and assisting with oral intake are included in NAP education and scope of practice. Assessing the patient and assisting the patient in choosing high nutrition foods require LPN/LVN- or RN-level education and scope of practice. DIF: Cognitive Level: Application REF: 928-929 OBJ: Special Questions: Delegation TOP: Nursing Process: Planning

21. The nurse is planning care for a patient who is chronically malnourished. Which action is appropriate for the nurse to delegate to unlicensed assistive personnel (UAP)? a. Assist the patient to choose high-nutrition items from the menu. b. Monitor the patient for skin breakdown over the bony prominences. c. Offer the patient the prescribed nutritional supplement between meals. d. Assess the patient's strength while ambulating the patient in the room.

C Feeding the patient and assisting with oral intake are included in UAP education and scope of practice. Assessing the patient and assisting the patient in choosing high-nutrition foods require licensed practical/vocational nurse (LPN/LVN)-or registered nurse (RN)-level education and scope of practice. DIF: Cognitive Level: Apply (application) REF: 867 OBJ: Special Questions: Delegation TOP: Nursing Process: Planning

1. A nurse assesses clients on the medical-surgical unit. Which client is at greatest risk for the development of bacterial cystitis? a. A 36-year-old female who has never been pregnant b. A 42-year-old male who is prescribed cyclophosphamide c. A 58-year-old female who is not taking estrogen replacement d. A 77-year-old male with mild congestive heart failure

C Females at any age are more susceptible to cystitis than men because of the shorter urethra in women. Postmenopausal women who are not on hormone replacement therapy are at increased risk for bacterial cystitis because of changes in the cells of the urethra and vagina. The middle-aged woman who has never been pregnant would not have a risk potential as high as the older woman who is not using hormone replacement therapy. DIF: Understanding/Comprehension REF: 1367 KEY: Cystitis| health screening

8. The nurse will anticipate teaching a patient with nephrotic syndrome who develops flank pain about treatment with a. antibiotics. c. anticoagulants. b. antifungals. d. antihypertensives.

C Flank pain in a patient with nephrotic syndrome suggests a renal vein thrombosis and anticoagulation is needed. Antibiotics are used to treat a patient with flank pain caused by pyelonephritis. Fungal pyelonephritis is uncommon and is treated with antifungals. Antihypertensives are used if the patient has high blood pressure. DIF: Cognitive Level: Apply (application) REF: 1051 TOP: Nursing Process: Planning

39. Which information noted by the nurse when caring for a patient with a bladder infection is most important to report to the health care provider? a. Dysuria b. Hematuria c. Left-sided flank pain d. Temperature 100.1° F

C Flank pain indicates that the patient may have developed pyelonephritis as a complication of the bladder infection. The other clinical manifestations are consistent with a lower urinary tract infection (UTI). DIF: Cognitive Level: Application REF: 1132-1133 OBJ: Special Questions: Prioritization TOP: Nursing Process: Assessment

6. A patient who has gastroesophageal reflux disease (GERD) is experiencing increasing discomfort. Which patient statement to the nurse indicates that additional teaching about GERD is needed? a. "I take antacids between meals and at bedtime each night." b. "I sleep with the head of the bed elevated on 4-inch blocks." c. "I eat small meals during the day and have a bedtime snack." d. "I quit smoking several years ago, but I still chew a lot of gum."

C GERD is exacerbated by eating late at night, and the nurse should plan to teach the patient to avoid eating at bedtime. The other patient actions are appropriate to control symptoms of GERD. DIF: Cognitive Level: Apply (application) REF: 902 TOP: Nursing Process: Evaluation

9. Which patient choice for a snack 3 hours before bedtime indicates that the nurse's teaching about gastroesophageal reflux disease (GERD) has been effective? a. Chocolate pudding c. Cherry gelatin with fruit b. Glass of low-fat milk d. Peanut butter and jelly sandwich

C Gelatin and fruit are low fat and will not decrease lower esophageal sphincter (LES) pressure. Foods such as chocolate are avoided because they lower LES pressure. Milk products increase gastric acid secretion. High-fat foods such as peanut butter decrease both gastric emptying and LES pressure. DIF: Cognitive Level: Apply (application) REF: 902 TOP: Nursing Process: Evaluation

5. The nurse determines that further instruction is needed for a patient with interstitial cystitis when the patient says which of the following? a. "I should stop having coffee and orange juice for breakfast." b. "I will buy calcium glycerophosphate (Prelief) at the pharmacy." c. "I will start taking high potency multiple vitamins every morning." d. "I should call the doctor about increased bladder pain or foul urine."

C High-potency multiple vitamins may irritate the bladder and increase symptoms. The other patient statements indicate good understanding of the teaching. DIF: Cognitive Level: Apply (application) REF: 1041 TOP: Nursing Process: Evaluation

16. A 68-yr-old female patient admitted to the hospital with dehydration is confused and incontinent of urine. Which nursing action should be included in the plan of care? a. Restrict fluids between meals and after the evening meal. b. Insert an indwelling catheter until the symptoms have resolved. c. Assist the patient to the bathroom every 2 hours during the day. d. Apply absorbent adult incontinence diapers and pads over the bed linens.

C In older or confused patients, incontinence may be avoided by using scheduled toileting times. Indwelling catheters increase the risk for urinary tract infection. Incontinent pads and diapers increase the risk for skin breakdown. Restricting fluids is not appropriate in a patient with dehydration. DIF: Cognitive Level: Apply (application) REF: 1059 TOP: Nursing Process: Planning

2. A nursing student is studying nutritional problems and learns that kwashiorkor is distinguished from marasmus with which finding? a. Deficit of calories b. Lack of all nutrients c. Specific lack of protein d. Unknown cause of malnutrition

C Kwashiorkor is a lack of protein when total calories are adequate. Marasmus is a caloric malnutrition. DIF: Remembering/Knowledge REF: 1236 KEY: Nutritional disorders| nutritional assessment

13. A nurse is teaching a client about magnesium hydroxide with aluminum hydroxide (Maalox). What instruction is most appropriate? a. "Aspirin must be avoided." b. "Do not worry about black stools." c. "Report diarrhea to your provider." d. "Take 1 hour before meals."

C Maalox can cause hypermagnesemia, which causes diarrhea, so the client should be taught to report this to the provider. Aspirin is avoided with bismuth sulfate (Pepto-Bismol). Black stools can be caused by Pepto-Bismol. Maalox should be taken after meals. DIF: Understanding/Comprehension REF: 1129 KEY: Gastrointestinal disorders| antacids| patient education

8. A patient's capillary blood glucose level is 120 mg/dL 6 hours after the nurse initiated a parenteral nutrition (PN) infusion. The appropriate action by the nurse is to a. obtain a venous blood glucose specimen. b. slow the infusion rate of the PN infusion. c. recheck the capillary blood glucose level in 4 to 6 hours. d. contact the health care provider for infusion rate changes.

C Mild hyperglycemia is expected during the first few days after PN is started and requires ongoing monitoring. Because the glucose elevation is small and expected, infusion rate changes are not needed. There is no need to obtain a venous specimen for comparison. Slowing the rate of the infusion is beyond the nurse's scope of practice and will decrease the patient's nutritional intake. DIF: Cognitive Level: Apply (application) REF: 870 TOP: Nursing Process: Implementation

8. After 6 hours of parenteral nutrition (PN) infusion, the nurse checks a patient's capillary blood glucose level and finds it to be 120 mg/dL. The most appropriate action by the nurse is to a. obtain a venous blood glucose specimen. b. slow the infusion rate of the PN infusion. c. recheck the capillary blood glucose in 4 hours. d. notify the health care provider of the glucose level.

C Mild hyperglycemia is expected during the first few days after PN is started and requires ongoing monitoring. Because the glucose elevation is small and expected, notification of the health care provider is not necessary. There is no need to obtain a venous specimen for comparison. Slowing the rate of the infusion is beyond the nurse's scope of practice and will decrease the patient's nutritional intake. DIF: Cognitive Level: Application REF: 938-939 TOP: Nursing Process: Implementation

14. For which client would the nurse suggest the provider not prescribe misoprostol (Cytotec)? a. Client taking antacids b. Client taking antibiotics c. Client who is pregnant d. Client over 65 years of age

C Misoprostol can cause abortion, so pregnant women should not take this drug. The other clients have no contraindications to taking misoprostol. DIF: Remembering/Knowledge REF: 1129 KEY: Gastrointestinal disorders| prostaglandin analogues

19. A patient admitted to the hospital with pneumonia has a history of functional urinary incontinence. Which nursing action will be included in the plan of care? a. Demonstrate the use of the Credé maneuver. b. Teach exercises to strengthen the pelvic floor. c. Place a bedside commode close to the patient's bed. d. Use an ultrasound scanner to check postvoiding residuals.

C Modifications in the environment make it easier to avoid functional incontinence. Checking for residual urine and performing the Credé maneuver are interventions for overflow incontinence. Kegel exercises are useful for stress incontinence. DIF: Cognitive Level: Apply (application) REF: 1059 TOP: Nursing Process: Planning

14. Which nursing action included in the plan of care for a patient who is being admitted for bariatric surgery can the nurse delegate to nursing assistive personnel (NAP)? a. Demonstrate use of the incentive spirometer. b. Plan methods for bathing and turning the patient. c. Assist with IV insertion by holding adipose tissue out of the way. d. Develop strategies to provide privacy and decrease embarrassment.

C NAP can assist with IV placement by assisting with patient positioning or holding skinfolds aside. Planning for care and patient teaching require RN level education and scope of practice. DIF: Cognitive Level: Application REF: 957-958 OBJ: Special Questions: Delegation TOP: Nursing Process: Planning

7. A client is 1 day postoperative after having Zenker's diverticula removed. The client has a nasogastric (NG) tube to suction, and for the last 4 hours there has been no drainage. There are no specific care orders for the NG tube in place. What action by the nurse is most appropriate? a. Document the findings as normal. b. Irrigate the NG tube with sterile saline. c.. Notify the surgeon about this finding. d. Remove and reinsert the NG tube.

C NG tubes placed during surgery should not be irrigated or moved unless prescribed by the surgeon. The nurse should notify the surgeon about this finding. Documentation is important, but this finding is not normal. DIF: Applying/Application REF: 1123 KEY: Gastrointestinal disorders| postoperative nursing| nasogastric tubes| communication

9. A nurse cares for a client with colon cancer who has a new colostomy. The client states, "I think it would be helpful to talk with someone who has had a similar experience." How should the nurse respond? a. "I have a good friend with a colostomy who would be willing to talk with you." b. "The enterostomal therapist will be able to answer all of your questions." c. "I will make a referral to the United Ostomy Associations of America." d. "You'll find that most people with colostomies don't want to talk about them."

C Nurses need to become familiar with community-based resources to better assist clients. The local chapter of the United Ostomy Associations of America has resources for clients and their families, including Ostomates (specially trained visitors who also have ostomies). The nurse should not suggest that the client speak with a personal contact of the nurse. Although the enterostomal therapist is an expert in ostomy care, talking with him or her is not the same as talking with someone who actually has had a colostomy. The nurse should not brush aside the client's request by saying that most people with colostomies do not want to talk about them. Many people are willing to share their ostomy experience in the hope of helping others. DIF: Applying/Application REF: 1157 KEY: Colorectal cancer| ostomy care| coping| support

11. A client has gastroesophageal reflux disease (GERD). The provider prescribes a proton pump inhibitor. About what medication should the nurse anticipate teaching the client? a. Famotidine (Pepcid) b. Magnesium hydroxide (Maalox) c. Omeprazole (Prilosec) d. Ranitidine (Zantac)

C Omeprazole is a proton pump inhibitor used in the treatment of GERD. Famotidine and ranitidine are histamine blockers. Maalox is an antacid. DIF: Remembering/Knowledge REF: 1113 KEY: Gastrointestinal disorders| proton pump inhibitors| patient education

35. A young adult been admitted to the emergency department with nausea and vomiting. Which action could the RN delegate to unlicensed assistive personnel (UAP)? a. Auscultate the bowel sounds. c. Assist the patient with oral care. b. Assess for signs of dehydration. d. Ask the patient about the nausea.

C Oral care is included in UAP education and scope of practice. The other actions are all assessments that require more education and a higher scope of nursing practice. DIF: Cognitive Level: Apply (application) REF: 907 OBJ: Special Questions: Delegation TOP: Nursing Process: Planning

10. A patient's renal calculus is analyzed as being very high in uric acid. To prevent recurrence of stones, the nurse teaches the patient to avoid eating a. milk and dairy products. b. legumes and dried fruits. c. organ meats and sardines. d. spinach, chocolate, and tea.

C Organ meats and fish such as sardines increase purine levels and uric acid. Spinach, chocolate, and tomatoes should be avoided in patients who have oxalate stones. Milk, dairy products, legumes, and dried fruits may increase the incidence of calcium-containing stones. DIF: Cognitive Level: Application REF: 1139 TOP: Nursing Process: Implementation

3. Which information will the nurse include when teaching the patient with a urinary tract infection (UTI) about the use of phenazopyridine (Pyridium)? a. Take the medication for at least 7 days. b. Use sunscreen while taking the Pyridium. c. The urine may turn a reddish-orange color. d. Use the Pyridium before sexual intercourse.

C Patients should be taught that Pyridium will color the urine deep orange. Urinary analgesics should only be needed for a few days until the prescribed antibiotics decrease the bacterial count. Taking Pyridium before intercourse will not be helpful in reducing the risk for UTI. Pyridium does not cause photosensitivity. DIF: Cognitive Level: Application REF: 1124-1125 TOP: Nursing Process: Implementation

3. Which information will the nurse include when teaching the patient with a urinary tract infection (UTI) about the use of phenazopyridine? a. Take phenazopyridine for at least 7 days. b. Phenazopyridine may cause photosensitivity c. Phenazopyridine may change the urine color d. Take phenazopyridine before sexual intercourse.

C Patients should be taught that phenazopyridine will color the urine deep orange. Urinary analgesics should only be needed for a few days until the prescribed antibiotics decrease the bacterial count. Phenazopyridine does not cause photosensitivity. Taking phenazopyridine before intercourse will not be helpful in reducing the risk for UTI. DIF: Cognitive Level: Apply (application) REF: 1036 TOP: Nursing Process: Implementation

20. The home health nurse teaches a patient with a neurogenic bladder how to use intermittent catheterization for bladder emptying. Which patient statement indicates that the teaching has been effective? a. "I will buy seven new catheters weekly and use a new one every day." b. "I will use a sterile catheter and gloves for each time I self-catheterize." c. "I will clean the catheter carefully before and after each catheterization." d. "I will take prophylactic antibiotics to prevent any urinary tract infections."

C Patients who are at home can use a clean technique for intermittent self-catheterization and change the catheter every 7 days. There is no need to use a new catheter every day, to use sterile catheters, or to take prophylactic antibiotics. DIF: Cognitive Level: Apply (application) REF: 1063 TOP: Nursing Process: Evaluation

4. After teaching a client with bacterial cystitis who is prescribed phenazopyridine (Pyridium), the nurse assesses the client's understanding. Which statement made by the client indicates a correct understanding of the teaching? a. "I will not take this drug with food or milk." b. "If I think I am pregnant, I will stop the drug." c. "An orange color in my urine should not alarm me." d. "I will drink two glasses of cranberry juice daily."

C Phenazopyridine discolors urine, most commonly to a deep reddish orange. Many clients think they have blood in their urine when they see this. In addition, the urine can permanently stain clothing. Phenazopyridine is safe to take if the client is pregnant. There are no dietary restrictions or needs while taking this medication. DIF: Applying/Application REF: 1372 KEY: Cystitis| medication safety

26. A nurse provides phone triage to a pregnant client. The client states, "I am experiencing a burning pain when I urinate." How should the nurse respond? a. "This means labor will start soon. Prepare to go to the hospital." b. "You probably have a urinary tract infection. Drink more cranberry juice." c. "Make an appointment with your provider to have your infection treated." d. "Your pelvic wall is weakening. Pelvic muscle exercises should help."

C Pregnant clients with a urinary tract infection require prompt and aggressive treatment because cystitis can lead to acute pyelonephritis during pregnancy. The nurse should encourage the client to make an appointment and have the infection treated. Burning pain when urinating does not indicate the start of labor or weakening of pelvic muscles. DIF: Applying/Application REF: 1369 KEY: Cystitis

37. Four hours after a bowel resection, a 74-yr-old male patient with a nasogastric tube to suction complains of nausea and abdominal distention. The first action by the nurse should be to a. auscultate for hypotonic bowel sounds. b. notify the patient's health care provider. c. check for tube placement and reposition it. d. remove the tube and replace it with a new one.

C Repositioning the tube will frequently facilitate drainage. Because this is a common occurrence, it is not appropriate to notify the health care provider unless other interventions do not resolve the problem. Information about the presence or absence of bowel sounds will not be helpful in improving drainage. Removing the tube and replacing it are unnecessarily traumatic to the patient, so that would only be done if the tube was completely occluded. DIF: Cognitive Level: Analyze (analysis) REF: 939 OBJ: Special Questions: Prioritization TOP: Nursing Process: Implementation

38. Following an exploratory laparotomy and bowel resection, a patient who has a nasogastric tube to suction complains of nausea and stomach distention. The first action by the nurse should be to a. auscultate for hypotonic bowel sounds. b. notify the patient's health care provider. c. reposition the tube and check for placement. d. remove the tube and replace it with a new one.

C Repositioning the tube will frequently facilitate drainage. Because this is a common occurrence, it is not appropriate to notify the health care provider. Information about the presence or absence of bowel tones will not be helpful in improving drainage. Removing the tube and replacing it are unnecessarily traumatic to the patient. DIF: Cognitive Level: Application REF: 1016-1017 OBJ: Special Questions: Prioritization TOP: Nursing Process: Implementation

32. When performing an admission assessment for a patient with abdominal pain, the nurse palpates the left lower quadrant and the patient complains of right lower quadrant pain. The nurse will document this as a. rebound pain. b. Cullen sign. c. Rovsing sign. d. McBurney point.

C Rovsing sign occurs when palpation of the left lower quadrant causes pain in the right lower quadrant. McBurney point, rebound pain, and Cullen sign are used to describe other aspects of the abdominal assessment. DIF: Cognitive Level: Knowledge REF: 1020 TOP: Nursing Process: Assessment

5. When the health care provider in the outpatient clinic is considering prescribing sibutramine (Meridia) for a patient, which patient information is most important for the nurse to discuss with the provider? a. The patient has a permanent pacemaker. b. The patient's goal is to lose 90 lb (41 kg). c. The patient's blood pressure is usually 135-145/85-95. d. The patient used fenfluramine (Pondimin) in the past for weight loss.

C Side effects of sibutramine (Meridia) include hypertension. A permanent pacemaker and a history of fenfluramine use are not contraindications for sibutramine. Sibutramine is prescribed for patients who have large weight loss goals. DIF: Cognitive Level: Application REF: 953-954 TOP: Nursing Process: Assessment

7. Which finding by the nurse for a patient admitted with glomerulonephritis indicates that treatment has been effective? a. The patient denies pain with voiding. b. The urine dipstick is negative for nitrites. c. Peripheral and periorbital edema is resolved. d. The antistreptolysin-O (ASO) titer is decreased.

C Since edema is a common clinical manifestation of glomerulonephritis, resolution of the edema indicates that the prescribed therapies have been effective. Antibodies to streptococcus will persist after a streptococcal infection. Nitrites will be negative and the patient will not experience dysuria since the patient does not have a urinary tract infection. DIF: Cognitive Level: Application REF: 1131-1133 TOP: Nursing Process: Evaluation

7. The nurse caring for clients with gastrointestinal disorders should understand that which category best describes the mechanism of action of sucralfate (Carafate)? a. Gastric acid inhibitor b. Histamine receptor blocker c. Mucosal barrier fortifier d. Proton pump inhibitor

C Sucralfate is a mucosal barrier fortifier (protector). It is not a gastric acid inhibitor, a histamine receptor blocker, or a proton pump inhibitor. DIF: Remembering/Knowledge REF: 1135 KEY: Gastrointestinal disorders| mucosal barrier fortifier

29. The health care provider prescribes antacids and sucralfate (Carafate) for treatment of a patient's peptic ulcer. The nurse will teach the patient to take a. antacids 30 minutes before the sucralfate. b. sucralfate at bedtime and antacids before meals. c. antacids after eating and sucralfate 30 minutes before eating. d. sucralfate and antacids together 30 minutes before each meal.

C Sucralfate is most effective when the pH is low and should not be given with or soon after antacid. Antacids are most effective when taken after eating. Administration of sucralfate 30 minutes before eating and antacids just after eating will ensure that both drugs can be most effective. The other regimens will decrease the effectiveness of the medications. DIF: Cognitive Level: Comprehension REF: 992 TOP: Nursing Process: Implementation

37. A patient had a cystectomy with an ileal conduit yesterday. Which new assessment data is most important for the nurse to communicate to the health care provider? a. Cloudy appearing urine c. Heart rate 102 beats/minute b. Hypotonic bowel sounds d. Continuous stoma drainage

C Tachycardia may indicate infection, hemorrhage, or hypovolemia, which are all serious complications of this surgery. The urine from an ileal conduit normally contains mucus and is cloudy. Hypotonic bowel sounds are expected after bowel surgery. Continuous drainage of urine from the stoma is normal. DIF: Cognitive Level: Analyze (analysis) REF: 1064 OBJ: Special Questions: Prioritization TOP: Nursing Process: Implementation

37. Which assessment finding for a patient who has had a cystectomy with an ileal conduit the previous day is most important for the nurse to communicate to the physician? a. Cloudy appearing urine b. Hypotonic bowel sounds c. Heart rate 102 beats/minute d. Continuous drainage from stoma

C Tachycardia may indicate infection, hemorrhage, or hypovolemia, which are all serious complications of this surgery. The urine from an ileal conduit normally contains mucus and is cloudy. Hypotonic bowel sounds are expected after bowel surgery. Continuous drainage of urine from the stoma is normal. DIF: Cognitive Level: Application REF: 1157 | 1159-1160 | 1158-1159 OBJ: Special Questions: Prioritization TOP: Nursing Process: Implementation

22. A 68-yr-old male patient who has bladder cancer had a cystectomy with creation of an Indiana pouch. Which topic will be included in patient teaching? a. Application of ostomy appliances b. Barrier products for skin protection c. Catheterization technique and schedule d. Analgesic use before emptying the pouch

C The Indiana pouch enables the patient to self-catheterize every 4 to 6 hours. There is no need for an ostomy device or barrier products. Catheterization of the pouch is not painful. DIF: Cognitive Level: Apply (application) REF: 1065 TOP: Nursing Process: Implementation

38. A 26-year-old patient with a history of polycystic kidney disease is admitted to the surgical unit after having knee surgery. Which of the routine postoperative orders is most important for the nurse to discuss with the health care provider? a. Infuse 5% dextrose in normal saline at 75 mL/hr. b. Order regular diet after patient is awake and alert. c. Give ketorolac (Toradol) 10 mg PO PRN for pain. d. Obtain blood urea nitrogen (BUN), creatinine, and electrolytes in 2 hours.

C The NSAIDs should be avoided in patients with decreased renal function because nephrotoxicity is a potential adverse effect. The other orders do not need any clarification or change. DIF: Cognitive Level: Application REF: 1142-1143 OBJ: Special Questions: Prioritization TOP: Nursing Process: Implementation

35. The nurse observes unlicensed assistive personnel (UAP) taking the following actions when caring for a patient with a urethral catheter. Which action requires that the nurse intervene? a. Taping the catheter to the skin on the patient's upper inner thigh b. Cleaning around the patient's urinary meatus with soap and water c. Disconnecting the catheter from the drainage tube to obtain a specimen d. Using an alcohol-based gel hand cleaner before performing catheter care

C The catheter should not be disconnected from the drainage tube because this increases the risk for urinary tract infection. The other actions are appropriate and do not require any intervention. DIF: Cognitive Level: Apply (application) REF: 1059 OBJ: Special Questions: Delegation TOP: Nursing Process: Implementation

9. A client has a nasogastric (NG) tube. What action by the nursing student requires the registered nurse to intervene? a.. Checking tube placement every 4 to 8 hours b. Monitoring and documenting drainage from the NG tube c. Pinning the tube to the gown so the client cannot turn the head d. Providing oral care every 4 to 8 hours

C The client should be able to turn his or her head to prevent pulling the tube out with movement. The other actions are appropriate. DIF: Applying/Application REF: 1123 KEY: Gastrointestinal disorders| nasogastric tube| supervision

19. A nurse teaches a client who is at risk for colon cancer. Which dietary recommendation should the nurse teach this client? a. "Eat low-fiber and low-residual foods." b. "White rice and bread are easier to digest." c. "Add vegetables such as broccoli and cauliflower to your new diet." d. "Foods high in animal fat help to protect the intestinal mucosa."

C The client should be taught to modify his or her diet to decrease animal fat and refined carbohydrates. The client should also increase high-fiber foods and Brassica vegetables, including broccoli and cauliflower, which help to protect the intestinal mucosa from colon cancer. DIF: Applying/Application REF: 1149 KEY: Colorectal cancer| nutritional requirements

13. A nurse works on the surgical unit. After receiving the hand-off report, which client should the nurse see first? a. Client who underwent diverticula removal with a pulse of 106/min b. Client who had esophageal dilation and is attempting first postprocedure oral intake c. Client who had an esophagectomy with a respiratory rate of 32/min d. Client who underwent hernia repair, reporting incisional pain of 7/10

C The client who had an esophagectomy has a respiratory rate of 32/min, which is an early sign of sepsis; this client needs to be assessed first. The client who underwent diverticula removal has a pulse that is out of the normal range (106/min), but not terribly so. The client reporting pain needs pain medication, but the client with the elevated respiratory rate needs investigation first. The nurse should see the client who had esophageal dilation prior to and during the first attempt at oral feedings, but this can wait until the other clients are cared for. DIF: Analyzing/Analysis REF: 1121 KEY: Gastrointestinal disorders| sepsis| nursing assessment

17. A patient has peptic ulcer disease that has been associated with Helicobacter pylori. About which medications will the nurse plan to teach the patient? a. Sucralfate (Carafate), nystatin, and bismuth (Pepto-Bismol) b. Metoclopramide (Reglan), bethanechol (Urecholine), and promethazine c. Amoxicillin (Amoxil), clarithromycin (Biaxin), and omeprazole (Prilosec) d. Famotidine (Pepcid), magnesium hydroxide (Mylanta), and pantoprazole (Protonix)

C The drugs used in triple drug therapy include a proton pump inhibitor such as omeprazole and the antibiotics amoxicillin and clarithromycin. The other combinations listed are not included in the protocol for H. pylori infection. DIF: Cognitive Level: Apply (application) REF: 903 TOP: Nursing Process: Planning

6. A patient is receiving tube feedings through a percutaneous endoscopic gastrostomy (PEG). Which action will the nurse include in the plan of care? a. Keep the patient positioned on the left side. b. Obtain a daily x-ray to verify tube placement. c. Check the gastric residual volume every 4 to 6 hours. d. Avoid giving bolus tube feedings through the PEG tube.

C The gastric residual volume is assessed every 4 to 6 hours to decrease the risk for aspiration. The patient does not need to be positioned on the left side. An x-ray is obtained immediately after placement of the PEG tube to check position, but daily x-rays are not needed. Bolus feedings can be administered through a PEG tube. DIF: Cognitive Level: Application REF: 933-934 TOP: Nursing Process: Planning

21. After ureterolithotomy, a patient has a left ureteral catheter and a urethral catheter in place. Which action will the nurse include in the plan of care? a. Provide teaching about home care for both catheters. b. Apply continuous steady tension to the ureteral catheter. c. Call the health care provider if the ureteral catheter output drops suddenly. d. Clamp the ureteral catheter off when output from the urethral catheter stops.

C The health care provider should be notified if the ureteral catheter output decreases because obstruction of this catheter may result in an increase in pressure in the renal pelvis. Tension on the ureteral catheter should be avoided to prevent catheter displacement. To avoid pressure in the renal pelvis, the catheter is not clamped. Because the patient is not usually discharged with a ureteral catheter in place, patient teaching about both catheters is not needed. DIF: Cognitive Level: Apply (application) REF: 1062 TOP: Nursing Process: Planning

46. A 76-yr-old patient with obstipation has a fecal impaction and is incontinent of liquid stool. Which action should the nurse take first? a. Administer bulk-forming laxatives. b. Assist the patient to sit on the toilet. c. Manually remove the impacted stool. d. Increase the patient's oral fluid intake.

C The initial action with a fecal impaction is manual disimpaction. The other actions will be used to prevent future constipation and impactions. DIF: Cognitive Level: Analyze (analysis) REF: 933 OBJ: Special Questions: Prioritization TOP: Nursing Process: Implementation

38. A 19-yr-old woman is brought to the emergency department with a knife handle protruding from her abdomen. During the initial assessment of the patient, the nurse should a. remove the knife and assess the wound. b. determine the presence of Rovsing sign. c. check for circulation and tissue perfusion. d. insert a urinary catheter and assess for hematuria.

C The initial assessment is focused on determining whether the patient has hypovolemic shock. The knife should not be removed until the patient is in surgery, where bleeding can be controlled. Rovsing sign is assessed in the patient with suspected appendicitis. Assessment for bladder trauma is not part of the initial assessment. DIF: Cognitive Level: Apply (application) REF: 939 TOP: Nursing Process: Assessment

13. After successfully losing 1 lb weekly for several months, a patient at the clinic has not lost any weight for the past month. The nurse should first a. review the diet and exercise guidelines with the patient. b. instruct the patient to weigh and record weights weekly. c. ask the patient whether there have been any changes in exercise or diet patterns. d. discuss the possibility that the patient has reached a temporary weight loss plateau.

C The initial nursing action should be assessment of any reason for the change in weight loss. The other actions may be needed, but further assessment is required before any interventions are planned or implemented. DIF: Cognitive Level: Analyze (analysis) REF: 883 OBJ: Special Questions: Prioritization TOP: Nursing Process: Assessment

15. A patient who has been successfully losing 1 lb weekly for several months is weighed at the clinic and has not lost any weight for the last month. The nurse should first a. review the diet and exercise guidelines with the patient. b. instruct the patient to weigh weekly and record the weights. c. ask the patient whether there have been any changes in exercise or diet patterns. d. discuss the possibility that the patient has reached a temporary weight loss plateau.

C The initial nursing action should be assessment of any reason for the change in weight loss. The other actions may be needed, but further assessment is required before any interventions are planned or implemented. DIF: Cognitive Level: Application REF: 949-952 OBJ: Special Questions: Prioritization TOP: Nursing Process: Assessment

28. Which assessment finding for a patient who has just been admitted with acute pyelonephritis is most important for the nurse to report to the health care provider? a. Foul-smelling urine b. Complaint of flank pain c. Blood pressure 88/45 mm Hg d. Temperature 100.1° F (57.8° C)

C The low blood pressure indicates that urosepsis and septic shock may be occurring and should be immediately reported. The other findings are typical of pyelonephritis. DIF: Cognitive Level: Application REF: 1126 OBJ: Special Questions: Prioritization TOP: Nursing Process: Assessment

25. A nurse cares for a client with urinary incontinence. The client states, "I am so embarrassed. My bladder leaks like a young child's bladder." How should the nurse respond? a. "I understand how you feel. I would be mortified." b. "Incontinence pads will minimize leaks in public." c. "I can teach you strategies to help control your incontinence." d. "More women experience incontinence than you might think."

C The nurse should accept and acknowledge the client's concerns, and assist the client to learn techniques that will allow control of urinary incontinence. The nurse should not diminish the client's concerns with the use of pads or stating statistics about the occurrence of incontinence. DIF: Applying/Application REF: 1376 KEY: Urinary incontinence| psychosocial response| coping

17. A nurse attempted to assist a morbidly obese client back to bed and had immediate pain in the lower back. What action by the nurse is most appropriate? a. Ask another nurse to help next time. b. Demand better equipment to use. c. Fill out and file a variance report. d. Refuse to assist the client again.

C The nurse should complete a variance report per agency policy. Asking another nurse to help and requesting better equipment are both good ideas, but the nurse may have an injury that needs care. It would be unethical to refuse to care for this client again. DIF: Applying/Application REF: 1246 KEY: Nutritional disorders| obesity| variance report

5. A client having a tube feeding begins vomiting. What action by the nurse is most appropriate? a. Administer an antiemetic. b. Check the client's gastric residual. c. Hold the feeding until the nausea subsides. d. Reduce the rate of the tube feeding by half.

C The nurse should hold the feeding until the nausea and vomiting have subsided and consult with the provider on the rate at which to restart the feeding. Giving an antiemetic is not appropriate. After vomiting, a gastric residual will not be accurate. The nurse should not continue to feed the client while he or she is vomiting. DIF: Applying/Application REF: 1244 KEY: Nutritional disorders| tube feedings

2. A 67-year-old patient tells the nurse, "I have problems with constipation now that I am older, so I use a suppository every morning." Which action should the nurse take first? a. Encourage the patient to increase oral fluid intake. b. Inform the patient that a daily bowel movement is unnecessary. c. Assess the patient about individual risk factors for constipation. d. Suggest that the patient increase dietary intake of high-fiber foods.

C The nurse's initial action should be further assessment of the patient for risk factors for constipation and for usual bowel pattern. The other actions may be appropriate but will be based on the assessment. DIF: Cognitive Level: Application REF: 1012-1013 TOP: Nursing Process: Implementation

27. At his first postoperative checkup appointment after a gastrojejunostomy (Billroth II), a patient reports that dizziness, weakness, and palpitations occur about 20 minutes after each meal. The nurse will teach the patient to a. increase the amount of fluid with meals. b. eat foods that are higher in carbohydrates. c. lie down for about 30 minutes after eating. d. drink sugared fluids or eat candy after meals.

C The patient is experiencing symptoms of dumping syndrome, which may be reduced by lying down after eating. Increasing fluid intake and choosing high carbohydrate foods will increase the risk for dumping syndrome. Having a sweet drink or hard candy will correct the hypoglycemia that is associated with dumping syndrome but will not prevent dumping syndrome. DIF: Cognitive Level: Apply (application) REF: 918 TOP: Nursing Process: Implementation

25. When preparing a female patient with bladder cancer for intravesical chemotherapy, the nurse will teach about a. premedicating to prevent nausea. b. obtaining wigs and scarves to wear. c. emptying the bladder before the medication. d. maintaining oral care during the treatments.

C The patient will be asked to empty the bladder before instillation of the chemotherapy. Systemic side effects are not usually experienced with intravesical chemotherapy. DIF: Cognitive Level: Apply (application) REF: 1055 TOP: Nursing Process: Implementation

9. A patient is admitted to the hospital with new onset nephrotic syndrome. Which assessment data will the nurse expect to find related to this illness? a. Poor skin turgor b. High urine ketones c. Recent weight gain d. Low blood pressure

C The patient with a nephrotic syndrome will have weight gain associated with edema. Hypertension is a clinical manifestation of nephrotic syndrome. Skin turgor is normal because of the edema. Urine protein is high. DIF: Cognitive Level: Comprehension REF: 1132-1134 TOP: Nursing Process: Assessment

7. A patient is admitted to the hospital for evaluation of right lower quadrant abdominal pain with nausea and vomiting. Which action should the nurse take? a. Check for rebound tenderness. b. Assist the patient to cough and deep breathe. c. Apply an ice pack to the right lower quadrant. d. Encourage the patient to take sips of clear liquids.

C The patient's clinical manifestations are consistent with appendicitis, and application of an ice pack will decrease inflammation at the area. The patient should be NPO in case immediate surgery is needed. Checking for rebound tenderness frequently is unnecessary and uncomfortable for the patient. The patient will need to know how to cough and deep breathe postoperatively, but coughing will increase pain at this time. DIF: Cognitive Level: Application REF: 1020-1021 TOP: Nursing Process: Implementation

7. A young adult patient is admitted to the hospital for evaluation of right lower quadrant abdominal pain with nausea and vomiting. Which action should the nurse take? a. Assist the patient to cough and deep breathe. b. Palpate the abdomen for rebound tenderness. c. Suggest the patient lie on the side, flexing the right leg. d. Encourage the patient to sip clear, noncarbonated liquids.

C The patient's clinical manifestations are consistent with appendicitis. Lying still with the right leg flexed is often the most comfortable position. Checking for rebound tenderness frequently is unnecessary and uncomfortable for the patient. The patient should be NPO in case immediate surgery is needed. The patient will need to know how to cough and deep breathe postoperatively, but coughing will increase pain at this time. DIF: Cognitive Level: Apply (application) REF: 942 TOP: Nursing Process: Implementation

14. When assessing a 30-year-old man who complains of a feeling of incomplete bladder emptying and a split, spraying urine stream, the nurse asks about a history of a. bladder infection. b. recent kidney trauma. c. gonococcal urethritis. d. benign prostatic hyperplasia.

C The patient's clinical manifestations are consistent with urethral strictures, a possible complication of gonococcal urethritis. These symptoms are not consistent with benign prostatic hyperplasia, kidney trauma, or bladder infection. DIF: Cognitive Level: Application REF: 1141 TOP: Nursing Process: Assessment

37. Which patient should the nurse assess first after receiving change-of-shift report? a. A patient with nausea who has a dose of metoclopramide (Reglan) due b. A patient who is crying after receiving a diagnosis of esophageal cancer c. A patient with esophageal varices who has a blood pressure of 92/58 mm Hg d. A patient admitted yesterday with gastrointestinal (GI) bleeding who has melena

C The patient's history and blood pressure indicate possible hemodynamic instability caused by GI bleeding. The data about the other patients do not indicate acutely life-threatening complications. DIF: Cognitive Level: Analyze (analysis) REF: 922 OBJ: Special Questions: Prioritization | Special Questions: Multiple Patients TOP: Nursing Process: Assessment

37. After receiving change-of-shift report, which patient should the nurse assess first? a. A patient who was admitted yesterday with gastrointestinal (GI) bleeding and has melena b. A patient who is crying after receiving a diagnosis of esophageal cancer c. A patient with esophageal varices who has a blood pressure of 96/54 mm Hg d. A patient with nausea who has a dose of metoclopramide (Reglan) scheduled

C The patient's history and blood pressure indicate possible hemodynamic instability caused by GI bleeding. The data about the other patients do not indicate acutely life-threatening complications. DIF: Cognitive Level: Application REF: 980-982 OBJ: Special Questions: Prioritization TOP: Nursing Process: Assessment

41. A patient with acute gastrointestinal (GI) bleeding is receiving normal saline IV at a rate of 500 mL/hr. Which assessment finding obtained by the nurse is most important to communicate immediately to the health care provider? a. The patient's blood pressure (BP) has increased to 142/94 mm Hg. b. The nasogastric (NG) suction is returning coffee-ground material. c. The patient's lungs have crackles audible to the midline. d. The bowel sounds are very hyperactive in all four quadrants.

C The patient's lung sounds indicate that pulmonary edema may be developing as a result of the rapid infusion of IV fluid and that the fluid infusion rate should be slowed. The return of coffee-ground material in an NG tube is expected for a patient with upper GI bleeding. The BP is slightly elevated but would not be an indication to contact the health care provider immediately. Hyperactive bowel sounds are common when a patient has GI bleeding. DIF: Cognitive Level: Application REF: 984 OBJ: Special Questions: Prioritization TOP: Nursing Process: Assessment

7. A nurse is caring for four clients receiving enteral tube feedings. Which client should the nurse see first? a. Client with a blood glucose level of 138 mg/dL b. Client with foul-smelling diarrhea c. Client with a potassium level of 2.6 mEq/L d. Client with a sodium level of 138 mEq/L

C The potassium is critically low, perhaps due to hyperglycemia-induced hyperosmolarity. The nurse should see this client first. The blood glucose reading is high, but not extreme. The sodium is normal. The client with the diarrhea should be seen last to avoid cross-contamination. DIF: Applying/Application REF: 1244 KEY: Nutritional disorders| tube feedings| electrolyte imbalances

48. A 72-yr-old male patient with dehydration caused by an exacerbation of ulcerative colitis is receiving 5% dextrose in normal saline at 125 mL/hour. Which assessment finding by the nurse is most important to report to the health care provider? a. Patient has not voided for the last 4 hours. b. Skin is dry with poor turgor on all extremities. c. Crackles are heard halfway up the posterior chest. d. Patient has had 5 loose stools over the previous 6 hours.

C The presence of crackles in an older patient receiving IV fluids at a high rate suggests volume overload and a need to reduce the rate of the IV infusion. The other data will also be reported but are consistent with the patient's age and diagnosis and do not require a change in the prescribed treatment. DIF: Cognitive Level: Analyze (analysis) REF: 948 OBJ: Special Questions: Prioritization TOP: Nursing Process: Assessment

6. A nurse assessing a client with colorectal cancer auscultates high-pitched bowel sounds and notes the presence of visible peristaltic waves. Which action should the nurse take? a. Ask if the client is experiencing pain in the right shoulder. b. Perform a rectal examination and assess for polyps. c. Contact the provider and recommend computed tomography. d. Administer a laxative to increase bowel movement activity.

C The presence of visible peristaltic waves, accompanied by high-pitched or tingling bowel sounds, is indicative of partial obstruction caused by the tumor. The nurse should contact the provider with these results and recommend a computed tomography scan for further diagnostic testing. This assessment finding is not associated with right shoulder pain; peritonitis and cholecystitis are associated with referred pain to the right shoulder. The registered nurse is not qualified to complete a rectal examination for polyps, and laxatives would not help this client. DIF: Applying/Application REF: 1151 KEY: Colorectal cancer| intestinal obstruction

36. A patient is admitted to the emergency department with severe abdominal pain and rebound tenderness. Vital signs include temperature 102°F (38.3°C), pulse 120 beats/min, respirations 32 breaths/min, and blood pressure (BP) 82/54 mm Hg. Which prescribed intervention should the nurse implement first? a. Administer IV ketorolac 15 mg for pain relief. b. Draw a blood sample for a complete blood count (CBC). c. Infuse a liter of lactated Ringer's solution over 30 minutes. d. Send the patient for an abdominal computed tomography (CT) scan.

C The priority for this patient is to treat the patient's hypovolemic shock with fluid infusion. The other actions should be implemented after starting the fluid infusion. DIF: Cognitive Level: Analyze (analysis) REF: 939 OBJ: Special Questions: Prioritization TOP: Nursing Process: Implementation

10. What information will the nurse include for an overweight 35-yr-old woman who is starting a weight-loss plan? a. Weigh yourself at the same time every morning and evening. b. Stick to a 600- to 800-calorie diet for the most rapid weight loss. c. Low carbohydrate diets lead to rapid weight loss but are difficult to maintain. d. Weighing all foods on a scale is necessary to choose appropriate portion sizes.

C The restrictive nature of fad diets makes the weight loss achieved by the patient more difficult to maintain. Portion size can be estimated in other ways besides weighing. Severely calorie-restricted diets are not necessary for patients in the overweight category and need to be closely supervised. Patients should weigh weekly rather than daily. DIF: Cognitive Level: Apply (application) REF: 882 TOP: Nursing Process: Implementation

12. What specific information will the nurse include in patient teaching for an overweight patient who is starting a weight loss plan? a. Weigh yourself at the same time every morning. b. Start dieting with a 600- to 800-calorie diet for rapid weight loss. c. Low carbohydrate diets lead to rapid weight loss but are difficult to maintain. d. Weighing all foods on a scale is necessary to choose appropriate portion sizes.

C The restrictive nature of fad diets makes the weight loss achieved by the patient more difficult to maintain. Portion size can be estimated in other ways besides weighing. Severely calorie-restricted diets are not necessary for patients in the overweight category of obesity and need to be closely supervised. Patients should weigh weekly rather than daily. DIF: Cognitive Level: Application REF: 951-953 TOP: Nursing Process: Implementation

3. A client with peptic ulcer disease is in the emergency department and reports the pain has gotten much worse over the last several days. The client's blood pressure when lying down was 122/80 mm Hg and when standing was 98/52 mm Hg. What action by the nurse is most appropriate? a. Administer ibuprofen (Motrin). b. Call the Rapid Response Team. c. Start a large-bore IV with normal saline. d. Tell the client to remain lying down.

C This client has orthostatic changes to the blood pressure, indicating fluid volume loss. The nurse should start a large-bore IV with isotonic solution. Ibuprofen will exacerbate the ulcer. The Rapid Response Team is not needed at this point. The client should be put on safety precautions, which includes staying in bed, but this is not the priority. DIF: Applying/Application REF: 1136 KEY: Gastrointestinal disorders| fluid imbalances| nursing assessment

14. The following data relate to an older client who is 2 hours postoperative after an esophagogastrostomy: Physical Assessment Vital Signs Physician Orders Skin dry Urine output 20 mL/hr NG tube patent with 100 mL brown drainage/hr Restless Pulse: 128 beats/min Blood pressure: 88/50 mm Hg Respiratory rate: 20 on ventilator Cardiac output: 2.1 L/min Oxygen saturation: 99% Normal saline at 75 mL/hr Morphine sulfate 2 mg IV push every 1 hr PRN pain Intake and output every hour Vital signs every hour Vancomycin (Vancocin) 1 g IV every 8 hr What action by the nurse is best? a. Administer the prescribed pain medication. b. Consult the surgeon about a different antibiotic. c. Consult the surgeon about increased IV fluids. d. Have respiratory therapy reduce the respiratory rate.

C This client's vital signs, cardiac output, dry skin, and urine output indicate hypovolemia or possible hypotension resulting from pressure placed on the posterior heart during surgery. The client needs more fluids, so the nurse should consult with the surgeon about increasing the fluid intake. The client may be restless as a result of the hypotension and may not need pain medication at this time. There is no reason to request a different antibiotic. The respiratory rate does not need to be adjusted. DIF: Analyzing/Analysis REF: 1121 KEY: Gastrointestinal disorders| fluid and electrolyte imbalances| communication

45. After change-of-shift report, which patient should the nurse assess first? a. A 42-yr-old patient who has acute gastritis and ongoing epigastric pain b. A 70-yr-old patient with a hiatal hernia who experiences frequent heartburn c. A 60-yr-old patient with nausea and vomiting who has dry mucosa and lethargy d. 53-yr-old patient who has dumping syndrome after a recent partial gastrectomy

C This patient is at high risk for problems such as aspiration, dehydration, and fluid and electrolyte disturbances. The other patients will also need to be assessed, but the information about them indicates symptoms that are typical for their diagnoses and are not life threatening. DIF: Cognitive Level: Apply (application) REF: 910 OBJ: Special Questions: Prioritization | Special Questions: Multiple Patients TOP: Nursing Process: Assessment

11. A 58-yr-old woman who was recently diagnosed with esophageal cancer tells the nurse, "I do not feel ready to die yet." Which response by the nurse is most appropriate? a. "You may have quite a few years still left to live." b. "Thinking about dying will only make you feel worse." c. "Having this new diagnosis must be very hard for you." d. "It is important that you be realistic about your prognosis."

C This response is open ended and will encourage the patient to further discuss feelings of anxiety or sadness about the diagnosis. Patients with esophageal cancer have a low survival rate, so the response "You may have quite a few years still left to live" is misleading. The response beginning, "Thinking about dying" indicates that the nurse is not open to discussing the patient's fears of dying. The response beginning, "It is important that you be realistic" discourages the patient from feeling hopeful, which is important to patients with any life-threatening diagnosis. DIF: Cognitive Level: Analyze (analysis) REF: 907 TOP: Nursing Process: Implementation

11. A 62-year-old patient who has been diagnosed with esophageal cancer tells the nurse, "I know that my chances are not very good, but I do not feel ready to die yet." Which response by the nurse is most appropriate? a. "You may have quite a few years still left to live." b. "Thinking about dying will only make you feel worse." c. "Having this new diagnosis must be very hard for you." d. "It is important that you be realistic about your prognosis."

C This response is open-ended and will encourage the patient to further discuss feelings of anxiety or sadness about the diagnosis. Patients with esophageal cancer have only a low survival rate, so the response "You may have quite a few years still left to live" is misleading. The response beginning, "Thinking about dying" indicates that the nurse is not open to discussing the patient's fears of dying. And the response beginning, "It is important that you be realistic," discourages the patient from feeling hopeful, which is important to patients with any life-threatening diagnosis. DIF: Cognitive Level: Application REF: 977-979 TOP: Nursing Process: Implementation

19. The nurse notes that the peripheral parenteral nutrition (PN) bag has only 20 mL left and a new PN bag has not yet arrived from the pharmacy. Which intervention is the priority? a. Monitor the patient's capillary blood glucose until a new PN bag is hung b. Flush the peripheral line with saline and wait until the new PN bag is available c. Infuse 5% dextrose in water until the new PN bag is delivered from the pharmacy d. Decrease the rate of the current PN infusion to 10 mL/hr until the new bag arrives

C To prevent hypoglycemia, the nurse should infuse a 5% dextrose solution until the next PN bag can be started. Decreasing the rate of the ordered PN infusion is beyond the nurse's scope of practice. Flushing the line and then waiting for the next bag may lead to hypoglycemia. Monitoring the capillary blood glucose is appropriate but is not the priority. DIF: Cognitive Level: Application REF: 938 OBJ: Special Questions: Prioritization TOP: Nursing Process: Implementation

12. A patient is being admitted for bariatric surgery. Which nursing action can the nurse delegate to unlicensed assistive personnel (UAP)? a. Demonstrate use of the incentive spirometer. b. Plan methods for turning the patient after surgery. c. Assist with IV insertion by holding adipose tissue out of the way. d. Develop strategies to provide privacy and decrease embarrassment.

C UAP can assist with IV placement by assisting with patient positioning or holding skinfolds aside. Planning for care and patient teaching require registered nurse (RN)-level education and scope of practice. DIF: Cognitive Level: Apply (application) REF: 885 OBJ: Special Questions: Delegation TOP: Nursing Process: Planning

41. Which care activity for a patient with a paralytic ileus is appropriate for the registered nurse (RN) to delegate to unlicensed assistive personnel (UAP)? a. Auscultation for bowel sounds b. Nasogastric (NG) tube irrigation c. Applying petroleum jelly to the lips d. Assessment of the nares for irritation

C UAP education and scope of practice include patient hygiene such as oral care. The other actions require education and scope of practice appropriate to the RN. DIF: Cognitive Level: Apply (application) REF: 960 OBJ: Special Questions: Delegation TOP: Nursing Process: Implementation

20. A 22-year-old patient with Escherichia coli O157:H7 food poisoning is admitted to the hospital with bloody diarrhea and dehydration. All of the following orders are received. Which order will the nurse question? a. Infuse lactated Ringer's solution at 250 mL/hr. b. Monitor blood urea nitrogen and creatinine daily. c. Administer loperamide (Imodium) after each stool. d. Provide a clear liquid diet and progress diet as tolerated.

C Use of antidiarrheal agents is avoided with this type of food poisoning. The other orders are appropriate. DIF: Cognitive Level: Application REF: 1001-1002 TOP: Nursing Process: Implementation

20. An adult with Escherichia coli O157:H7 food poisoning is admitted to the hospital with bloody diarrhea and dehydration. Which prescribed action will the nurse question? a. Infuse lactated Ringer's solution at 250 mL/hr. b. Monitor blood urea nitrogen and creatinine daily. c. Administer loperamide (Imodium) after each stool. d. Provide a clear liquid diet and progress diet as tolerated.

C Use of antidiarrheal agents is avoided with this type of food poisoning. The other orders are appropriate. DIF: Cognitive Level: Apply (application) REF: 926 TOP: Nursing Process: Implementation

16. The nurse is caring for a patient on the first postoperative day after a Roux-en- gastric bypass procedure. Which assessment finding should be reported immediately to the surgeon? a. Bilateral crackles audible at both lung bases b. Redness, irritation, and skin breakdown in skinfolds c. Emesis of bile-colored fluid past the nasogastric (NG) tube d. Use of patient-controlled analgesia (PCA) several times an hour for pain

C Vomiting with an NG tube in place indicates that the NG tube needs to be repositioned by the surgeon to avoid putting stress on the gastric sutures. The nurse should implement actions to decrease skin irritation and have the patient cough and deep breathe, but these do not indicate a need for rapid notification of the surgeon. Frequent PCA use after bariatric surgery is expected. DIF: Cognitive Level: Analyze (analysis) REF: 886 OBJ: Special Questions: Prioritization TOP: Nursing Process: Assessment

10. A client tells the nurse about losing weight and regaining it multiple times. Besides eating and exercising habits, for what additional data should the nurse assess as the priority? a. Economic ability to join a gym b. Food allergies and intolerances c. Psychosocial influences on weight d. Reasons for wanting to lose weight

C While all topics might be important to assess, people who lose and gain weight in cycles often are depressed or have poor self-esteem, which has a negative effect on weight-loss efforts. The nurse assesses the client's psychosocial status as the priority. DIF: Applying/Application REF: 1247 KEY: Nutritional disorders| psychosocial response| nursing assessment

12. A patient who has an exacerbation of ulcerative colitis is having 15 to 20 stools daily and has excoriated perianal skin. Which patient behavior indicates that teaching regarding maintenance of skin integrity has been effective? a. The patient uses incontinence briefs to contain loose stools. b. The patient asks for antidiarrheal medication after each stool. c. The patient uses witch hazel compresses to decrease anal irritation. d. The patient cleans the perianal area with soap and water after each stool.

C Witch hazel compresses are suggested to reduce anal irritation and discomfort. Incontinence briefs may trap diarrhea and increase the incidence of skin breakdown. Antidiarrheal medications are not given 15 to 20 times a day. The perianal area should be washed with plain water after each stool. DIF: Cognitive Level: Application REF: 1029 TOP: Nursing Process: Evaluation

5. A nurse working with a client who has possible gastritis assesses the client's gastrointestinal system. Which findings indicate a chronic condition as opposed to acute gastritis? (Select all that apply.) a. Anorexia b. Dyspepsia c. Intolerance of fatty foods d. Pernicious anemia e. Nausea and vomiting

C, D Intolerance of fatty or spicy foods and pernicious anemia are signs of chronic gastritis. Anorexia and nausea/vomiting can be seen in both conditions. Dyspepsia is seen in acute gastritis. DIF: Remembering/Knowledge REF: 1128 KEY: Gastrointestinal disorders| nursing assessment

6. A nurse assesses a male client with an abdominal hernia. Which abdominal hernias are correctly paired with their physiologic processes? (Select all that apply.) a. Indirect inguinal hernia - An enlarged plug of fat eventually pulls the peritoneum and often the bladder into a sac b. Femoral hernia - A peritoneum sac pushes downward and may descend into the scrotum c. Direct inguinal hernia - A peritoneum sac passes through a weak point in the abdominal wall d. Ventral hernia - Results from inadequate healing of an incision e. Incarcerated hernia - Contents of the hernia sac cannot be reduced back into the abdominal cavity

C, D, E A direct inguinal hernia occurs when a peritoneum sac passes through a weak point in the abdominal wall. A ventral hernia results from inadequate healing of an incision. An incarcerated hernia cannot be reduced or placed back into the abdominal cavity. An indirect inguinal hernia is a sac formed from the peritoneum that contains a portion of the intestine and pushes downward at an angle into the inguinal canal. An indirect inguinal hernia often descends into the scrotum. A femoral hernia protrudes through the femoral ring and, as the clot enlarges, pulls the peritoneum and often the urinary bladder into the sac. DIF: Applying/Application REF: 1146 KEY: Herniation

2. A nurse has delegated feeding a client to an unlicensed assistive personnel (UAP). What actions does the nurse include in the directions to the UAP? (Select all that apply.) a. Allow 30 minutes for eating so food doesn't get spoiled. b. Assess the client's mouth while providing premeal oral care. c. Ensure warm and cold items stay at appropriate temperatures. d. Remove bedpans, soiled linens, and other unpleasant items. e. Sit with the client, making the atmosphere more relaxed.

C, D, E The UAP should make sure food items remain at the appropriate temperatures for maximum palatability. Removing items such as bedpans, urinals, or soiled linens helps make the atmosphere more conducive to eating. The UAP should sit, not stand, next to the client to promote a relaxing experience. The client, especially older clients who tend to eat more slowly, should not be rushed. Assessment is done by the nurse. DIF: Understanding/Comprehension REF: 1240 KEY: Nutritional disorders| nutrition| unlicensed assistive personnel (UAP)

19. The nurse is providing preoperative teaching for a patient scheduled for an abdominal-perineal resection. Which information will the nurse include? a. The patient will begin sitting in a chair at the bedside on the first postoperative day. b. IV antibiotics will be started at least 24 hours before surgery to reduce the bowel bacteria. c. An additional surgery in 8 to 12 weeks will be used to create an ileal-anal reservoir. d. The site where the stoma will be located will be marked on the abdomen preoperatively.

D A WOCN should select the site where the ostomy will be positioned and mark the abdomen preoperatively. The site should be within the rectus muscle, on a flat surface, and in a place that the patient is able to see. A permanent colostomy is created with this surgery. Sitting is contraindicated after an abdominal-perineal resection. Oral antibiotics (rather than IV antibiotics) are given to reduce colonic and rectal bacteria. DIF: Cognitive Level: Apply (application) REF: 960 TOP: Nursing Process: Implementation

19. During preoperative preparation for a patient scheduled for an abdominal-perineal resection, the nurse will a. give IV antibiotics starting 24 hours before surgery to reduce the number of bowel bacteria. b. teach the patient that activities such as sitting at the bedside will be started the first postoperative day. c. instruct the patient that another surgery in 8 to 12 weeks will be used to create an ileal-anal reservoir. d. administer polyethylene glycol lavage solution (GoLYTELY) to ensure that the bowel is empty before the surgery.

D A bowel-cleansing agent is used to empty the bowel before surgery to reduce the risk for infection. Oral antibiotics (rather than IV antibiotics) are given to reduce colonic and rectal bacteria. Sitting is contraindicated after an abdominal-perineal resection. A permanent colostomy is created with this surgery. DIF: Cognitive Level: Application REF: 1036-1038 TOP: Nursing Process: Implementation

14. A nurse assesses a client with a mechanical bowel obstruction who reports intermittent abdominal pain. An hour later the client reports constant abdominal pain. Which action should the nurse take next? a. Administer intravenous opioid medications. b. Position the client with knees to chest. c. Insert a nasogastric tube for decompression. d. Assess the client's bowel sounds.

D A change in the nature and timing of abdominal pain in a client with a bowel obstruction can signal peritonitis or perforation. The nurse should immediately check for rebound tenderness and the absence of bowel sounds. The nurse should not medicate the client until the provider has been notified of the change in his or her condition. The nurse may help the client to the knee-chest position for comfort, but this is not the priority action. The nurse need not insert a nasogastric tube for decompression. DIF: Applying/Application REF: 1157 KEY: Intestinal obstruction| pain management

11. To prevent the recurrence of renal calculi, the nurse teaches the patient to a. use a filter to strain all urine. b. avoid dietary sources of calcium. c. drink diuretic fluids such as coffee. d. have 2000 to 3000 mL of fluid a day.

D A fluid intake of 2000 to 3000 mL daily is recommended to help flush out minerals before stones can form. Avoidance of calcium is not usually recommended for patients with renal calculi. Coffee tends to increase stone recurrence. There is no need for a patient to strain all urine routinely after a stone has passed, and this will not prevent stones. DIF: Cognitive Level: Application REF: 1138-1139 TOP: Nursing Process: Implementation

3. In teaching a patient who has chronic constipation about the use of psyllium (Metamucil), which information will the nurse include? a. Absorption of fat-soluble vitamins may be reduced by fiber-containing laxatives. b. Dietary sources of fiber should be eliminated to prevent excessive gas formation. c. Use of this type of laxative to prevent constipation does not cause adverse effects. d. Large amounts of fluid should be taken to prevent impaction or bowel obstruction.

D A high fluid intake is needed when patients are using bulk-forming laxatives to avoid worsening constipation. Although bulk-forming laxatives are generally safe, the nurse should emphasize the possibility of constipation or obstipation if inadequate fluid intake occurs. Although increased gas formation is likely to occur with increased dietary fiber, the patient should gradually increase dietary fiber and eventually may not need the psyllium. Fat-soluble vitamin absorption is blocked by stool softeners and lubricants, not by bulk-forming laxatives. DIF: Cognitive Level: Application REF: 1012-1014 TOP: Nursing Process: Implementation

3. A patient who has chronic constipation asks the nurse about the use of psyllium (Metamucil). Which information will the nurse include in the response? a. Absorption of fat-soluble vitamins may be reduced by fiber-containing laxatives. b. Dietary sources of fiber should be eliminated to prevent excessive gas formation. c. Use of this type of laxative to prevent constipation does not cause adverse effects. d. Large amounts of fluid should be taken to prevent impaction or bowel obstruction.

D A high fluid intake is needed when patients are using bulk-forming laxatives to avoid worsening constipation. Although bulk-forming laxatives are generally safe, the nurse should emphasize the possibility of constipation or obstipation if inadequate fluid intake occurs. Although increased gas formation is likely to occur with increased dietary fiber, the patient should gradually increase dietary fiber and eventually may not need the psyllium. Fat-soluble vitamin absorption is blocked by stool softeners and lubricants, not by bulk-forming laxatives. DIF: Cognitive Level: Apply (application) REF: 935 TOP: Nursing Process: Implementation

27. Which patient education will the nurse provide before discharge for a patient who has had a herniorrhaphy to repair an incarcerated inguinal hernia? a. Encourage the patient to cough. b. Provide sitz baths several times daily. c. Avoid use of acetaminophen (Tylenol) for pain. d. Apply a scrotal support and ice to reduce swelling.

D A scrotal support and ice are used to reduce edema and pain. Coughing will increase pressure on the incision. Sitz baths will not relieve pain and would not be of use after this surgery. Acetaminophen can be used for postoperative pain. DIF: Cognitive Level: Application REF: 1049-1050 TOP: Nursing Process: Implementation

26. A 40-yr-old male patient has had a herniorrhaphy to repair an incarcerated inguinal hernia. Which patient teaching will the nurse provide before discharge? a. Soak in sitz baths several times each day. b. Cough 5 times each hour for the next 48 hours. c. Avoid use of acetaminophen (Tylenol) for pain. d. Apply a scrotal support and ice to reduce swelling.

D A scrotal support and ice are used to reduce edema and pain. Coughing will increase pressure on the incision. Sitz baths will not relieve pain and would not be of use after this surgery. Acetaminophen can be used for postoperative pain. DIF: Cognitive Level: Apply (application) REF: 965 TOP: Nursing Process: Implementation

21. A nurse is weighing and measuring a client with severe kyphosis. What is the best method to obtain this client's height? a. Add the trunk and leg measurements. b. Ask the client how tall he or she is. c. Estimate by measuring clothing. d. Use knee-height calipers.

D A sliding blade knee-height caliper is used to obtain the height of a client who cannot stand upright, such as those with kyphosis or lower extremity contractures. The other methods will not yield accurate data. DIF: Remembering/Knowledge REF: 1236 KEY: Nutritional assessment

17. A patient has a large bowel obstruction that occurred as a result of diverticulosis. When assessing the patient, the nurse will plan to monitor for a. referred back pain. b. metabolic alkalosis. c. projectile vomiting. d. abdominal distention.

D Abdominal distention is seen in lower intestinal obstruction. Metabolic alkalosis is common in high intestinal obstruction because of the loss of HCl acid from vomiting. Referred back pain is not a common clinical manifestation of intestinal obstruction. Bile-colored vomit is associated with higher intestinal obstruction. DIF: Cognitive Level: Application REF: 1031-1033 TOP: Nursing Process: Assessment

17. A patient with diverticulosis has a large bowel obstruction. The nurse will monitor for a. referred back pain. c. projectile vomiting. b. metabolic alkalosis. d. abdominal distention.

D Abdominal distention is seen in lower intestinal obstruction. Referred back pain is not a common clinical manifestation of intestinal obstruction. Metabolic alkalosis is common in high intestinal obstruction because of the loss of HCl acid from vomiting. Projectile vomiting is associated with higher intestinal obstruction. DIF: Cognitive Level: Apply (application) REF: 939 TOP: Nursing Process: Assessment

12. After hiatal hernia repair surgery, a client is on IV pantoprazole (Protonix). The client asks the nurse why this medication is given since there is no history of ulcers. What response by the nurse is best? a. "Bacteria can often cause ulcers." b. "This operation often causes ulcers." c. "The medication keeps your blood pH low." d. "It prevents stress-related ulcers."

D After surgery, anti-ulcer medications such as pantoprazole are often given to prevent stress-related ulcers. The other responses are incorrect. DIF: Understanding/Comprehension REF: 1113 KEY: Gastrointestinal disorders| proton pump inhibitors| patient education

13. An emergency room nurse cares for a client who has been shot in the abdomen and is hemorrhaging heavily. Which action should the nurse take first? a. Send a blood sample for a type and crossmatch. b. Insert a large intravenous line for fluid resuscitation. c. Obtain the heart rate and blood pressure. d. Assess and maintain a patent airway.

D All of the options are important nursing actions in the care of a trauma client. However, airway always comes first. The client must have a patent airway, or other interventions will not be helpful. DIF: Applying/Application REF: 1162 KEY: GI trauma| emergency nursing

10. A client is scheduled for a total gastrectomy for gastric cancer. What preoperative laboratory result should the nurse report to the surgeon immediately? a. Albumin: 2.1 g/dL b. Hematocrit: 28% c. Hemoglobin: 8.1 mg/dL d. International normalized ratio (INR): 4.2

D An INR as high as 4.2 poses a serious risk of bleeding during the operation and should be reported. The albumin is low and is an expected finding. The hematocrit and hemoglobin are also low, but this is expected in gastric cancer. DIF: Applying/Application REF: 1140 KEY: Gastrointestinal disorders| cancer| laboratory values| communication

33. Which assessment finding in a patient who had a total gastrectomy 12 hours previously is most important to report to the health care provider? a. Absent bowel sounds b. Scant nasogastric (NG) tube drainage c. Complaints of incisional pain d. Temperature 102.1° F (38.9° C)

D An elevation in temperature may indicate leakage at the anastomosis, which may require return to surgery or keeping the patient NPO. The other findings are expected in the immediate postoperative period for patients who have this surgery. DIF: Cognitive Level: Application REF: 1001 TOP: Nursing Process: Assessment

27. Which breakfast choice indicates a patient's good understanding of information about a diet for celiac disease? a. Oatmeal with nonfat milk c. Bagel with low-fat cream cheese b. wheat toast with butter d. Corn tortilla with scrambled eggs

D Avoidance of gluten-containing foods is the only treatment for celiac disease. Corn does not contain gluten, but oatmeal and wheat do. DIF: Cognitive Level: Apply (application) REF: 967 TOP: Nursing Process: Evaluation

10. A patient who recently has been experiencing frequent heartburn is seen in the clinic. The nurse will anticipate teaching the patient about a. barium swallow. b. radionuclide tests. c. endoscopy procedures. d. proton pump inhibitors.

D Because diagnostic testing for heartburn that is probably caused by gastroesophageal reflux disease (GERD) is expensive and uncomfortable, proton pump inhibitors are frequently used for a short period as the first step in the diagnosis of GERD. The other tests may be used but are not usually the first step in diagnosis. DIF: Cognitive Level: Application REF: 972-974 TOP: Nursing Process: Planning

10. The nurse will anticipate teaching a patient experiencing frequent heartburn about a. a barium swallow. c. endoscopy procedures. b. radionuclide tests. d. proton pump inhibitors.

D Because diagnostic testing for heartburn that is probably caused by gastroesophageal reflux disease (GERD) is expensive and uncomfortable, proton pump inhibitors are frequently used for a short period as the first step in the diagnosis of GERD. The other tests may be used but are not usually the first step in diagnosis. DIF: Cognitive Level: Apply (application) REF: 902 TOP: Nursing Process: Planning

7. Which finding for a patient admitted with glomerulonephritis indicates to the nurse that treatment has been effective? a. The patient denies pain with voiding. b. The urine dipstick is negative for nitrites. c. The antistreptolysin-O (ASO) titer has decreased. d. The periorbital and peripheral edema are resolved.

D Because edema is a common clinical manifestation of glomerulonephritis, resolution of the edema indicates that the prescribed therapies have been effective. Nitrites will be negative and the patient will not experience dysuria because the patient does not have a urinary tract infection. Antibodies to streptococcus will persist after a streptococcal infection. DIF: Cognitive Level: Apply (application) REF: 1042 TOP: Nursing Process: Evaluation

33. Which assessment finding is most important to report to the health care provider regarding a patient who has had left-sided extracorporeal shock wave lithotripsy? a. Blood in urine c. Left flank discomfort b. Left flank bruising d. Decreased urine output

D Because lithotripsy breaks the stone into small sand, which could cause obstruction, it is important to report a drop in urine output. Left flank pain, bruising, and hematuria are common after lithotripsy. DIF: Cognitive Level: Analyze (analysis) REF: 1048 OBJ: Special Questions: Prioritization TOP: Nursing Process: Assessment

33. When the nurse is caring for a patient who has had left-sided extracorporeal shock wave lithotripsy, which assessment finding is most important to report to the health care provider? a. Blood in urine b. Left flank pain c. Left flank bruising d. Drop in urine output

D Because lithotripsy breaks the stone into small sand, which could cause obstruction, it is important to report a drop in urine output. Left flank pain, bruising, and hematuria are common after lithotripsy. DIF: Cognitive Level: Application REF: 1138-1139 OBJ: Special Questions: Prioritization TOP: Nursing Process: Assessment

9. A patient with protein calorie malnutrition who has had abdominal surgery is receiving parenteral nutrition (PN). Which assessment information obtained by the nurse is the best indicator that the patient is receiving adequate nutrition? a. Blood glucose is 110 mg/dL. b. Serum albumin level is 3.5 mg/dL. c. Fluid intake and output are balanced. d. Surgical incision is healing normally.

D Because poor wound healing is a possible complication of malnutrition for this patient, normal healing of the incision is an indicator of the effectiveness of the PN in providing adequate nutrition. Blood glucose is monitored to prevent the complications of hyperglycemia and hypoglycemia, but it does not indicate that the patient's nutrition is adequate. The intake and output will be monitored but do not indicate that the PN is effective. The albumin level is in the low-normal range but does not reflect adequate caloric intake, which is also important for the patient. DIF: Cognitive Level: Application REF: 928-930 TOP: Nursing Process: Evaluation

40. All of the following orders are received for a patient who has vomited 1500 mL of bright red blood. Which order will the nurse implement first? a. Insert a nasogastric (NG) tube and connect to suction. b. Administer intravenous (IV) famotidine (Pepcid) 40 mg. c. Draw blood for typing and crossmatching. d. Infuse 1000 mL of lactated Ringer's solution.

D Because the patient has vomited a large amount of blood, correction of hypovolemia and prevention of hypovolemic shock are the priorities. The other actions also are important to implement quickly but are not the highest priorities. DIF: Cognitive Level: Application REF: 981-982 | 984 OBJ: Special Questions: Prioritization TOP: Nursing Process: Implementation

41. The nurse who is interviewing a 40-year-old obtains information about the following patient problems. Which information is most important to communicate to the health care provider? a. The patient had an appendectomy at age 17. b. The patient smokes a pack/day of cigarettes. c. The patient has a history of frequent constipation. d. The patient has recently noticed blood in the stools.

D Blood in the stools is a possible clinical manifestation of colorectal cancer and requires further testing by the health care provider. The other patient information also will be communicated to the health care provider, but does not indicate an urgent need for further testing or intervention. DIF: Cognitive Level: Application REF: 1035-1036 OBJ: Special Questions: Prioritization TOP: Nursing Process: Assessment

3. A patient who is receiving chemotherapy develops a Candida albicans oral infection. The nurse will anticipate the need for a. hydrogen peroxide rinses. b. the use of antiviral agents. c. referral to a dentist for professional tooth cleaning. d. administration of nystatin (Mycostatin) oral tablets.

D Candida albicans is treated with an antifungal such as nystatin. Oral saltwater rinses may be used but will not cure the infection. Antiviral agents are used for viral infections such as herpes simplex. Referral to a dentist is indicated for gingivitis but not for Candida infection. DIF: Cognitive Level: Application REF: 968 TOP: Nursing Process: Planning

20. A client is receiving total parenteral nutrition (TPN). What action by the nurse is most important? a. Assessing blood glucose as directed b. Changing the IV dressing each day c. Checking the TPN with another nurse d. Performing appropriate hand hygiene

D Clients on TPN are at high risk for infection. The nurse performs appropriate hand hygiene as a priority intervention. Checking blood glucose is also an important measure, but preventing infection takes priority. The IV dressing is changed every 48 to 72 hours. TPN does not need to be double-checked with another nurse. DIF: Applying/Application REF: 1245 KEY: Nutritional disorders| parenteral nutrition| infection control

5. A nurse assesses clients at a community health center. Which client is at highest risk for the development of colorectal cancer? a. A 37-year-old who drinks eight cups of coffee daily b. A 44-year-old with irritable bowel syndrome (IBS) c. A 60-year-old lawyer who works 65 hours per week d. A 72-year-old who eats fast food frequently

D Colon cancer is rare before the age of 40, but its incidence increases rapidly with advancing age. Fast food tends to be high in fat and low in fiber, increasing the risk for colon cancer. Coffee intake, IBS, and a heavy workload do not increase the risk for colon cancer. DIF: Applying/Application REF: 1149 KEY: Colorectal cancer| health screening

4. Which finding by the nurse will be most helpful in determining whether a 67-yr-old patient with benign prostatic hyperplasia has an upper urinary tract infection (UTI)? a. Bladder distention c. Suprapubic discomfort b. Foul-smelling urine d. Costovertebral tenderness

D Costovertebral tenderness is characteristic of pyelonephritis. Bladder distention, foul-smelling urine, and suprapubic discomfort are characteristic of a lower UTI and are likely to be present if the patient also has an upper UTI. DIF: Cognitive Level: Analyze (analysis) REF: 1038 TOP: Nursing Process: Assessment

4. A 72-year-old who has benign prostatic hyperplasia is admitted to the hospital with chills, fever, and vomiting. Which finding by the nurse will be most helpful in determining whether the patient has an upper urinary tract infection (UTI)? a. Suprapubic pain b. Bladder distention c. Foul-smelling urine d. Costovertebral tenderness

D Costovertebral tenderness is characteristic of pyelonephritis. The other symptoms are characteristic of lower UTI and are likely to be present if the patient also has an upper UTI. DIF: Cognitive Level: Application REF: 1128 TOP: Nursing Process: Assessment

31. A patient with Crohn's disease has megaloblastic anemia. The nurse will anticipate teaching the patient about the ongoing need for a. oral ferrous sulfate tablets. b. regular blood transfusions. c. iron dextran (Imferon) infusion. d. cobalamin (B12) nasal spray or injections.

D Crohn's disease frequently affects the ileum, where absorption of cobalamin occurs, and it must be administered regularly by nasal spray or IM to correct the anemia. Iron deficiency does not cause megaloblastic anemia. The patient may need occasional transfusions but not regularly scheduled transfusions. DIF: Cognitive Level: Application REF: 1024-1025 | 1028 TOP: Nursing Process: Planning

30. The nurse will plan to teach a patient with Crohn's disease who has megaloblastic anemia about the need for a. iron dextran infusions b. oral ferrous sulfate tablets. c. routine blood transfusions. d. cobalamin (B12) supplements.

D Crohn's disease frequently affects the ileum, where absorption of cobalamin occurs. Cobalamin must be administered regularly by nasal spray or IM to correct the anemia. Iron deficiency does not cause megaloblastic anemia. The patient may need occasional transfusions but not regularly scheduled transfusions. DIF: Cognitive Level: Apply (application) REF: 946 TOP: Nursing Process: Planning

12. Which information will the nurse include when teaching a patient with newly diagnosed gastroesophageal reflux disease (GERD)? a. "Peppermint tea may be helpful in reducing your symptoms." b. "You should avoid eating between meals to reduce acid secretion." c. "Vigorous physical activities may increase the incidence of reflux." d. "It will be helpful to keep the head of your bed elevated on blocks."

D Elevating the head of the bed will reduce the incidence of reflux while the patient is sleeping. Peppermint will lower LES pressure and increase the chance for reflux. Small, frequent meals are recommended to avoid abdominal distention. There is no need to make changes in physical activities because of GERD. DIF: Cognitive Level: Application REF: 975 TOP: Nursing Process: Implementation

14. When a patient is diagnosed with achalasia, the nurse will teach the patient that a. lying down after meals is recommended. b. a liquid or blenderized diet will be necessary. c. drinking fluids with meals should be avoided. d. treatment may include endoscopic procedures.

D Endoscopic and laparoscopic procedures are the most effective therapy for improving symptoms caused by achalasia. Keeping the head elevated after eating will improve esophageal emptying. A semisoft diet is recommended to improve esophageal emptying. Patients are advised to drink fluid with meals. DIF: Cognitive Level: Apply (application) REF: 908 TOP: Nursing Process: Planning

17. A 62-year-old asks the nurse for a perineal pad, stating that laughing or coughing causes leakage of urine. Which intervention is most appropriate to include in the care plan? a. Assist the patient to the bathroom q3hr. b. Place a commode at the patient's bedside. c. Demonstrate how to perform the Credé maneuver. d. Teach the patient how to perform Kegel exercises.

D Exercises to strengthen the pelvic floor muscles will help reduce stress incontinence. The Credé maneuver is used to help empty the bladder for patients with overflow incontinence. Placing the commode close to the bedside and assisting the patient to the bathroom are helpful for functional incontinence. DIF: Cognitive Level: Application REF: 1148 TOP: Nursing Process: Planning

3. Which choice from the hospital menu indicates that the patient has understood the nurse's teaching about choosing high calorie, high protein foods? a. Baked fish with applesauce b. Beef noodle soup and canned corn c. Fresh vegetables with yogurt topping d. Fried chicken with potatoes and gravy

D Foods that are high in calories include fried foods and those covered with sauces. High protein foods include meat and dairy products. The other choices are lower in calories and protein. DIF: Cognitive Level: Application REF: 929 TOP: Nursing Process: Evaluation

3. Which menu choice best indicates that the patient is implementing the nurse's suggestion to choose high-calorie, high-protein foods? a. Baked fish with applesauce b. Beef noodle soup and canned corn c. Fresh fruit salad with yogurt topping d. Fried chicken with potatoes and gravy

D Foods that are high in calories include fried foods and those covered with sauces. High-protein foods include meat and dairy products. The other choices are lower in calories and protein. DIF: Cognitive Level: Analyze (analysis) REF: 863 TOP: Nursing Process: Evaluation

19. A nurse teaches a client with functional urinary incontinence. Which statement should the nurse include in this client's teaching? a. "You must clean around your catheter daily with soap and water." b. "Wash the vaginal weights with a 10% bleach solution after each use." c. "Operations to repair your bladder are available, and you can consider these." d. "Buy slacks with elastic waistbands that are easy to pull down."

D Functional urinary incontinence occurs as the result of problems not related to the client's bladder, such as trouble ambulating or difficulty accessing the toilet. One goal is that the client will be able to manage his or her clothing independently. Elastic waistband slacks that are easy to pull down can help the client get on the toilet in time to void. The other instructions do not relate to functional urinary incontinence. DIF: Applying/Application REF: 1382 KEY: Urinary incontinence

6. The nurse is assessing a patient with gastroesophageal reflux disease (GERD) who is experiencing increasing discomfort. Which patient statement indicates that additional patient education about GERD is needed? a. "I take antacids between meals and at bedtime each night." b. "I sleep with the head of the bed elevated on 4-inch blocks." c. "I quit smoking several years ago, but I still chew a lot of gum." d. "I eat small meals throughout the day and have a bedtime snack."

D GERD is exacerbated by eating late at night, and the nurse should plan to teach the patient to avoid eating at bedtime. The other patient actions are appropriate to control symptoms of GERD. DIF: Cognitive Level: Application REF: 973-974 TOP: Nursing Process: Evaluation

9. After the nurse teaches a patient with gastroesophageal reflux disease (GERD) about recommended dietary modifications, which diet choice for a snack 2 hours before bedtime indicates that the teaching has been effective? a. Chocolate pudding b. Glass of low-fat milk c. Peanut butter sandwich d. Cherry gelatin and fruit

D Gelatin and fruit are low fat and will not decrease lower esophageal sphincter (LES) pressure. Foods like chocolate are avoided because they lower LES pressure. Milk products increase gastric acid secretion. High-fat foods such as peanut butter decrease both gastric emptying and LES pressure. DIF: Cognitive Level: Application REF: 973-974 TOP: Nursing Process: Evaluation

8. A nurse plans care for a client with overflow incontinence. Which intervention should the nurse include in this client's plan of care to assist with elimination? a. Stroke the medial aspect of the thigh. b. Use intermittent catheterization. c. Provide digital anal stimulation. d. Use the Valsalva maneuver.

D In clients with overflow incontinence, the voiding reflex arc is not intact. Mechanical pressure, such as that achieved through the Valsalva maneuver (holding the breath and bearing down as if to defecate), can initiate voiding. Stroking the medial aspect of the thigh or providing digital anal stimulation requires the reflex arc to be intact to initiate elimination. Due to the high risk for infection, intermittent catheterization should only be implemented when other interventions are not successful. DIF: Applying/Application REF: 1381 KEY: Urinary incontinence

16. A 78-year-old who has been admitted to the hospital with dehydration is confused and incontinent of urine. Which nursing action will be best to include in the plan of care? a. Apply absorbent incontinent pads. b. Restrict fluids after the evening meal. c. Insert an indwelling catheter until the symptoms have resolved. d. Assist the patient to the bathroom every 2 hours during the day.

D In older or confused patients, incontinence may be avoided by using scheduled toileting times. Indwelling catheters increase the risk for urinary tract infection (UTI). Incontinent pads increase the risk for skin breakdown. Restricting fluids is not appropriate in a patient with dehydration. DIF: Cognitive Level: Application REF: 1151-1152 TOP: Nursing Process: Planning

17. A 55-yr-old woman admitted for shoulder surgery asks the nurse for a perineal pad, stating that laughing or coughing causes leakage of urine. Which intervention is most appropriate to include in the care plan? a. Assist the patient to the bathroom q3hr. b. Place a commode at the patient's bedside. c. Demonstrate how to perform the Credé maneuver. d. Teach the patient how to perform Kegel exercises.

D Kegel exercises to strengthen the pelvic floor muscles will help reduce stress incontinence. The Credé maneuver is used to help empty the bladder for patients with overflow incontinence. Placing the commode close to the bedside and assisting the patient to the bathroom are helpful for functional incontinence. DIF: Cognitive Level: Apply (application) REF: 1056 TOP: Nursing Process: Planning

43. An 80-yr-old patient who is hospitalized with peptic ulcer disease develops new-onset auditory hallucinations. Which prescribed medication will the nurse discuss with the health care provider before administration? a. Sucralfate (Carafate) c. Omeprazole (Prilosec) b. Aluminum hydroxide d. Metoclopramide (Reglan)

D Metoclopramide can cause central nervous system side effects ranging from anxiety to hallucinations. Hallucinations are not a side effect of proton pump inhibitors, mucosal protectants, or antacids. DIF: Cognitive Level: Apply (application) REF: 895 TOP: Nursing Process: Implementation

28. A patient who requires daily use of a nonsteroidal anti-inflammatory drug (NSAID) for management of severe rheumatoid arthritis has recently developed melena. The nurse will anticipate teaching the patient about a. substitution of acetaminophen (Tylenol) for the NSAID. b. use of enteric-coated NSAIDs to reduce gastric irritation. c. reasons for using corticosteroids to treat the rheumatoid arthritis. d. the benefits of misoprostol (Cytotec) in protecting the gastrointestinal (GI) mucosa.

D Misoprostol, a prostaglandin analog, reduces acid secretion and incidence of upper GI bleeding associated with NSAID use. Enteric coating of NSAIDs does not reduce the risk for GI bleeding. Corticosteroids increase the risk for ulcer development and will not be substituted for NSAIDs for this patient. Acetaminophen will not be effective in treating the patient's rheumatoid arthritis. DIF: Cognitive Level: Application REF: 983-984 | 990-991 TOP: Nursing Process: Planning

28. A patient who requires daily use of a nonsteroidal antiinflammatory drug (NSAID) for the management of severe rheumatoid arthritis has recently developed melena. The nurse will anticipate teaching the patient about a. substitution of acetaminophen (Tylenol) for the NSAID. b. use of enteric-coated NSAIDs to reduce gastric irritation. c. reasons for using corticosteroids to treat the rheumatoid arthritis. d. misoprostol (Cytotec) to protect the gastrointestinal (GI) mucosa.

D Misoprostol, a prostaglandin analog, reduces acid secretion and the incidence of upper GI bleeding associated with NSAID use. Enteric coating of NSAIDs does not reduce the risk for GI bleeding. Corticosteroids increase the risk for ulcer development and will not be substituted for NSAIDs for this patient. Acetaminophen will not be effective in treating rheumatoid arthritis. DIF: Cognitive Level: Apply (application) REF: 914 TOP: Nursing Process: Planning

1. The nurse is developing a weight loss plan for a 21-year-old patient who is morbidly obese. Which statement by the nurse is most likely to help the patient in losing weight on the planned 1000-calorie diet? a. "It will be necessary to change lifestyle habits permanently to maintain weight loss." b. "You will decrease your risk for future health problems such as diabetes by losing weight now." c. "Most of the weight that you lose during the first weeks of dieting is water weight rather than fat." d. "You are likely to start to notice changes in how you feel with just a few weeks of diet and exercise."

D Motivation is a key factor in successful weight loss and a short-term outcome provides a higher motivation. A 21-year-old patient is unlikely to be motivated by future health problems. Telling a patient that the initial weight loss is water will be discouraging, although this may be correct. Changing lifestyle habits is necessary, but this process occurs over time and discussing this is not likely to motivate the patient. DIF: Cognitive Level: Application REF: 951 TOP: Nursing Process: Implementation

34. When interviewing a patient with abdominal pain and possible irritable bowel syndrome, which question will be most important for the nurse to ask? a. "Have you been passing a lot of gas?" b. "What foods affect your bowel patterns?" c. "Do you have any abdominal distention?" d. "How long have you had abdominal pain?"

D One criterion for the diagnosis of irritable bowel syndrome (IBS) is the presence of abdominal discomfort or pain for at least 3 months. Abdominal distention, flatulence, and food intolerance also are associated with IBS, but are not diagnostic criteria. DIF: Cognitive Level: Application REF: 1017-1018 TOP: Nursing Process: Assessment

33. Which question from the nurse would help determine if a patient's abdominal pain might indicate irritable bowel syndrome (IBS)? a. "Have you been passing a lot of gas?" b. "What foods affect your bowel patterns?" c. "Do you have any abdominal distention?" d. "How long have you had abdominal pain?"

D One criterion for the diagnosis of irritable bowel syndrome is the presence of abdominal discomfort or pain for at least 3 months. Abdominal distention, flatulence, and food intolerance are associated with IBS but are not diagnostic criteria. DIF: Cognitive Level: Apply (application) REF: 940 TOP: Nursing Process: Assessment

35. Which of these nursing actions should the RN working in the emergency department delegate to nursing assistive personnel who help with the care of a patient who has been admitted with nausea and vomiting? a. Auscultate the bowel sounds. b. Assess for signs of dehydration. c. Ask the patient what precipitated the nausea. d. Assist the patient with oral care after vomiting.

D Oral care is included in nursing assistive personnel education and scope of practice. The other actions are all assessments that require more education and a higher scope of nursing practice. DIF: Cognitive Level: Application REF: 965-966 OBJ: Special Questions: Delegation TOP: Nursing Process: Planning

27. When assessing the patient who has a lower urinary tract infection (UTI), the nurse will initially ask about a. nausea. b. flank pain. c. poor urine output. d. pain with urination.

D Pain with urination is a common symptom of a lower UTI. Urine output does not decrease, but frequency may be experienced. Flank pain and nausea are associated with an upper UTI. DIF: Cognitive Level: Application REF: 1123-1124 OBJ: Special Questions: Prioritization TOP: Nursing Process: Assessment

27. Which assessment data reported by a patient is consistent with a lower urinary tract infection (UTI)? a. Low urine output c. Nausea and vomiting b. Bilateral flank pain d. Burning on urination

D Pain with urination is a common symptom of a lower UTI. Urine output does not decrease, but frequency may be experienced. Flank pain and nausea are associated with an upper UTI. DIF: Cognitive Level: Apply (application) REF: 1035 TOP: Nursing Process: Assessment

7. A few months after bariatric surgery, a 62-year-old patient tells the nurse, "My skin is hanging in folds. I think I need cosmetic surgery." Which response by the nurse is most appropriate? a. "Perhaps you would like to talk to a counselor about your body image." b. "The important thing is that your weight loss is improving your health." c. "The skinfolds will gradually disappear once most of the weight is lost." d. "Cosmetic surgery is certainly a possibility once your weight has stabilized."

D Reconstructive surgery may be used to eliminate excess skinfolds after at least a year has passed since the surgery. Skinfolds may not disappear over time, especially in older patients. The response, "The important thing is that your weight loss is improving your health" ignores the patient's concerns about appearance and implies that the nurse knows what is important. Whereas it may be helpful for the patient to talk to a counselor, it is more likely to be helpful to know that cosmetic surgery is available. DIF: Cognitive Level: Application REF: 959 TOP: Nursing Process: Implementation

5. A few months after bariatric surgery, a 56-yr-old male patient tells the nurse, "My skin is hanging off of me. I think I might want to surgery to remove the skinfolds." Which response by the nurse is most appropriate? a. "The important thing is that you are improving your health." b. "The skinfolds show everyone how much weight you have lost." c. "Perhaps you should talk to a counselor about your body image." d. "Cosmetic surgery may be possible once your weight has stabilized."

D Reconstructive surgery may be used to eliminate excess skinfolds after at least a year has passed since the surgery. The responses, "The important thing is that your weight loss is improving your health," and "The skinfolds show everyone how much weight you have lost, "ignore the patient's concerns about appearance and implies that the nurse knows what is important. It may be helpful for the patient to talk to a counselor, however, there is no indication given that the concern about skinfolds is dysfunctional. DIF: Cognitive Level: Apply (application) REF: 889 TOP: Nursing Process: Implementation

45. A patient with Crohn's disease who is taking infliximab (Remicade) calls the nurse in the outpatient clinic about all of these symptoms. Which symptom is most important to communicate to the health care provider? a. Nausea b. Joint pain c. Frequent headaches d. Elevated temperature

D Since infliximab suppresses immune function, rapid treatment of infection is essential. The other patient complaints are common side effects of the medication, but do not indicate any potentially life-threatening complications. DIF: Cognitive Level: Application REF: 1026-1027 OBJ: Special Questions: Prioritization TOP: Nursing Process: Evaluation

11. A nurse cares for a client who has kidney stones from secondary hyperoxaluria. Which medication should the nurse anticipate administering? a. Phenazopyridine (Pyridium) b. Propantheline (Pro-Banthine) c. Tolterodine (Detrol LA) d. Allopurinol (Zyloprim)

D Stones caused by secondary hyperoxaluria respond to allopurinol (Zyloprim). Phenazopyridine is given to clients with urinary tract infections. Propantheline is an anticholinergic. Tolterodine is an anticholinergic with smooth muscle relaxant properties. DIF: Applying/Application REF: 1386 KEY: Urolithiasis| medications

29. The health care provider prescribes antacids and sucralfate (Carafate) for treatment of a patient's peptic ulcer. The nurse will teach the patient to take a. sucralfate at bedtime and antacids before each meal. b. sucralfate and antacids together 30 minutes before meals. c. antacids 30 minutes before each dose of sucralfate is taken. d. antacids after meals and sucralfate 30 minutes before meals.

D Sucralfate is most effective when the pH is low and should not be given with or soon after antacids. Antacids are most effective when taken after eating. Administration of sucralfate 30 minutes before eating and antacids just after eating will ensure that both drugs can be most effective. The other regimens will decrease the effectiveness of the medications. DIF: Cognitive Level: Understand (comprehension) REF: 914 TOP: Nursing Process: Implementation

35. The nurse observes nursing assistive personnel (NAP) taking the following actions when caring for a patient with a retention catheter. Which action requires that the nurse intervene? a. Taping the catheter to the skin on the patient's upper inner thigh b. Cleaning around the patient's urinary meatus with soap and water c. Using an alcohol-based hand cleaner before performing catheter care d. Disconnecting the catheter from the drainage tube to obtain a specimen

D The catheter should not be disconnected from the drainage tube because this increases the risk for urinary tract infection (UTI). The other actions are appropriate and do not require any intervention. DIF: Cognitive Level: Application REF: 1152-1154 OBJ: Special Questions: Delegation TOP: Nursing Process: Assessment

2. A client has returned to the nursing unit after an open Nissen fundoplication. The client has an indwelling urinary catheter, a nasogastric (NG) tube to low continuous suction, and two IVs. The nurse notes bright red blood in the NG tube. What action should the nurse take first? a. Document the findings in the chart. b. Notify the surgeon immediately. c. Reassess the drainage in 1 hour. d. Take a full set of vital signs.

D The drainage in the NG tube should initially be brown with old blood. The presence of bright red blood indicates bleeding. The nurse should take a set of vital signs to assess for shock and then notify the surgeon. Documentation should occur but is not the first thing the nurse should do. The nurse should not wait an additional hour to reassess. DIF: Applying/Application REF: 1116 KEY: Gastrointestinal disorders| postoperative nursing| nursing assessment

17. A nurse cares for a client who is scheduled for the surgical creation of an ileal conduit. The client states, "I am anxious about having an ileal conduit. What is it like to have this drainage tube?" How should the nurse respond? a. "I will ask the provider to prescribe you an antianxiety medication." b. "Would you like to discuss the procedure with your doctor once more?" c. "I think it would be nice to not have to worry about finding a bathroom." d. "Would you like to speak with someone who has an ileal conduit?"

D The goal for the client who is scheduled to undergo a procedure such as an ileal conduit is to have a positive self-image and a positive attitude about his or her body. Discussing the procedure candidly with someone who has undergone the same procedure will foster such feelings, especially when the current client has an opportunity to ask questions and voice concerns to someone with first-hand knowledge. Medications for anxiety will not promote a positive self-image and a positive attitude, nor will discussing the procedure once more with the physician or hearing the nurse's opinion. DIF: Applying/Application REF: 1390 KEY: Urothelial cancer| psychosocial response| coping

14. A nurse assesses clients on the medical-surgical unit. Which client is at greatest risk for bladder cancer? a. A 25-year-old female with a history of sexually transmitted diseases b. A 42-year-old male who has worked in a lumber yard for 10 years c. A 55-year-old female who has had numerous episodes of bacterial cystitis d. An 86-year-old male with a 50-pack-year cigarette smoking history

D The greatest risk factor for bladder cancer is a long history of tobacco use. The other factors would not necessarily contribute to the development of this specific type of cancer. DIF: Remembering/Knowledge REF: 1388 KEY: Urothelial cancer| health screening

21. Which action will the nurse include in the plan of care for a patient who has had a ureterolithotomy and has a left ureteral catheter and a urethral catheter in place? a. Provide education about home care for both catheters. b. Apply continuous steady tension to the ureteral catheter. c. Clamp the ureteral catheter unless output from the urethral catheter stops. d. Call the health care provider if the ureteral catheter output drops suddenly.

D The health care provider should be notified if the ureteral catheter output decreases since obstruction of this catheter may result in an increase in pressure in the renal pelvis. Tension on the ureteral catheter should be avoided in order to prevent catheter displacement. To avoid pressure in the renal pelvis, the catheter is not clamped. Since the patient is not usually discharged with a ureteral catheter in place, patient teaching about both catheters is not needed. DIF: Cognitive Level: Application REF: 1153-1154 TOP: Nursing Process: Planning

6. After sleeve gastrectomy, a 42-yr-old male patient returns to the surgical nursing unit with a nasogastric tube to low, intermittent suction and a patient-controlled analgesia (PCA) machine for pain control. Which nursing action should be included in the postoperative plan of care? a. Offer sips of fruit juices at frequent intervals. b. Irrigate the nasogastric (NG) tube frequently. c. Remind the patient that PCA use may slow the return of bowel function. d. Support the surgical incision during patient coughing and turning in bed.

D The incision should be protected from strain to decrease the risk for wound dehiscence. The patient should be encouraged to use the PCA because pain control will improve the cough effort and patient mobility. NG irrigation may damage the suture line or overfill the stomach pouch. Sugar-free clear liquids are offered during the immediate postoperative time to decrease the risk for dumping syndrome. DIF: Cognitive Level: Apply (application) REF: 888 TOP: Nursing Process: Planning

8. A patient returns to the surgical nursing unit following a vertical banded gastroplasty with a nasogastric tube to low, intermittent suction and a patient-controlled analgesia (PCA) machine for pain control. Which nursing action should be included in the postoperative plan of care? a. Irrigate the nasogastric (NG) tube frequently with normal saline. b. Offer sips of sweetened liquids at frequent intervals. c. Remind the patient that PCA use may slow the return of bowel function. d. Support the surgical incision during patient coughing and turning in bed.

D The incision should be protected from strain to decrease the risk for wound dehiscence. The patient should be encouraged to use the PCA since pain control will improve cough effort and patient mobility. NG irrigation may damage the suture line or overfill the stomach pouch. Sugar-free clear liquids are offered during the immediate postoperative time to decrease the risk for dumping syndrome. DIF: Cognitive Level: Application REF: 958-959 TOP: Nursing Process: Planning

13. The nurse is performing an admission assessment on a 20-year-old college student who is being admitted for electrolyte disorders of unknown etiology. Which assessment finding is most important to report to the health care provider? a. The patient's knuckles are macerated. b. The patient uses laxatives on a daily basis. c. The patient has a history of weight fluctuations. d. The patient's serum potassium level is 2.9 mEq/L.

D The low serum potassium level may cause life-threatening cardiac dysrhythmias and potassium supplementation is needed rapidly. The other information also will be reported because it suggests that bulimia may be the etiology of the patient's electrolyte disturbances, but it does not suggest imminent life-threatening complications. DIF: Cognitive Level: Application REF: 940-941 OBJ: Special Questions: Prioritization TOP: Nursing Process: Assessment

14. A 20-yr-old woman is being admitted with electrolyte disorders of unknown etiology. Which assessment finding is most important to report to the health care provider? a. The patient uses laxatives daily. b. The patient's knuckles are macerated. c. The patient has a history of extreme fluctuations. d. The patient's serum potassium level is 2.9 mEq/L.

D The low serum potassium level may cause life-threatening cardiac dysrhythmias, and potassium supplementation is needed rapidly. The other information will also be reported because it suggests that bulimia may be the etiology of the patient's electrolyte disturbances, but it does not suggest imminent life-threatening complications. DIF: Cognitive Level: Analyze (analysis) REF: 871 OBJ: Special Questions: Prioritization TOP: Nursing Process: Assessment

39. A patient who is vomiting bright red blood is admitted to the emergency department. Which assessment should the nurse perform first? a. Checking the level of consciousness b. Measuring the quantity of any emesis c. Auscultating the chest for breath sounds d. Taking the blood pressure (BP) and pulse

D The nurse is concerned about blood loss and possible hypovolemic shock in a patient with acute gastrointestinal (GI) bleeding; BP and pulse are the best indicators of these complications. The other information also is important to obtain, but BP and pulse rate are the best indicators for hypoperfusion. DIF: Cognitive Level: Application REF: 982-983 OBJ: Special Questions: Prioritization TOP: Nursing Process: Assessment

39. Which assessment should the nurse perform first for a patient who just vomited bright red blood? a. Measuring the quantity of emesis b. Palpating the abdomen for distention c. Auscultating the chest for breath sounds d. Taking the blood pressure (BP) and pulse

D The nurse is concerned about blood loss and possible hypovolemic shock in a patient with acute gastrointestinal bleeding. BP and pulse are the best indicators of these complications. The other information is important to obtain, but BP and pulse rate are the best indicators for assessing intravascular volume. DIF: Cognitive Level: Analyze (analysis) REF: 924 OBJ: Special Questions: Prioritization TOP: Nursing Process: Assessment

21. A nurse cares for a client who has a family history of colon cancer. The client states, "My father and my brother had colon cancer. What is the chance that I will get cancer?" How should the nurse respond? a. "If you eat a low-fat and low-fiber diet, your chances decrease significantly." b. "You are safe. This is an autosomal dominant disorder that skips generations." c.. "Preemptive surgery and chemotherapy will remove cancer cells and prevent cancer." d. "You should have a colonoscopy more frequently to identify abnormal polyps early."

D The nurse should encourage the client to have frequent colonoscopies to identify abnormal polyps and cancerous cells early. The abnormal gene associated with colon cancer is an autosomal dominant gene mutation that does not skip a generation and places the client at high risk for cancer.. Changing the client's diet, preemptive chemotherapy, and removal of polyps will decrease the client's risk but will not prevent cancer. However, a client at risk for colon cancer should eat a low-fat and high-fiber diet. DIF: Applying/Application REF: 1155 KEY: Colorectal cancer| genetics

17. When developing a weight reduction plan for an obese patient who wants to lose weight, which question should the nurse ask first? a. "Which food types do you like best?" b. "How long have you been overweight?" c. "What kind of physical activities do you enjoy?" d. "What factors do you think led to your obesity?"

D The nurse should obtain information about the patient's perceptions of the reasons for the obesity to develop a plan individualized to the patient. The other information also will be obtained from the patient, but the patient is more likely to make changes when the patient's beliefs are considered in planning. DIF: Cognitive Level: Application REF: 950 OBJ: Special Questions: Prioritization TOP: Nursing Process: Assessment

31. A patient with a recent 20-pound unintended weight loss is diagnosed with stomach cancer. Which nursing action will be included in the plan of care? a. Refer the patient for hospice services. b. Infuse IV fluids through a central line. c. Teach the patient about antiemetic therapy. d. Offer supplemental feedings between meals.

D The patient data indicate a poor nutritional state and improvement in nutrition will be helpful in improving response to therapies such as surgery, chemotherapy, or radiation. Nausea and vomiting are not common clinical manifestations of stomach cancer. There is no indication that the patient requires hospice or IV fluid infusions. DIF: Cognitive Level: Application REF: 999 | 1000-1001 TOP: Nursing Process: Planning

31. A 73-yr-old patient is diagnosed with stomach cancer after an unintended 20-lb weight loss. Which nursing action will be included in the plan of care? a. Refer the patient for hospice services. b. Infuse IV fluids through a central line. c. Teach the patient about antiemetic therapy. d. Offer supplemental feedings between meals.

D The patient data indicate a poor nutritional state and improvement in nutrition will be helpful in improving the response to therapies such as surgery, chemotherapy, or radiation. Nausea and vomiting are not common clinical manifestations of stomach cancer. There is no indication that the patient requires hospice or IV fluid infusions. DIF: Cognitive Level: Apply (application) REF: 920 TOP: Nursing Process: Planning

17. The nurse receives change-of-shift report about the following four patients. Which patient will the nurse assess first? a. A patient who has malnutrition associated with 4+ generalized pitting edema b. A patient whose parenteral nutrition has 10 mL of solution left in the infusion bag c. A patient whose gastrostomy tube is plugged after crushed medications were given through the tube d. A patient who is receiving continuous enteral feedings and has new-onset crackles throughout the lungs

D The patient data suggest aspiration has occurred and rapid assessment and intervention are needed. The other patients also should be assessed as quickly as possible, but the data about them do not suggest any immediately life-threatening complications. DIF: Cognitive Level: Application REF: 932-934 OBJ: Special Questions: Multiple Patients TOP: Nursing Process: Planning

25. A patient with bladder cancer is scheduled for intravesical chemotherapy. In preparation for the treatment the nurse will teach the patient about a. premedicating to prevent nausea. b. where to obtain wigs and scarves. c. the importance of oral care during treatment. d. the need to empty the bladder before treatment.

D The patient will be asked to empty the bladder before instillation of the chemotherapy. Systemic side effects are not experienced with intravesical chemotherapy. DIF: Cognitive Level: Application REF: 1146 TOP: Nursing Process: Implementation

44. After receiving change-of-shift report, which of the following patients should the nurse assess first? a. A patient whose new ileostomy has drained 800 mL over the previous 8 hours b. A patient with familial adenomatous polyposis who has occult blood in the stool c. A patient with ulcerative colitis who has had six liquid stools in the previous 4 hours d. A patient who has abdominal distention and an apical heart rate of 136 beats/minute

D The patient's abdominal distention and tachycardia suggest hypovolemic shock caused by problems such as peritonitis or intestinal obstruction, which will require rapid intervention. The other patients also should be assessed as quickly as possible, but the data do not indicate any life-threatening complications associated with their diagnoses. DIF: Cognitive Level: Analysis REF: 1031-1034 OBJ: Special Questions: Multiple Patients TOP: Nursing Process: Planning

43. Which patient should the nurse assess first after receiving change-of-shift report? a. A 60-yr-old patient whose new ileostomy has drained 800 mL over the previous 8 hours b. A 50-yr-old patient with familial adenomatous polyposis who has occult blood in the stool c. A 40-yr-old patient with ulcerative colitis who has had six liquid stools in the previous 4 hours d. A 30-yr-old patient who has abdominal distention and an apical heart rate of 136 beats/minute

D The patient's abdominal distention and tachycardia suggest hypovolemic shock caused by problems such as peritonitis or intestinal obstruction, which will require rapid intervention. The other patients should also be assessed as quickly as possible, but the data do not indicate any life-threatening complications associated with their diagnoses. DIF: Cognitive Level: Analyze (analysis) REF: 938 OBJ: Special Questions: Prioritization | Special Questions: Multiple Patients TOP: Nursing Process: Planning

43. A hospitalized patient who has been taking antibiotics for several days develops watery diarrhea. Which action should the nurse take first? a. Notify the health care provider. b. Obtain a stool specimen for analysis. c. Provide education about handwashing. d. Place the patient on contact precautions.

D The patient's history and new onset diarrhea suggest a C. difficile infection, which requires implementation of contact precautions to prevent spread of the infection to other patients. The other actions also are appropriate but can be accomplished after contact precautions are implemented. DIF: Cognitive Level: Application REF: 1008-1010 OBJ: Special Questions: Prioritization TOP: Nursing Process: Implementation

42. After several days of antibiotic therapy, an older hospitalized patient develops watery diarrhea. Which action should the nurse take first? a. Notify the health care provider. b. Obtain a stool specimen for analysis. c. Teach the patient about handwashing. d. Place the patient on contact precautions.

D The patient's history and new onset diarrhea suggest a C. difficile infection, which requires implementation of contact precautions to prevent spread of the infection to other patients. The other actions are also appropriate but can be accomplished after contact precautions are implemented. DIF: Cognitive Level: Analyze (analysis) REF: 932 OBJ: Special Questions: Prioritization TOP: Nursing Process: Implementation

10. A patient who has a wound infection after major surgery has only been taking in about 50% to 75% of the ordered meals and states, "Nothing on the menu really appeals to me." Which action by the nurse will be most effective in improving the patient's oral intake? a. Make a referral to the dietician. b. Order at least six small meals daily. c. Teach the patient about high-calorie, high-protein foods. d. Have family members bring in favorite foods from home.

D The patient's statement that the hospital foods are unappealing indicates that favorite home-cooked foods might improve intake. The other interventions also may help improve the patient's intake, but the most effective action will be to offer the patient more appealing foods. DIF: Cognitive Level: Application REF: 929 TOP: Nursing Process: Implementation

10. A 60-yr-old man who is hospitalized with an abdominal wound infection has been eating very little and states, "Nothing on the menu sounds good." Which action by the nurse will be most effective in improving the patient's oral intake? a. Order six small meals daily. b. Make a referral to the dietitian. c. Teach the patient about high-calorie foods. d. Have family members bring favorite foods.

D The patient's statement that the hospital foods are unappealing indicates that favorite home-cooked foods might improve intake. The other interventions may also help improve the patient's intake, but the most effective action will be to offer the patient more appealing foods. DIF: Cognitive Level: Analyze (analysis) REF: 862 TOP: Nursing Process: Implementation

37. A patient is admitted to the emergency department with severe abdominal pain with rebound tenderness. The vital signs include temperature 101° F (38.3° C), pulse 130, respirations 34, and blood pressure (BP) 84/50. Which of the following interventions should the nurse implement first? a. Administer IV ketorolac (Toradol) 15 mg. b. Draw blood for a complete blood count (CBC). c. Obtain a computed tomography (CT) scan of the abdomen. d. Infuse 1000 mL of lactated Ringer's solution over 30 minutes.

D The priority for this patient is to treat the patient's hypovolemic shock with fluid infusion. The other actions should be implemented after starting the fluid infusion. DIF: Cognitive Level: Application REF: 1016 OBJ: Special Questions: Prioritization TOP: Nursing Process: Implementation

8. A patient with recurring heartburn receives a new prescription for esomeprazole (Nexium). In teaching the patient about this medication, the nurse explains that this drug a. neutralizes stomach acid and provides relief of symptoms in a few minutes. b. reduces the reflux of gastric acid by increasing the rate of gastric emptying. c. coats and protects the lining of the stomach and esophagus from gastric acid. d. treats gastroesophageal reflux disease by decreasing stomach acid production.

D The proton pump inhibitors decrease the rate of gastric acid secretion. Promotility drugs such as metoclopramide (Reglan) increase the rate of gastric emptying. Cryoprotective medications such as sucralfate (Carafate) protect the stomach. Antacids neutralize stomach acid and work rapidly. DIF: Cognitive Level: Comprehension REF: 973-974 TOP: Nursing Process: Implementation

8. The nurse explaining esomeprazole (Nexium) to a patient with recurring heartburn describes that the medication a. reduces gastroesophageal reflux by increasing the rate of gastric emptying. b. neutralizes stomach acid and provides relief of symptoms in a few minutes. c. coats and protects the lining of the stomach and esophagus from gastric acid. d. treats gastroesophageal reflux disease by decreasing stomach acid production.

D The proton pump inhibitors decrease the rate of gastric acid secretion. Promotility drugs such as metoclopramide (Reglan) increase the rate of gastric emptying. Cryoprotective medications such as sucralfate (Carafate) protect the stomach. Antacids neutralize stomach acid and work rapidly. DIF: Cognitive Level: Understand (comprehension) REF: 903 TOP: Nursing Process: Implementation

22. A patient is transferred from the recovery room to a surgical unit after a transverse colostomy. The nurse observes the stoma to be deep pink with edema and a small amount of sanguineous drainage. The nurse should a. place ice packs around the stoma. b. notify the surgeon about the stoma. c. monitor the stoma every 30 minutes. d. document stoma assessment findings.

D The stoma appearance indicates good circulation to the stoma. There is no indication that surgical intervention is needed or that frequent stoma monitoring is required. Swelling of the stoma is normal for 2 to 3 weeks after surgery, and an ice pack is not needed. DIF: Cognitive Level: Apply (application) REF: 960 TOP: Nursing Process: Implementation

13. Which of these patients in the clinic will the nurse plan to teach about risks associated with obesity? a. Patient who has a BMI of 18 kg/m2 b. Patient with a waist circumference 34 inches (86 cm) c. Patient who has a body mass index (BMI) of 24 kg/m2 d. Patient whose waist measures 30 in (75 cm) and hips measure 34 in (85 cm)

D The waist-to-hip ratio for this patient is 0.88, which exceeds the recommended level of <0.80. A BMI of 24 kg/m2 is normal. Health risks associated with obesity increase in women with a waist circumference larger than 35 in (89 cm) and men with a waist circumference larger than 40 in (102 cm). A patient with a BMI of 18 kg/m2 is considered underweight. DIF: Cognitive Level: Comprehension REF: 944-945 TOP: Nursing Process: Planning

11. A client asks the nurse about drugs for weight loss. What response by the nurse is best? a. "All weight-loss drugs can cause suicidal ideation." b. "No drugs are currently available for weight loss." c. "Only over-the-counter medications are available." d. "There are three drugs currently approved for this."

D There are three drugs available by prescription for weight loss, including orlistat (Xenical), lorcaserin (Belviq), and phentermine-topiramate (Qsymia). Suicidal thoughts are possible with lorcaserin and phentermine-topiramate.. Orlistat is also available in a reduced-dose over-the-counter formulation. DIF: Understanding/Comprehension REF: 1249 KEY: Nutritional disorders| obesity| anorectic drugs

5. A client is being taught about drug therapy for Helicobacter pylori infection. What assessment by the nurse is most important? a. Alcohol intake of 1 to 2 drinks per week b. Family history of H. pylori infection c. Former smoker still using nicotine patches d. Willingness to adhere to drug therapy

D Treatment for this infection involves either triple or quadruple drug therapy, which may make it difficult for clients to remain adherent. The nurse should assess the client's willingness and ability to follow the regimen. The other assessment findings are not as critical. DIF: Applying/Application REF: 1127 KEY: Gastrointestinal disorders| nursing assessment| anti-ulcer therapy

15. A patient who is nauseated and vomiting up blood-streaked fluid is admitted to the hospital with acute gastritis. To determine possible risk factors for gastritis, the nurse will ask the patient about a. the amount of fat in the diet. b. history of recent weight gain or loss. c. any family history of gastric problems. d. use of nonsteroidal anti-inflammatory drugs (NSAIDs).

D Use of an NSAID is associated with damage to the gastric mucosa, which can result in acute gastritis. Family history, recent weight gain or loss, and fatty foods are not risk factors for acute gastritis. DIF: Cognitive Level: Comprehension REF: 985-986 TOP: Nursing Process: Assessment

15. A patient vomiting blood-streaked fluid is admitted to the hospital with acute gastritis. To determine possible risk factors for gastritis, the nurse will ask the patient about a. the amount of saturated fat in the diet. b. a family history of gastric or colon cancer. c. a history of a large recent weight gain or loss. d. use of nonsteroidal antiinflammatory drugs (NSAIDs).

D Use of an NSAID is associated with damage to the gastric mucosa, which can result in acute gastritis. Family history, recent weight gain or loss, and fatty foods are not risk factors for acute gastritis. DIF: Cognitive Level: Understand (comprehension) REF: 909 TOP: Nursing Process: Assessment

2. The nurse determines that instruction regarding prevention of future urinary tract infections (UTIs) for a patient with cystitis has been effective when the patient states, a. "I can use vaginal sprays to reduce bacteria." b. "I will drink a quart of water or other fluids every day." c. "I will wash with soap and water before sexual intercourse." d. "I will empty my bladder every 3 to 4 hours during the day."

D Voiding every 3 to 4 hours is recommended to prevent UTIs. Use of vaginal sprays is dis- couraged. The bladder should be emptied before and after intercourse, but cleaning with soap and water is not necessary. A quart of fluids is insufficient to provide adequate urine output to decrease risk for UTI. DIF: Cognitive Level: Application REF: 1125 TOP: Nursing Process: Evaluation

2. The nurse determines that instruction regarding prevention of future urinary tract infections (UTIs) has been effective for a 22-yr-old female patient with cystitis when the patient states which of the following? a. "I can use vaginal antiseptic sprays to reduce bacteria." b. "I will drink a quart of water or other fluids every day." c. "I will wash with soap and water before sexual intercourse." d. "I will empty my bladder every 3 to 4 hours during the day."

D Voiding every 3 to 4 hours is recommended to prevent UTIs. Use of vaginal sprays is discouraged. The bladder should be emptied before and after intercourse, but cleaning with soap and water is not necessary to prevent UTI. A quart of fluids is insufficient to provide adequate urine output to decrease risk for UTI. DIF: Cognitive Level: Apply (application) REF: 1038 TOP: Nursing Process: Evaluation

18. On the first postoperative day the nurse is caring for a patient who has had a Roux-en-Y gastric bypass procedure. Which assessment finding should be reported immediately to the surgeon? a. Use of patient-controlled analgesia (PCA) several times an hour for pain b. Irritation and skin breakdown in skinfolds c. Bilateral crackles audible at both lung bases d. Emesis of bile-colored fluid past the nasogastric (NG) tube

D Vomiting with an NG tube in place indicates that the NG tube needs to be repositioned by the surgeon to avoid putting stress on the gastric sutures. The nurse should implement actions to decrease skin irritation and have the patient cough and deep breathe, but these do not indicate a need for rapid notification of the surgeon. Frequent PCA use after bariatric surgery is expected. DIF: Cognitive Level: Application REF: 958-959 OBJ: Special Questions: Prioritization TOP: Nursing Process: Assessment

1. The nurse is caring for a 47-yr-old female patient who is comatose and is receiving continuous enteral nutrition through a soft nasogastric tube. The nurse notes the presence of new crackles in the patient's lungs. In which order will the nurse take action? (Put a comma and a space between each answer choice [A, B, C, D].) a. Check the patient's oxygen saturation. b. Notify the patient's health care provider. c. Measure the tube feeding residual volume. d. Stop administering the continuous feeding.

D, A, C, B The assessment data indicate that aspiration may have occurred, and the nurse's first action should be to turn off the tube feeding to avoid further aspiration. The next action should be to check the oxygen saturation because this may indicate the need for immediate respiratory suctioning or oxygen administration. The residual volume should be obtained because it provides data about possible causes of aspiration. Finally, the health care provider should be notified and informed of all the assessment data the nurse has just obtained. DIF: Cognitive Level: Analyze (analysis) REF: 865 OBJ: Special Questions: Prioritization TOP: Nursing Process: Implementation


Set pelajaran terkait

ACS - Alkanes, Alkenes, Alkynes and their Alicyclic Couterparts

View Set

Procedural Programming midterm review

View Set

applied mathematics - unit 4: probability in the social science

View Set

Evolve questions for fundamentals exam 4 04/30

View Set